Missed Practice Problems Flashcards

You may prefer our related Brainscape-certified flashcards:
1
Q

Choose from among the following utility claims and identify the claim which is not indefinite.

(a) A device as described in col. 1, line 46 - 64 of the specification.

(b)A method of painting an article, comprising:
providing the article;
spraying said article with paint;
and allowing said article to dry.

(c) The ornamental design for a device as shown and described.

(d) The ornamental design for a device, the improvement comprising: a non-stick coating provided on a cooking surface.

(e) None of the above.

A

(b)A method of painting an article, comprising:
providing the article;
spraying said article with paint;
and allowing said article to dry.

Reasoning: Method claims are under utility patents.

How well did you know this?
1
Not at all
2
3
4
5
Perfectly
2
Q

Which of the following statements is not false?

(a) Claims in a patent application cannot be rejected when the disclosure fails to adequately disclose the invention.

(b) A claim cannot be rejected for failing to interconnect elements of the invention, when it is clear to one of ordinary skill where the elements would go.

(c) A claim can be rejected for including improper connection, even if undue experimentation is not required to perform the invention.

(d) A claim can be rejected as being vague and indefinite when all of the words and phrases can be found and defined in the supporting specification, if the examiner believes more definite words could have been used in the field.

(e) C and D.

A

(c) A claim can be rejected for including improper connection, even if undue experimentation is not required to perform the invention.

Reasoning: You can reject a claim for being improperly connected. Everything has to connect back to the spec.

How well did you know this?
1
Not at all
2
3
4
5
Perfectly
3
Q

In an After Final amendment amendments are made which place the case in condition for allowance. Because this amendment will place the case in condition for allowance:

(a) The remarks do not have to request reconsideration.

(b) The remarks do not have to show why the amendments were not earlier presented.

(c) No remarks are necessary.

(d) The remarks must address each and every objection and rejection of the examiner.

(e) The remarks must address each and every objection and rejection of the examiner and request reconsideration.

A

(e) The remarks must address each and every objection and rejection of the examiner and request reconsideration.

Reasoning: Remarks are always necessary and and address every objection.

How well did you know this?
1
Not at all
2
3
4
5
Perfectly
4
Q

If the broad generic claim in an application includes an inoperative species of the invention___________________

(a) The claim is properly rejected under 35 USC 112(a).

(b) The claim is properly rejected under 35 USC 112(b).

(c) The claim cannot be rejected as long as other operable species of the invention are also within the scope of the claim.

(d) The claim will not be rejected as long as the inoperable species does not correspond to the best mode or a preferred embodiment of the invention.

(e) A and B.

A

(c) The claim cannot be rejected as long as other operable species of the invention are also within the scope of the claim.

Reasoning: Every generic claim has inoperative species. As long as one operative species is within the scope, it is fine.

How well did you know this?
1
Not at all
2
3
4
5
Perfectly
5
Q

The application as filed includes the following series of claims.

  1. A radio transmitter, comprising:

a transistor
a resistor electrically connected to said transistor, and
a switch electrically connected to said resistor and a power source, for switching said radio on and off.

  1. A radio as in claim 1, further comprising:

a loudspeaker

  1. A loudspeaker as in claim 1 or 2, wherein:

said loudspeaker has a rating of 20 watts.

  1. A radio as in any one of the preceding claims, wherein:

said power source is a battery

A British reference discloses a radio having a micro-processor having all of the functions of the components recited in the claims, and has a publication date more than one year prior to the filing date of the application. From among the filed claims, choose the claims which are in an acceptable format.

(a) Claim 2.

(b) Claim 3.

(c) Claim 4.

(d) All of the claims.

(e) Claim 1.

A

(e) Claim 1.

Reasoning: The only good claim is claim 1 because it is complete. Claim 2 isn’t good because it isn’t connected to the radio. It is just sitting there on the table. Claim 3 has the preamble “loudspeaker” when that isn’t the subject of claim 1 or 2, making the preamble bad. Claim 4 is a multiple dependent claim linked to bad claims, making it unacceptable.

How well did you know this?
1
Not at all
2
3
4
5
Perfectly
6
Q

Claim 1 in the application is directed to an apparatus. Claim 2 is directed to the same apparatus and further defines the apparatus. Claim 3 is directed to a method of using the apparatus set forth in claims 1 and 2. Claim 4 is an independent method claim directed to a method of using the apparatus. A rejection/objection of these claims would be:

(a) Claim 3 is rejected under 35 USC 112(b), for being in improper multiple dependent form.

(b) Claim 3 is objected to as being in improper multiple dependent form and is rejected under 35 USC 112(b), as being indefinite for mixing claim types.

(c) Claim 4 is rejected under 35 USC 112(b), for mixing claims types in an application.

(d) All of the claims are rejected under 112(a), as lacking antecedent in the specification.

(e) B and C.

A

(b) Claim 3 is objected to as being in improper multiple dependent form and is rejected under 35 USC 112(b), as being indefinite for mixing claim types.

Reasoning: Claim 3 says “claims 1 and 2” when it should say “claims 1 or 2”. Additionally, claim 3 is a method claim while the others are directed towards an apparatus, which can’t be done.

How well did you know this?
1
Not at all
2
3
4
5
Perfectly
7
Q

An application is filed describing a series of species of the invention in the specification. A generic claim covering the various species is the only claim in the application. The claim is rejected under 35 USC 103 based on the broadest reasonable interpretation of the claim in light of the Abstract of the Disclosure. In response to this rejection a proper argument would be:

(a) Seek reconsideration on the basis that the Abstract may not be used as a basis to determine the scope of the claim in question.

(b)Amend the generic claim to avoid the prior art.

(c)File an Affidavit under 37 CFR 1.132 showing unexpected results regarding several of the species of the invention also falling within the scope of the claim.

(d) File an Affidavit under 1.131 swearing behind the reference in question.

(e) Cancel the Abstract and submit a rewritten Abstract which excludes the rejected subject matter.

A

(b)Amend the generic claim to avoid the prior art.

Reasoning: Can rely on abstract for prior art.

How well did you know this?
1
Not at all
2
3
4
5
Perfectly
8
Q

Your client comes into your office with an ashen face following receipt of the patent you obtained for her. The claims barely leave out an important disclosed but sadly unclaimed invention because of inartful narrowness in the claims. That very invention has been copied by her ex-husband’s company and is being sold widely. You quickly explain the basis for filing a correction to the patent. You make five statements. Which, if any, are incorrect?

(a) She can file either an ex parte reexamination or reissue at this early date to fix the claims. A reexam is better since the claims are already being infringed.

(b) She can file a reissue to broaden the claims to include subject matter given up to avoid prior art, but she must act quickly to file within two years of the issue date.

(c) She can file a reissue, but not an ex parte reexamination, and she must act within two years to file the reissue application.

(d) A certificate of correction is not possible to correct the defect in this patent.

(e) A and B.

A

(e) A and B.

Reasoning: She can only file a reissue, not an ex-parte reexam. Can’t ever recover broaden claims using stuff given up

How well did you know this?
1
Not at all
2
3
4
5
Perfectly
9
Q

The application is filed in a foreign language and later translated into English with an appropriate certification. To your horror, the application is not enabling with respect to one of the two inventions falling within the scope of the only generic claim in the application. Your client’s invention is being actively copied by the United States Government at a missile test facility in Kansas. Prior art disclosing the identical invention published in Missile Weekly by Werner von Rocket between the priority date and the U. S. filing date of your application. Your client insists the application must issue quickly.

(a) You advise your client the application is fatally defective and should be abandoned.

(b) Since the prior art is identical, you advise your client to abandon the application.

(c) You cancel the claims, including those to the non-enabled invention, through amendment.

(d) You cancel the generic claim and submit a new claim directed to the enabled version of the invention.

(e) You file a CIP adding the enabling disclosure previously left out to enable the invention as to the scope of the generic claim.

A

(d) You cancel the generic claim and submit a new claim directed to the enabled version of the invention.

Reasoning: Don’t just give up. See what you can claim, and claim it. A CIP adds things and it doesn’t help with the prior-art.

How well did you know this?
1
Not at all
2
3
4
5
Perfectly
10
Q

The invention is to a method of cleaning a surface of a crystal prior to etching the surface. The cleaning method requires first placing the crystal in an evacuated chamber, exposing the crystal to a pure inert gas at a temperature above 500 degrees Celcius, and thereafter allowing the crystal to cool and removing the crystal from the chamber.

The prior art discloses a method of cleaning a crystal which includes the steps of placing the crystal in a pressure vessel containing a pure inert gas, heating the vessel to a temperature of not less than 500 degrees Celcius, and rapidly quenching the crystal in an ammonia solution which is sprayed under pressure into the pressure vessel. A patentable claim to the method would be:

(a) A method of cleaning a crystal prior to etching, comprising the steps of: placing the crystal into a vessel containing a pure inert gas, heating said crystal and gas to a temperature above 500 degrees Celcius, and thereafter, quenching said crystal.

(b) A method of cleaning a crystal wherein said crystal is placed into an evacuated chamber and is exposed to an inert gas at a temperature above 500 degrees Celcius and then allowed to cool whereupon the crystal is removed from the chamber and is etched.

(c) A method of preparing a crystal for etching wherein the evacuated chamber surrounds the crystal and an inert gas is placed into the chamber at a temperature of more than 500 degrees and cooling is then allowed to occur and the crystal is etched.

(d) A method of cleaning a crystal, comprising the steps of: placing said crystal into an evacuated chamber, exposing said crystal to a pure inert gas at a temperature above 500 degrees Celcius, allowing said crystal to cool, and, removing said crystal from said chamber.

(e) An apparatus for cleaning a crystal comprising: an evacuated chamber, a source of inert gas, a heater, and a quenching spray device connected to said chamber.

A

(d) A method of cleaning a crystal, comprising the steps of: placing said crystal into an evacuated chamber, exposing said crystal to a pure inert gas at a temperature above 500 degrees Celcius, allowing said crystal to cool, and, removing said crystal from said chamber.

Reasoning: Need the verbs to be in -ing form. This answer matches the verbs in the prompt.

How well did you know this?
1
Not at all
2
3
4
5
Perfectly
11
Q

The invention is an assembled CD holder which includes a cover member hingedly connected to a backing member. Inside the cover and backing member, the CD holder includes a snap connected to the inside surface of the cover member which retains the outer rim edge of the CD which is to be placed inside. The prior art is a conventional CD holder which includes a cover member hingedly connected to a backing member. Inside the CD cover, in the space between the cover and the backing member, is a snap which retains the CD by gripping the inner rim of the through passage in the center of the CD. You are asked to write the claims for the invented CD cover for your friend the computer wiz, and you write the following claims. Which, if any, are not indefinite.

(a) A CD holder comprising: a cover member, a backing member connected to said cover member, and a snap member inside said cover and backing member which retains an outer edge of a CD which is inserted into the CD cover.

(b) A combination CD holder, comprising a cover member hingedly connected to a backing member, and a snap adapted to retain said CD by retaining an outer rim edge of said CD.

(c) A CD holder comprising: snap means for holding an outer rim edge of a CD, said snap means attached inside a space, defined between a cover member which is hingedly connected to a backing member, to an inwardly directed surface of said cover member.

(d) A CD adapted for placement into a holder that snaps to its outer rim.

(e) A CD holder consisting of a cover member hingedly connected to a backing member and having therein a snap member attached thereto which retains an outer edge of a CD.

A

(c) A CD holder comprising: snap means for holding an outer rim edge of a CD, said snap means attached inside a space, defined between a cover member which is hingedly connected to a backing member, to an inwardly directed surface of said cover member.

Reasoning: (b) is bad because it isn’t connected. In (d) it is unsure where the snap is connected.

How well did you know this?
1
Not at all
2
3
4
5
Perfectly
12
Q

Your client has discovered in a remote part of the South American jungle a plant called planti cokius which has remarkable curative properties when the plant is chopped, cooked, and combined with equal parts of Jack Daniels brand 100 proof sour mash whiskey. Your client claims it can cure most anything but has not performed any tests which would not result in undue experimentation to discover the dosage. Instead, your client seems to drink up most of the experiments, but is in a remarkable state of health. While discussing the invention at your office, your client becomes disoriented and tips over the earthen jug containing the mixture, and it spills onto what you thought to be a long dead office plant. The plant is revived within minutes and blossoms. You commence writing the application and include the description of the mixture as having remarkable curative properties, and also its use as a fertilizer. You write the following claims, some of which will not succeed at the Patent Office, but you want your client to be happy so that she will sign the oath and leave your office. Which claims will not be cancelled by your preliminary amendment because you know the Patent Office needs at least one claim to examine?

A) A curative mixture for treating human ailments, comprising: chopped, cooked planti cokius mixed with equal parts Tennessee whiskey.

B) A fertilizer prepared according to the following process: chopping and cooking planti cokius and mixing the chopped and cooked plant with whiskey.

C) A fertilizer mixture comprising: chopped and cooked planti cokius mixed in equal parts with Jack Daniels Old No. 7 Sour Mash 100 Proof Tennessee Whiskey.

D) A method of reviving a house plant, comprising: adding a predetermined amount of a combination of chopped and cooked planti cokius mixed with equal parts 100 proof whiskey, to the soil of said plant, allowing said plant to revive and blossom.

E) All of the above.

A

C) A fertilizer mixture comprising: chopped and cooked planti cokius mixed in equal parts with Jack Daniels Old No. 7 Sour Mash 100 Proof Tennessee Whiskey.

Reasoning: Claiming a fertilizer mixture mixed 50/50. A and B don’t include the specific whiskey. D doesn’t have information about the pre-determined amount.

How well did you know this?
1
Not at all
2
3
4
5
Perfectly
13
Q

To comply with 112, 1st paragraph, support requirements in the specification, the applicant cannot leave out any except one of the following:

A) At least one successful example of applying the invention for its stated utility so that undue experimentation is not required to prove utility by another in the field.

B) The best mode of practicing the invention known to the inventor at the time of filing the application.

C) A written description of sufficient adequacy to enable one of skill in the art to practice the invention.

D) A written description sufficient to make and use the invention.

E) The best mode of the invention known to co-workers of the inventor who had made this best mode known to the inventor who thought it over and agreed it was better than what he had come up with on his own.

A

A) At least one successful example of applying the invention for its stated utility so that undue experimentation is not required to prove utility by another in the field.

Reasoning: Best mode known to inventor has to be there, including those known to the co-workers. Can leave out a successful example, it can be prophetic.

How well did you know this?
1
Not at all
2
3
4
5
Perfectly
14
Q

Independently of each other, Bert and Ernie invented the same rubberized, floating toy in the United States. A U.S. Patent was granted to Bert on February 18, 2012 on an application filed on April 12, 2010, claiming the toy. On April 10, 2012, Ernie filed a patent application in the USPTO claiming the same toy. There is no common assignee. Under which of the following provisions of 35 U.S.C. 102 is the U.S. patent to Bert prior art with regard to the toy claimed by Ernie?

A) 35 U.S.C. 102(a)
B) 35 U.S.C. 102(b)
C) 35 U.S.C. 102(e)
D) A and C
E) A, B, and C

A

D) A and C

Reasoning: The AIA went into effect March 16, 2013. Bert’s patent will be examined under pre-AIA.

How well did you know this?
1
Not at all
2
3
4
5
Perfectly
15
Q

Independently of each other, Oscar and Felix invented identical ergonomic golf clubs in the United States. Oscar filed his patent application in the U.S. on April 12, 2012, and a patent issued on March 12, 2013. On April 10, 2013, Felix filed his patent application in the USPTO claiming golf clubs identical to those claimed in Oscar’s patent. There is no common assignee. Under which of the following provisions of 35 U.S.C. 102 is the U.S. patent to Oscar prior art with regard to the golf club claimed by Felix?

A) 35 U.S.C. 102(a)
B) 35 U.S.C. 102(b)
C) 35 U.S.C. 102(e)
D) A and C
E) A, B, and C

A

A) 35 U.S.C. 102(a)

Reasoning: The patent is filed after the AIA effective date, and so the issue will come from a 102(a)(1) issue.

How well did you know this?
1
Not at all
2
3
4
5
Perfectly
16
Q

Which of the following references could be used by an examiner to appropriately reject claims in a patent application filed by Jonathan Williamson under 35 U.S.C. 102(a)(2)? Assume Williamson conceived of his invention on March 1, 2013, and filed a patent application fully disclosing the invention on October 1, 2013.

A)
An article in Science Daily describing the invention in detail, which was authored by Williamson and published on August 1, 2013.

B)
An article in Scientific American describing the invention in detail, which was authored by Williamson and his colleague Carter Thomas and published on September 15, 2013.

C)
An Irish patent application publication that lists Williamson as the inventor and which published on November 1, 2013, and with the Irish filing date relied upon to support a priority claim for a PCT application that published.

D)
A published PCT claiming priority to a French patent application that lists Williamson and Carter Thomas as inventors, which was filed on June 1, 2012.

E)
None of the above.

A

D) A published PCT claiming priority to a French patent application that lists Williamson and Carter Thomas as inventors, which was filed on June 1, 2012.

Reasoning: 102(a)(2) relates to filing dates. AIA 35 U.S.C. 102(a)(2) states: “A person shall be entitled to a patent unless— the claimed invention was described in a patent issued under section 151, or in an application for patent published or deemed published under section 122(b), in which the patent or application, as the case may be, names another inventor and was effectively filed before the effective filing date of the claimed invention.”

The key to getting this question right is to know what 102(a)(2) captures as prior art. By the explicit terms of this section of the statute, 102(a)(2) captures prior-filed U.S. patent applications, as well as prior-filed patent applications that are deemed published, provided however that they must name “another inventor.”

We can rule out (A) and (B) because these relate to publications. (C) can be ruled out because the only inventor is Williamson, who is also the inventor on the U.S. patent application in question. (D) lists Williamson and Thomas as co-inventors, which satisfies the “another inventor” requirement of 102(a)(2). The French filing date will be considered a prior art date because it was relied upon in a published PCT application. Notice that the fact that the French patent application will not publish until after Williamson’s U.S. filing date does not matter since it is the French application date that creates the problem.

The key here is to notice that, if there is any difference in inventive entity between the application under examination on the one hand and either a prior art U.S. patent, U.S. patent application publication or WIPO-published PCT application, the U.S. patent, U.S. patent application publication, or WIPO-published PCT application satisfies the “names another inventor” requirement of AIA 35 U.S.C. 102(a)(2). Thus, in the case of joint inventors, only one inventor needs to be different for the inventive entities to be different.

How well did you know this?
1
Not at all
2
3
4
5
Perfectly
17
Q

In which of the following circumstances would it not be possible for an applicant who filed on December 1, 2013, to rely on a Rule 1.130 affidavit to take advantage of the exceptions of 102(b)(1)?

A)
The inventor publicly disclosed the subject matter in question via a slide presentation at a scientific meeting on January 15, 2013, while the intervening disclosure of the subject matter was made in a journal article on September 1, 2013.

B)
The party making the intervening disclosure learned of the subject matter in question after attending a lecture given by the inventor on October 1, 2012, and subsequently published an article detailing the inventor’s invention on December 15, 2012.

C)
The party making the intervening disclosure learned of the subject matter in question after attending a lecture given by the inventor on April 1, 2013, and subsequently published an article detailing the inventor’s invention on November 15, 2013.

D)
The party making the intervening disclosure learned of the subject matter in question after seeing a draft of an academic article provided to him for purpose of peer review. The draft was circulated on May 1, 2013, and an article was subsequently published detailing the inventor’s invention as described in the draft article on November 30, 2013.

E)
None of the above.

A

B) The party making the intervening disclosure learned of the subject matter in question after attending a lecture given by the inventor on October 1, 2012, and subsequently published an article detailing the inventor’s invention on December 15, 2012.

Reasoning: In (A), (C) and (D), the initial disclosure occurred within one year of filing the patent application. In (B), however, the initial disclosure happened some 14 months prior to the filing of the patent application on December 1, 2013. Thus, the applicant could not rely on a Rule 1.130 affidavit. The applicant would not be able to remove their own disclosure of October 1, 2013.

How well did you know this?
1
Not at all
2
3
4
5
Perfectly
18
Q

Michael Arthur is the inventor of a cordless jump rope. Essentially, the invention is a jump-rope simulator incorporating rotatable, counterbalancing weights within a handle and marketed to those individuals who really like to jump rope but who lack the physical prowess necessary to actually jump the rope. Arthur, who is very proud of his invention, writes up a description of the invention together with several drawings and shows them to various individuals at his health club, all of whom like the invention. The first such disclosure occurred on June 15, 2013. One fellow by the name of Bradley Brady likes it so much that he decides to file a patent application on the cordless jump rope, which occurs on November 15, 2013. Arthur subsequently files a patent application on December 15, 2013. The application filed by Brady fully discloses the Arthur invention. The application filed by Arthur includes 20 claims, 3 of which are in independent format. The claims filed by Arthur are quite different compared to the claims submitted by Brady.

Will it be possible for Arthur to overcome a rejection by a patent examiner based on the Brady application?

A)
Yes. Arthur can submit a Rule 1.130 affidavit.

B)
Yes. Arthur can submit a Rule 1.131 affidavit.

C)
Yes. Arthur can submit a Rule 1.132 affidavit.

D)
Yes, but Arthur will be required to file a petition to institute a derivation proceeding under AIA 35 U.S. C. 135.

E)
No, Arthur will not be able to remove the reference under the AIA first-to-file rules.

A

A) Yes. Arthur can submit a Rule 1.130 affidavit.

Reasoning: The issue is removing the prior art, not showing derivation, SINCE the claims are different. Derivations are about claims that are the same or similar.

How well did you know this?
1
Not at all
2
3
4
5
Perfectly
19
Q

Spencer Kelly is the inventor of an attachment to a shotgun barrel, which modifies the pattern of the shot released from the shotgun shell when it is fired. Kelly, who is very proud of his invention, writes up a description of the invention together with several drawings and shows them to various individuals at his gun club, all of whom like the invention. The first such disclosure occurred on August 22, 2013. One fellow by the name of Alex Lexington likes it so much that he decides to file a patent application on the shotgun barrel attachment, which occurs on October 22, 2013. Kelly subsequently files a patent application on January 1, 2014. The application filed by Lexington fully discloses the Kelly invention. The application filed by Kelly includes 25 claims, 5 of which are in independent format. Claims 1 – 5 filed by Kelly are identical to claims submitted by Lexington, Claims 6 – 10 filed by Kelly are substantially similar to the claims filed by Lexington, and Claims 11 – 25 are quite different when compared with any claims filed by Lexington.

Will it be possible for Kelly to overcome a rejection by a patent examiner based on the Lexington application?

A)
Yes. Kelly can submit a Rule 1.130 affidavit to overcome any rejection of claims 1 – 25.

B)
Yes. Kelly can submit a Rule 1.130 affidavit to overcome any rejection of claims 11 – 25.

C)
Yes. Kelly can submit a Rule 1.131 affidavit.

D)
Yes. Kelly can submit a Rule 1.132 affidavit.

E)
Yes, but Kelly will be required to file a petition to institute a derivation proceeding under AIA 35 U.S. C. 135.

A

B) Yes. Kelly can submit a Rule 1.130 affidavit to overcome any rejection of claims 11 – 25.

Reasoning: If he wants claims 1-10, he will have to file a derivation proceeding since they are substantially similar.

How well did you know this?
1
Not at all
2
3
4
5
Perfectly
20
Q

Under the AIA, disclosures made one year or less before the effective filing date of a claimed invention may not be prior art. Which disclosures potentially qualify for this grace period and can be removed as prior art?

I. Printed publications.

II. Issued patents.

III. Public use.

IV. Offers for sale.

A) I and II.
B) III and IV.
C) I, II and IV
D) I, III and IV.
E) I, II, III and IV.

A

E) I, II, III and IV.

Reasoning: (E) is correct. AIA 35 U.S.C. 102(a)(1) says: “A person shall be entitled to a patent unless— (1) the claimed invention was patented, described in a printed publication, or in public use, on sale, or otherwise available to the public before the effective filing date of the claimed invention….”

The Office’s interpretation of AIA 35 U.S.C. 102(a)(1) is that the AIA grace period extends to all of the documents and activities enumerated in AIA 35 U.S.C. 102(a)(1) that would otherwise defeat patentability. This interpretation avoids the very odd potential result that the applicant who had made his invention accessible to the public for up to a year before filing an application could still obtain a patent, but the inventor who merely used his invention one day before he filed an application could not obtain a patent.

How well did you know this?
1
Not at all
2
3
4
5
Perfectly
21
Q

Which of the following are required in order to obtain a filing date of a nonprovisional patent application at the United States Patent and Trademark Office?

A) At least one claim.

B) A specification describing the invention.

C) A drawing if necessary to understand the invention.

D) A, B and C.

E) B and C.

A

B) A specification describing the invention.

Reasoning: After PLTIA was enacted, all you need is a spec to get a filing date.

How well did you know this?
1
Not at all
2
3
4
5
Perfectly
22
Q

AIA 35 U.S.C. 102(a)(2) provides that a person is not entitled to a patent if the claimed invention was described in a U.S. patent, a published U.S. patent application, or an application for patent “deemed published” prior to the effective filing date of the claimed invention. Which of the following would be considered “deemed published” within the meaning of 102(a)(2)?

A) A World Intellectual Property Organization (WIPO) publication of a Patent Cooperation Treaty (PCT) international application that designates the U.S.

B) A World Intellectual Property Organization (WIPO) publication of a Patent Cooperation Treaty (PCT) international application that designates the United States, but only if subsequent to publication, the application enters the national stage in the U.S.

C) A World Intellectual Property Organization (WIPO) publication of a Patent Cooperation Treaty (PCT) international application that designates the United States, but only if the application publishes in English.

D) A World Intellectual Property Organization (WIPO) publication of a Patent Cooperation Treaty (PCT) international application that designates the U.S, but only if the application matures into an issued U.S. patent.

E) None of the above.

A

A) A World Intellectual Property Organization (WIPO) publication of a Patent Cooperation Treaty (PCT) international application that designates the U.S.

Reasoning: 35 U.S.C. 122(b). A World Intellectual Property Organization (WIPO) publication of a Patent Cooperation Treaty (PCT) international application that designates the United States is an application for patent deemed published under 35 U.S.C. 122(b) for purposes of AIA 35 U.S.C. 102(a)(2). Thus, under the AIA, WIPO publications of PCT applications that designate the United States are treated as U.S. patent application publications for prior art purposes, regardless of the international filing date, whether they are published in English, or whether the PCT international application enters the national stage in the United States. Accordingly, a U.S. patent, a U.S. patent application publication, or a WIPO publication of a PCT application (WIPO published application) that designates the United States, that names another inventor and was effectively filed before the effective filing date of the claimed invention, is prior art under AIA 35 U.S.C. 102(a)(2).

How well did you know this?
1
Not at all
2
3
4
5
Perfectly
23
Q

Jocelyn Carter is the inventor of a unique and highly concealable spy camera, which incorporates facial recognition modules. Carter files a nonprovisional patent application on April 2, 2012. The nonprovisional patent application does not contain any additional disclosure, but is filed with 21 claims, 4 of which are independent claims. On March 14, 2013, a Continuation-in-Part (CIP) is filed, further disclosing a previously undisclosed computer-implemented filtering system for sorting the recognized faces. On March 17, 2013, a continuation of the April 2, 2012, nonprovisional patent application is filed.

Which of the following accurately states which law these applications will be examined under?

I. The April 2, 2012, nonprovisional patent application will be reviewed with the examiner applying pre-AIA.

II. The March 14, 2013, CIP will be reviewed with the examiner applying pre-AIA.

III. The March 17, 2013, Continuation will be reviewed with the examiner applying AIA.

A) I
B) II
C) I and II
D) I and III
E) I, II and III

A

C) I and II

Reasoning: The nonprovisional application was filed before the March 16, 2013 AIA date. The CIP was filed pre-AIA as well. A continuation is reviewed based on the date of it’s parent application, which is pre-AIA.

How well did you know this?
1
Not at all
2
3
4
5
Perfectly
24
Q

Stanley Thaddeus Wojciehowicz is the inventor of an improved, combination coffeemaker/motion detector, which is useful for making better than mediocre coffee and doubles as a home security system. He first files a patent application in Poland on February 28, 2012. Subsequently, he files a patent application in the U.S. on February 27, 2013, which claims the benefit of his Polish filing date. The Polish application publishes on August 28, 2013, and the U.S. patent application publishes on August 29, 2013. The Polish patent is granted on March 14, 2014, and the U.S. patent is granted on October 22, 2014.

On November 1, 2014, Carl Levitt files an application claiming a combination coffeemaker/motion detector with integrated radio.

Which of the follow dates is the earliest date the Wojociehowicz disclosure will be prior art to Levitt?

A) February 28, 2012.
B) February 27, 2013.
C) August 28, 2013.
D) March 14, 2014.
E) October 22, 2014

A

A) February 28, 2012.

Reasoning: Foreign filing date will become a prior art date IF it is used in a PCT, US patent that publishes, or a US patent that issues.

How well did you know this?
1
Not at all
2
3
4
5
Perfectly
25
Q

Arnold Horshack is the inventor of a new and improved turkey call, which he conceived and reduced to practice on January 30, 2012. The device creates the most obnoxious sound, which approximates a wheezing hyena. Field studies show that turkeys appear to be drawn to the noise out of curiosity, or perhaps pity for what sounds like a dying animal. Horshack files a patent application in Canada on April 1, 2012, and on May 1, 2012, he files a U.S. patent application claiming priority to his Canadian filing date. For financial reasons, the U.S. application is expressly abandoned on July 1, 2012. The Canadian application publishes on October 1, 2013.

Freddie Washington files a U.S. nonprovisional patent application on November 1, 2013, which relates to a turkey call that approximates a wheezing coyote.

Which of the following would be the earliest date the Horshack disclosure could be used as prior art against Washington?

A) January 12, 2012.
B) April 1, 2012.
C) May 1, 2012.
D) July 1, 2012.
E) October 1, 2013.

A

E) October 1, 2013.

Reasoning: The US patent application was expressly abandoned, which means that it won’t be published. That leaves only the Canada application which publishes October 1, 2013. Since the foreign application isn’t used for priority, the publication date is the only date that matters for prior art.

How well did you know this?
1
Not at all
2
3
4
5
Perfectly
26
Q

Sebastian Wilcox files a nonprovisional patent application on December 17, 2013. The application filed contains a specification sufficient to satisfy 35 U.S.C. 112(a), but does not include any claims. In due course, the Patent Office sends a Notice of Incomplete Application, indicating that a filing date has not been awarded. What should Wilcox do?

A) Notify the Patent Office that pursuant to the Patent Law Treaty Implementation Act of 2012, a claim is no longer required at the time of filing in order to have a filing date awarded. Specifically request that a filing date of December 17, 2013, be awarded.

B) Petition the Commissioner to award a filing date in light of the Patent Law Treaty Implementation Act of 2012.

C) Convert the nonprovisional patent application into a provisional patent application because provisional patent applications do not require claims.

D) Submit one or more claims in a Preliminary Amendment together with a request to retroactively grant December 17, 2013, as the official filing date.

E) It is too late to do anything to fix this and salvage December 17, 2013 as a filing date. therefore, Wilcox should file a substitute application as soon as possible.

A

C) Convert the nonprovisional patent application into a provisional patent application because provisional patent applications do not require claims.

Reasoning: It is true that the Patent Law Treaties Implementation Act of 2012 (“PLT”) changes the law with respect to obtaining a filing date. Specifically, a claim is no longer required in order to have a filing date awarded. However, the provisions of the PLT that make it unnecessary to have a claim in order to obtain a filing date apply only to applications filed on or after December 18, 2013. Thus, A and B are wrong.

How well did you know this?
1
Not at all
2
3
4
5
Perfectly
27
Q

For a non-provisional application filed on or after March 16, 2013, that claims priority to a foreign application, the applicant is not required to provide any “transition application” statement if the non-provisional application claims only subject matter disclosed in a foreign application filed prior to March 16, 2013

T/F

A

T

How well did you know this?
1
Not at all
2
3
4
5
Perfectly
28
Q

Under pre-AIA law, the effective filing date of a claimed invention is determined on a claim-by-claim basis and not an application-by-application basis, The principle that different claims in the same application may be entitled to different effective filing dates with respect to the prior art remains unchanged by the AIA.

T/F

A

T

How well did you know this?
1
Not at all
2
3
4
5
Perfectly
29
Q

The determination of whether pre-AIA 35 U.S.C. 102 and 103 or AIA 35 U.S.C. 102 and 103 apply is made on a claim-by-claim basis.

T/F

A

F

How well did you know this?
1
Not at all
2
3
4
5
Perfectly
30
Q

Under the AIA there still remains a public use exception to public use under AIA 102(a)

T/F

A

F

How well did you know this?
1
Not at all
2
3
4
5
Perfectly
31
Q

A U.S. application is filed with method claims 1-5 and apparatus claims 6-10. In a first action all the claims are rejected and arguments are presented in a timely response. A Final Rejection is then mailed, and the response filed is refused entry. A continuation is filed before the parent is abandoned and a Final Rejection is then mailed in the continuation. A CIP is filed before its parent is abandoned and includes new disclosure constituting new matter and claims 11-13 directed to that new disclosure. Claims 11-13 :

A) Are entitled to an effective filing date as of the filing date of the CIP because they are not supported by either of the earlier applications.

B) Are entitled to an effective filing date as of the filing date of the parent of the CIP.

C) Are entitled to an effective filing date as of the grandparent of the CIP.

D) Are improper because they are based upon new matter.

E) Must be cancelled.

A

A) Are entitled to an effective filing date as of the filing date of the CIP because they are not supported by either of the earlier applications.

Reasoning: The CIP includes a new disclosure, therefore the claims are directed towards that new disclosure and are not supported by the earlier applications and can’t use them for priority. They can only be used for priority if the disclosures in the applications support the claims.

How well did you know this?
1
Not at all
2
3
4
5
Perfectly
32
Q

A CIP can be filed under 37 CFR 1.53(d) with a preliminary amendment adding the new matter to the original disclosure.

T/F

A

F

How well did you know this?
1
Not at all
2
3
4
5
Perfectly
33
Q

A freshly executed oath must be filed in every continuation within sixty days of filing.

T/F

A

F

How well did you know this?
1
Not at all
2
3
4
5
Perfectly
34
Q

A new assignment is required only if a continuation is filed under Rule 53(b).

T/F

A

F

How well did you know this?
1
Not at all
2
3
4
5
Perfectly
35
Q

The examiner cannot make a first action in a continuation “Final” even if it is the same as the last action in the parent.

T/F

A

F

How well did you know this?
1
Not at all
2
3
4
5
Perfectly
36
Q

The application of question 1 was eventually finally rejected by the examiner. A continuation application was filed and the parent was subsequently abandoned. A proper claim for foreign priority was made in the parent application. Which of the following statements is true?

A) Priority has been lost and cannot be claimed in the continuation.

B) Priority needs to be claimed in the continuation application.

C) A claim for priority in the continuation must be accompanied by a translation of the Thai application.

D) A reissue application should be filed in order to claim the priority in the parent application.

E) None of the above is true.

A

B) Priority needs to be claimed in the continuation application.

Reasoning:
MPEP 201.07 - The inventorship in the continuation application must include at least one inventor named in the prior-filed application, and the continuation application must also claim the benefit of the prior-filed application under 35 U.S.C. 120, 121, 365(c), or 386(c). See 37 CFR 1.78, especially paragraphs (d) and (e), and MPEP § 211 et seq. for additional requirements and more information regarding entitlement to the benefit of the filing date of a prior-filed copending application.

How well did you know this?
1
Not at all
2
3
4
5
Perfectly
37
Q

In a continuation application, the examiner objects to the oath filed in the parent application as stale because it was signed more than three months before the U.S. filing date. The most appropriate response would be:

A) Abandon the continuation application.

B) File a new oath.

C) Point out that a stale oath no longer must be replaced.

D) Ignore the examiner’s objection.

E) File another continuation with a proper oath.

A

C) Point out that a stale oath no longer must be replaced.

Reasoning: MPEP 602.03 - The Office does not check the date of execution of
the oath or declaration and will not require a newly
executed oath or declaration based on an oath or
declaration being stale (i.e., when the date of
execution is more than three months prior to the
filing date of the application) or where the date of
execution has been omitted. However, applicants
are reminded that they have a continuing duty of
disclosure under 37 CFR 1.56.

How well did you know this?
1
Not at all
2
3
4
5
Perfectly
38
Q

A provisional application was filed on June 1, 2012 relating to an improved golf club. The drawings fully illustrate the exterior appearance of the club. The clubs were first sold in the United States on June 23, 2011. Today, August 27, 2012, your client contacts you and tells you she wants to obtain design patent protection on the club. Your advice would be:

A) No protection can be obtained because it is barred by 102(b).

B) A design patent can be filed and claim benefit of the provisional filing date.

C) Designs on golf clubs are not patentable subject matter.

D) File a utility application claiming benefit of the filing date of the provisional and file a design application as a divisional of the Utility application.

E) Do nothing.

A

A) No protection can be obtained because it is barred by 102(b).

Reasoning: Design applications can’t claim priority from provisional applications. I originally choose D because I thought that would be a way to get around it. This is what I thought why:

MPEP 201.06 - A design application may be considered to be a divisional of a utility application (but not of a provisional application), and is entitled to the filing date thereof if the drawings of the earlier filed utility application show the same article as that in the design application sufficiently to comply with 35 U.S.C. 112(a). However, such a divisional design application may only be filed under the procedure set forth in 37 CFR 1.53(b), and not under 37 CFR 1.53(d). See MPEP § 1504.20

However, the design has to be disclosed in the utility application.

How well did you know this?
1
Not at all
2
3
4
5
Perfectly
39
Q

A Final Rejection was mailed on February 21, 2013 setting a three month period for response. Your client asks you to arrange an interview with the examiner during July. Which of the following is true.

A) An interview is not permitted.
B) Your client can attend the interview.
C) No demonstrations can be shown to the examiner during the interview.
D) If agreement is reached during the interview, no written response to the Final Rejection need be filed.
E) An extension of time must be obtained before the interview can be held.

A

B) Your client can attend the interview.

Reasoning: I thought it was E initially. However, in 713.09, it says “Interviews may be held after the expiration of the
shortened statutory period and prior to the maximum
permitted statutory period of 6 months without an
extension of time. See MPEP § 706.07(f).”

How well did you know this?
1
Not at all
2
3
4
5
Perfectly
40
Q

A PCT application was filed on October 23, 1996 in the USPTO by an American citizen living in Paris claiming priority from a French application filed October 24, 1995. The PCT application designated the U.S. On March 22, 1997 a translation of the application is furnished to the USPTO to enter the national stage and the national fee was paid. An Oath was submitted on April 3, 1997. If the U.S. patent issues, assuming no additional patent term is awarded and all maintenance fees are paid, it will expire on ______ and its date as prior art under 102(e) will be__________ :

A) October 24, 2015 and March 22, 1997.
B) October 23, 2016 and March 22, 1997.
C) October 23, 2016 and April 3, 1997.
D) March 22, 2017 and April 3, 1997.
E) April 3, 2017 and April 3, 1997.

A

C) October 23, 2016 and April 3, 1997.

Reasoning: The patent term is 20 years from filing. The filing date of the PCT is the date it enters the national stage.

How well did you know this?
1
Not at all
2
3
4
5
Perfectly
41
Q

A defensive publication is prior art as of its filing date.

T/F

A

F

Reasoning: It is prior art as of its publication date

How well did you know this?
1
Not at all
2
3
4
5
Perfectly
42
Q

The pre-AIA 102(e) date for a U.S. patent issuing from a PCT application entering the National Stage in the U.S. is always the date that the oath or declaration, the fee and the copy of the international application and translation into English are received.

T/F

A

F

Reasoning: Depends if it is filed before or after 11-29-00. If after, then the international application just has to indicate the US, then it has a pre-AIA 102(e) date as of it’s filing date.

How well did you know this?
1
Not at all
2
3
4
5
Perfectly
43
Q

An abandoned application establishes that the invention disclosed in that application has been abandoned and is not prior art under pre-AIA 102(g).

T/F

A

F

Reasoning: Can still be prior art

How well did you know this?
1
Not at all
2
3
4
5
Perfectly
44
Q

The critical date for a foreign patent under pre-AIA 102(d) is the date it becomes enforceable.

T/F

A

T

Reasoning: 2135.01

How well did you know this?
1
Not at all
2
3
4
5
Perfectly
45
Q

If performance of the claimed method involves an application of a law of nature, which of the following are not among the factors for determining whether the method claim embodies merely an unpatentable abstract idea?

A) Whether the claim generically recites an effect of the law of nature or claims every mode of accomplishing that effect.

B) Whether the claimed method recites an application of a law of nature solely involving subjective determinations.

C) The extent to which the application imposes meaningful limits on the execution of the claimed method steps.

D) Whether one of ordinary skill in the art would understand the claimed method incorporating the law of nature to be obvious.

E) All of the above.

A

D) Whether one of ordinary skill in the art would understand the claimed method incorporating the law of nature to be obvious.

Reasoning: Don’t overthink it and look for particularities. A-C deal with 101 considerations while D deals with obviousness, which is a 103 consideration.

How well did you know this?
1
Not at all
2
3
4
5
Perfectly
46
Q

If a general concept is involved in executing the steps of the method, this can be a clue that the claim is drawn to an abstract idea. Where a general concept is present, which of the following factors are relevant to determine whether the claim embodies merely an unpatentable abstract idea?’1.The extent to which use of the concept, as expressed in the method, would preempt its use in other fields.’2.The extent to which the claim is so sweeping as to cover both known and unknown uses of the concept.’3.The extent to which the claim would effectively cover all possible solutions to a particular problem.

A) Statement 1
B) Statement 2
C) Statements 1 and 2
D) Statements 2 and 3
E) Statements 1, 2 and 3

A

E) Statements 1, 2 and 3

Reasoning: The use of the concept can’t preempt its use in other fields. If the claim covers all known and unknown uses of the concept, then it is abstract. Lastly, if the claim covers all possible solutions, then it is just a statement of the problem.

How well did you know this?
1
Not at all
2
3
4
5
Perfectly
47
Q

When considering whether a method claim embodies an unpatentable abstract idea, which of the following would tend to weigh in favor of patent elibigility?”1.A particular machine or apparatus is incorporated into the claimed method.”2.A machine or apparatus is an object on which the method operates.”3.The recited machine or apparatus contributes nominally to the execution of the claimed method.

A) Statement 1
B) Statement 2
C) Statement 3
D) Statement 1 and 2
E) Statement 1 and 3

A

A) Statement 1

Reasoning: If a particular machine or apparatus is incorporated into the claimed method, this weighs in favor of patent eligibility in that there is “significantly more” than the method alone. This is particularly true when the method steps are specifically tied to the machine or apparatus. Factors weighing against eligibility are: (1) No recitation of a machine or transformation (either express or inherent). (2) Insufficient recitation of a machine or transformation. (3) Involvement of machine or transformation with the steps is merely nominally, insignificantly, or tangentially related to the performance of the steps. (4) Machine is generically recited such that it covers any machine capable of performing the claimed step(s). (5) Machine is merely an object on which the method operates. (6) Transformation involves only a change in position or location of an article. (7) The claim would monopolize a natural force or patent a scientific fact; e.g., by claiming every mode of producing an effect of that law of nature. (8) A law of nature is applied in a merely subjective determination. (9) A law of nature is merely nominally, insignificantly, or tangentially related to the performance of the steps. (10) The claim is a mere statement of a general concept. (11) Use of the concept, as expressed in the method, would effectively grant a monopoly over the concept. (12) Both known and unknown uses of the concept are covered, and can be performed through any existing or future-devised machinery, or even without any apparatus. (13) The claim only states a problem to be solved. (14) The general concept is disembodied. (15) The mechanism(s) by which the steps are implemented is subjective or imperceptible.

Found in 2106.05(b)

How well did you know this?
1
Not at all
2
3
4
5
Perfectly
48
Q

The time period to reply to a notice that a response is bona fide but incomplete cannot be extended.

T/F

A

F

Reasoning: 37 CFR 1.135(c), 1.136

(c) When reply by the applicant is a bona fide
attempt to advance the application to final action, and is substantially a complete reply to the non-final Office action, but consideration of some matter or compliance with some requirement has been inadvertently omitted, applicant may be given a new time period for reply under § 1.134 to supply the
omission.

How well did you know this?
1
Not at all
2
3
4
5
Perfectly
49
Q

A Petition for Access is no longer required to obtain access to the file of a pending application, which has been incorporated by reference into an U.S. patent.

T/F

A

F

Reasoning: A copy of the application doesn’t require a petition, but access does. 37 CFR 1.14(c)(1)(i)

How well did you know this?
1
Not at all
2
3
4
5
Perfectly
50
Q

A patent issuing from a CPA will only indicate the application number and filing date of the prior parent application.

T/F

A

T

Reasoning: 37 CFR 1.53(d)

37 CFR 1.53(d) pertains to the filing requirements for a Continued Prosecution Application (CPA). According to this regulation, a CPA is considered a continuation of the prior application. Therefore, a patent issuing from a CPA would carry the same application number and filing date as the prior parent application.

How well did you know this?
1
Not at all
2
3
4
5
Perfectly
51
Q

In accordance with patent laws, rules and procedures, an invention is most likely non-obvious in which of the following circumstances?

A) The prior art contains no teaching, suggestion or motivation to combine the references cited by the patent examiner.
B) The prior art teaches away from the claimed combination and the combination yields more than predictable results.
C) One of ordinary skill in the art would have been motivated to combine known elements in a novel arrangement.
D) The prior art combined by the patent examiner is tangentially related to the problem that the inventor was trying to solve.
E) It was obvious to try a finite set of combinations, but at the outset it was not apparent which combination would be useful.

A

B) The prior art teaches away from the claimed combination and the combination yields more than predictable results.

Reasoning: I chose A originally, but A is just a lack of teaching, while B is a teaching away from the combination, which is a stronger argument.

How well did you know this?
1
Not at all
2
3
4
5
Perfectly
52
Q

Examiners will apply §112(f) to a claim limitation that meets which of the following conditions:

1.The claim limitation uses the phrase “means for” or “step for” or a non-structural term that further includes a structural modifier.

2.The phrase “means for” or “step for” or the non-structural term recited in the claim is modified by functional language.

  1. The phrase “means for” or “step for” or the non-structural term recited in the claim is not modified by sufficient structure, material, or acts for achieving the specified function.

A) Statement 1
B) Statement 2
C) Statement 3
D) Statements 2 and 3
E) Statement 1, 2 and 3

A

D) Statements 2 and 3

Reasoning: Statement 1 is not correct. When “means for” or “step for” is used in conjunction with further discussion of structure, §112(f) is not invoked. For example, although a non-structural term like ˜mechanism’’ standing alone may invoke §112(f) when coupled with a function, it will not invoke §112(f) when it is preceded by a structural modifier (e.g., ˜detent mechanism’’).

Look in MPEP 2181 page 496.

How well did you know this?
1
Not at all
2
3
4
5
Perfectly
53
Q

A process claim containing the term “computer” should:

A) Not be construed as limited to a computer having a specific set of characteristics and capabilities, unless the term is modified by other claim terms or clearly defined in the specification to be different from its common meaning.
B) Be given the broadest reasonable interpretation possible and should include devices such as a calculator.
C) Ordinarily not be considered sufficient to satisfy the machine-or-transformation test.
D) Not be rejected to under §112(a) for lack of written description because the term has a well established meaning.
E) None of the above.

A

A) Not be construed as limited to a computer having a specific set of characteristics and capabilities, unless the term is modified by other claim terms or clearly defined in the specification to be different from its common meaning.

Reasoning: In In re Paulsen, the claims, directed to a portable computer, were rejected as anticipated under §102 by a reference that disclosed a calculator, because the term ˜computer’’ was given the broadest reasonable interpretation consistent with the specification to include a calculator, and a calculator was considered to be a particular type of computer by those of ordinary skill in the art. Answer B is incorrect because you simply cannot give the broadest reasonable interpretation without reference to what the specification includes; otherwise you would be giving more claim scope than the written description supports. Answer C is incorrect because the inclusion of a “computer” does ordinarily satisfy the machine-or-transformation test, particularly when the process steps are indicated to be performed on a computer. Answer D is incorrect; the term “computer” does not have a well-defined meaning, and does frequently include far more than what the generally understood term encompasses.

Computer is a wide term.

How well did you know this?
1
Not at all
2
3
4
5
Perfectly
54
Q

You filed an international application under the Patent Cooperation Treaty (PCT) for a client on July 5, 2016. The application claimed priority from a prior Brazilian application filed August 30, 2015. The international application designated the European Patent Office, South Africa and the United States. A demand for international preliminary examination, electing all eligible states, was filed on February 1, 2017. Which of the following would be the last day on which the basic national fee to entry into the United States can be paid?

A) January 5, 2019.
B) March 5, 2018.
C) March 5, 2017.
D) February 28, 2018.
E) None of the above.

A

D) February 28, 2018.

Reasoning: An applicant seeking entry into the US National phase via PCT has to pay the national fee no later than 30 months from the earliest priority date being asserted. Here, the priority being asserted is August 30, 2015. When 30 months are added to that date, you obtain the last day of February, 2018. Don’t even consider that the year in question might have been a leap year (and so the perfect answer would be February 29, 2019). Unless specifically told otherwise on the Exam, the year in question is NOT a leap year, and February has 28 days.

Moreover, this is an example that you may not get the “right” answer among the choices; you’re looking for the “best” answer. February 28 is clearly the best answer here, even if it’s not “right.”

How well did you know this?
1
Not at all
2
3
4
5
Perfectly
55
Q

Which of the following is not required to obtain an international filing date for an application filed under the Patent Cooperation Treaty (PCT);

A) A claim
B) A specification.
C) The designation of at least one PCT contracting state.
D) The name of the inventor.
E) An indication that the application is intended to be a PCT application.

A

D) The name of the inventor.

Reasoning: MPEP 1810

How well did you know this?
1
Not at all
2
3
4
5
Perfectly
56
Q

An international application designating the United States and other countries was filed in London at the U.K. Receiving Office on September 3, 1994, claiming priority from a French application filed June 23, 1994. A demand for international preliminary examination was properly made on October 14, 1995. The basic national fee to enter the U.S. was submitted to the USPTO on December 12, 1996, one year after the International Application was published in English. On March 12, 1997, in response to a Notice of Missing Parts a translation of the international application and a declaration of the inventors were submitted. The declaration was objected to by the Examiner because it did not indicate the residence address of one of the three inventors in an Ex parte Quayle action dated April 14, 1997, setting a two month period for response. A new declaration was submitted together with a request for reconsideration on May 5, 1997. The application issued as a U.S. patent on July 23, 1998. What is the effective date of the patent under 35 USC 102(e):

A) July 23, 1998.
B) December 12, 1995.
C) March 12, 1997.
D) September 3, 1994.
E) May 5, 1997.

A

C) March 12, 1997.

Reasoning: Since this PCT was filed before 11/29/00, the 102(e) date for a later-issuing US patent is the day the filing requirements (under 371, a.k.a. the FOoT rule) were complete. Here, the day the last of these (fee, oath, translation) arrived was March 12, 1997. The later objection to the declaration does not move that date.

How well did you know this?
1
Not at all
2
3
4
5
Perfectly
57
Q

Which of the following is not a requirement for filing an international application in the U.S. Receiving Office:

A) At least one applicant must be a U.S. national or legal resident.
B) The application must be in English.
C) The application must designate at least one PCT contracting state.
D) The filing fee must be submitted.
E) A request for treatment as an international application must be included.

A

D) The filing fee must be submitted.

Reasoning:

How well did you know this?
1
Not at all
2
3
4
5
Perfectly
58
Q

On April 3, 2014 you filed an international (PCT) application in the U.S. receiving Office for one of your clients. At the time your client was short of funds and decided to designate only the “Asian Tiger” countries of Malaysia, Indonesia, China and Korea. However, last week a company located in New Zealand approached your client indicating they would license any patent rights, which existed in New Zealand. On April 1, 2015, your client calls, seeking protection in New Zealand. What would be your advice?

A) Nothing. No additional countries need to be designated.
B) A Request for Additional Designations can now be filed together with the surcharge for late designation.
C) File another PCT application this time designating New Zealand.
D) File a continuation of the PCT application together with a new designation including New Zealand.
E) Abandon the PCT application and file a replacement with the expanded designation.

A

A) Nothing. No additional countries need to be designated.

Reasoning: All member countries of WIPO are automatically designated in any PCT applications filed after 1/1/04. Hence, they are all already designated.

How well did you know this?
1
Not at all
2
3
4
5
Perfectly
59
Q

Today, August 23, 2018 Jane Roe asks you to advise her regarding her patent, which issued on March 13, 2017. The patent is directed to a method of playing chess that she conceived and reduced to practice in December of 2013 while vacationing in Quebec, Canada. The application was filed in the United States on May 2, 2015. Since January of this year, Jane has been in negotiation with Chess Is Us (CIS), the leading manufacture of chess sets to license her invention exclusively. CIS told her today that their patent attorney has advised them that all the claims of her patent contain unnecessary limitations, and that accordingly the patent has very little value to them. Which of the following is true:

A) Either Jane or CIS may file a Request for a Reexamination with new claims deleting the unnecessary limitations.
B) Only Jane may file a Request for a Reexamination with broadened claims.
C) Any Request for Reexamination submitting new claims must be filed before March 13, 2019.
D) Claiming less than Jane had a right to claim in her patent may be the basis of a reissue.
E) A reexamination may only be filed under 37 CFR 1.53(b).

A

D) Claiming less than Jane had a right to claim in her patent may be the basis of a reissue.

Reasoning: Broadening claims can only be filed in a reissue, not a reexamination. That removes A and B as answers. There is no deadline for a reexamination, only a reissue, so C is out. Reexaminations are filed under 37 CFR 1.510. That removes E as an answer.

How well did you know this?
1
Not at all
2
3
4
5
Perfectly
60
Q

A claim in your client’s patent application has been rejected on obviousness grounds. To overcome this rejection, your client may do any of the following except:

A) Amend the claim to include limitations not taught in the prior art.
B) Argue that the combination of references would not create a reasonable expectation of success.
C) Argue that obvious to try is an inappropriate standard to apply for a rejection under 35 U.S.C. 103.
D) Argue that the claimed invention shows unexpectedly strong advantages over the prior art.
E) Argue that evidence of commercial success is pertinent where a nexus between the success of the product and the claimed invention has been demonstrated.

A

C) Argue that obvious to try is an inappropriate standard to apply for a rejection under 35 U.S.C. 103.

Reasoning: “Obvious to try” is a valid standard to apply. MPEP 2141 page 263

How well did you know this?
1
Not at all
2
3
4
5
Perfectly
61
Q

Which of the following statements is most in accordance with patent laws, practice and procedures relative to analogous art?

1.When determining whether a reference in a different field of endeavor may be used to support a case of obviousness, it is necessary to consider the problem to be solved.

2.Analogous art is not limited to references in the field of endeavor of the invention, but also includes references that would have been recognized by those of ordinary skill in the art as useful for applicant’s purpose.

3.Prior art is not considered analogous merely because it would have been recognized by one of ordinary skill in the art as useful for the applicant’s purpose, but instead must have some “critical nexus” with the claimed invention.

A) Statement 1
B) Statement 2
C) Statement 3
D) Statement 1 and 2
E) Statement 1 and 3

A

D) Statement 1 and 2

Reasoning: See teaching points for In re ICON Health & Fitness and Agrizap, Inc. v. Woodstream Corp. Statement 3 contradicts the teaching point from Agrizap.

How well did you know this?
1
Not at all
2
3
4
5
Perfectly
62
Q

Which of the following statements accurately defines when “obvious to try” is erroneously equated with obviousness under 35 U.S.C. 103?

1.When what would have been “obvious to try” would have been to vary all parameters or try each of numerous possible choices until one possibly arrived at a successful result, where the prior art gave either no indication of which parameters were critical or no direction as to which of many possible choices is likely to be successful.

2.When what was “obvious to try” was to explore a new technology or general approach that seemed to be a promising field of experimentation, where the prior art gave only general guidance as to the particular form of the claimed invention or how to achieve it.

3.Where what is “obvious to try” is determined by merely throwing metaphorical darts at a board filled with combinations of prior art possibilities to find the requisite elements and limitations that together combine to yield the claimed invention.

A) Statement 1
B) Statement 2
C) Statement 3
D) Statement 1 and 3
E) Statements 1, 2 and 3.

A

E) Statements 1, 2 and 3.

Reasoning: In re Kubin specifically cites In re O’Farrell for statements 1 and 2. The Federal Circuit in summarizing statement 1 explains that “In such circumstances, where a defendant merely throws metaphorical darts at a board filled with combinatorial prior art possibilities, courts should not succumb to hindsight claims of obviousness.” Thus, statement 3 is also a correct articulation as well. Presumably, the throwing of darts at the board equates with not having any particular direction within the prior art, requiring the examiner to merely pick and choose based on the absence of reason or logic.

How well did you know this?
1
Not at all
2
3
4
5
Perfectly
63
Q

You file a patent application on behalf of your new client claiming a new and improved compound to treat memory loss. The patent examiner rejects the claims submitted to the compound on obviousness grounds, citing the Greg reference which discloses over 1 million compounds potentially useful for treating memory loss and additionally citing the Marsha reference which identified Compound Y as being among 20 compounds that seem to have the most positive therapeutic value for treatment of memory loss. The examiner explains in the rejection that Compound Y is the closest known compound to the claimed compound and that the claimed compound would have been obvious to one of skill in the art in view of Compound Y. Your client explains to you that Compound Y is indeed known to significantly enhance memory, but also is known to cause loss of bowel and bladder control in upwards of 40% of those individuals who were administered the compound. Which of the following arguments would not help you to convince the examiner that the obviousness rejection should be withdrawn?

A) Compound Y would not be selected by one of ordinary skill in the art for experimentation due to the undesirable negative side effects.

B) Given the negative side effects of Compound Y, it would have been logical to assume that those compounds structurally similar to Compound Y would not prove suitable.

C) With over 1 million compounds known to be useful for treating memory loss, one of skill in the art would not attempt to select Compound Y for further research.

D) Compound Y was one of only 20 compounds identified in the prior art to have the most positive therapeutic value for treating memory loss and, therefore, the prior art as a whole would not suggest to one of skill in the art that it would be obvious to try Compound Y over the other 19 compounds.

E) All of the above would be helpful arguments to make.

A

D) Compound Y was one of only 20 compounds identified in the prior art to have the most positive therapeutic value for treating memory loss and, therefore, the prior art as a whole would not suggest to one of skill in the art that it would be obvious to try Compound Y over the other 19 compounds.

Reasoning: The “obvious to try” reasoning for a obviousness rejection is a valid argument by the examiner.

How well did you know this?
1
Not at all
2
3
4
5
Perfectly
64
Q

With respect to obviousness, which of the following are true?

1.In cases involving new chemical compounds, it is necessary to identify some reason that would have led a chemist to modify a known compound in a particular manner to establish prima facie obviousness of a new claimed compound.

2.If a patent examiner makes a prima facie showing of obviousness, the applicant may rebut based on unexpected results by demonstrating that the claimed invention exhibits some superior property or advantage that a person of ordinary skill in the relevant art would have found surprising or unexpected.

3.In order to demonstrate that a claimed invention is obvious, a patent examiner can demonstrate how one of ordinary skill in the art would be able to retrace the steps of the applicant in order to achieve the claimed invention.

A) Statement 1
B) Statement 2
C) Statement 3
D) Statements 1 and 2
E) Statements 1, 2 and 3.

A
How well did you know this?
1
Not at all
2
3
4
5
Perfectly
65
Q

On September 23, 2016 a U.S. patent application was filed directed to a screwdriver. On January 8, 2017 a claim for priority was made and a certified copy filed of a Canadian application filed October 21, 2015. A continuation was filed in June of 2017 under 37 CFR 1.53(b) and the parent abandoned shortly thereafter. No claim for priority was made in the continuation, which issued as a U.S. patent on January 13, 2018. Which of the following statements is true:

A) The failure to claim priority in the continuation can only be corrected by filing a reissue application.

B) The failure to claim priority in the continuation application can be corrected by obtaining a Certificate of Correction and also filing a Grantable Petition and fee for Late Assertion of Priority.

C) The failure to claim priority cannot be corrected and the priority is lost.

D) There is no need to claim priority in the Continuation since priority was claimed in the parent.

E) None of the above is true.

A

B) The failure to claim priority in the continuation application can be corrected by obtaining a Certificate of Correction and also filing a Grantable Petition and fee for Late Assertion of Priority.

Reasoning: 216.01

How well did you know this?
1
Not at all
2
3
4
5
Perfectly
66
Q

Your client MegaGiant has discovered a patent, owned by its competitor GigaCorp, that MegaGiant feels may be infringed by MegaGiant’s new lawn mower. MegaGiant has learned as well that the lawn mower described in the patent was on sale in the United States by GigaCorp more than one year before the earliest effective filing date of the patent. MegaGiant is seeking your advice. Your advice is:

A) A Request for Reexamination can be filed based upon the prior sale.
B) All the claims of the patent are invalid under 35 USC 102(a)(2).
C) All the claims of the patent are invalid under 35 USC 102(a)(1).
D) If a Request for Reexamination is brought based upon a prior patent or publication, the PTO may also consider the prior sale.
E) MegaGiant may petition the PTO to reissue the patent.

A

C) All the claims of the patent are invalid under 35 USC 102(a)(1).

Reasoning: Don’t overthink it.

How well did you know this?
1
Not at all
2
3
4
5
Perfectly
67
Q

You have filed a request for reissue of a patent owned by your client Megabucks. The only reason for the reissue is to correct an error in the example where a weight was described in tons rather than ounces. The examiner has issued an action rejecting all of the claims based upon a U.S. patent which was filed more than one year before the date of filing of your patent, but which issued after your patent. The reference discloses, but does not claim every detail set forth in your claims. The most appropriate response would:

A) Request the examiner to withdraw the rejection because its scope exceeds the scope of the reissue issues.
B) Abandon the reissue application.
C) Petition the Commissioner to require the Examiner to withdraw the rejection as improper.
D) File a response arguing that the rejection cannot be made in the reissue since it could not have been made in the original patent.
E) Appeal

A

B) Abandon the reissue application.

Reasoning: The objection cannot be overcome.

68
Q

Alfred, the inventor of an emergency escape device, confidentially discloses his invention to Professor William Omaha McElroy on June 1, 2012, in hopes of obtaining independent verification of his test results. McElroy is intrigued by the invention and, without Alfred’s knowledge, decides to write about Alfred’s invention for publication in Science Daily. The article describing Alfred’s invention is published March 15, 2013. Harvey Dent reads Professor McElroy’s publication and files a patent application on April 30, 2013, claiming Alfred’s invention. Alfred finally files a patent application on May 14, 2013, which discloses the same invention as does the Dent application, but which claims different subject matter.

With respect to Dent’s patent application, which of the following are true?

A) The disclosure to McElroy will create an insurmountable 102(a) rejection.
B) The McElroy publication will create an insurmountable 102(a) rejection.
C) Dent will be able to overcome a rejection based on the McElroy publication by filing a 130 affidavit.
D) Dent will be able to overcome a rejection based on the McElroy publication because he was first to file.
E) None of the above.

A

B) The McElroy publication will create an insurmountable 102(a) rejection.

Reasoning: Read the question closely! It’s asking about the Dent patent.

Dent will not be able to obtain a patent on this invention because the McElroy publication occurred before the effective filing date of his application. No exception under AIA 102(b) can save Dent because he is not an inventor. This was formerly a problem under pre-AIA 102(f) and still remains a problem under AIA 102. That is why the new law is characterized as a “first inventor to file” rule.

(A) is incorrect because the disclosure to McElroy was done in confidence, which means that it would not be prior art.

(C) and (D) are incorrect because Dent will not be able to overcome the rejections because he is not an inventor, even with a 130 Affidavit. He merely filed an application on an invention where he did not provide any conception whatsoever.

(E) is incorrect because (B) is correct.

69
Q

Phillip Foreman is the inventor of a magnetic retrieving tool. The magnet is mounted on a carrier that can be extended from and retracted into a magnetic shield that is affixed to the distal end of an extended hollow flexible tube. A provisional patent application was filed on behalf of Foreman on April 2, 2012. A subsequent nonprovisional patent application was filed on April 1, 2013. The nonprovisional patent application was identical to the provisional filing with the exception of the additional inclusion of a latching device at the proximal end of the tool which was not disclosed in the provisional patent application. The latching device is an important improvement over the earlier version of the invention because the magnet, after having been extended to an exposed position, can be latched in the exposed position for the remainder of the retrieving operation. The nonprovisional patent application discloses but does not claim the latching device confining the claims to the invention as disclosed in the provisional patent application. Foreman’s attorney calls the patent examiner prior to her work on the First Office Action and conducts an informational interview. The examiner gives certain insights into prior art that will likely be relevant. Foreman’s attorney asks the Examiner to hold off working on the First Office Action so that new claims can be submitted. Subsequently a Preliminary Amendment is filed which adds claims to the previously unclaimed latching device. In the First Office Action, the patent examiner treats the application under AIA and rejects the application based on the foreign filing date a US patent to Jones.

What is the most appropriate response for Foreman to take in accordance with patent laws and USPTO rules of procedure?

A)
Argue that the nonprovisional patent application filed by Foreman is not considered a transition application because, as of the filing date, the claims were identical in scope to the invention disclosed in the provisional patent application.

B)
Argue that the nonprovisional patent application is entitled to the priority of the provisional filing date, which entitles the applicant to consideration using pre-AIA law.

C)
Argue that the nonprovisional patent application is entitled to the priority of the provisional filing date. That entitles the application to consideration using pre-AIA law, which further means that the filing date of the Jones reference is unavailable, since foreign filings are not prior art as of the date they are filed.

D)
Cancel all claims that in any way refer to the latching device and argue that the claimed invention now only relates to the invention disclosed in the provisional filing.

E)
None of the above.

A

E) None of the above.

Reasoning: This may seem unfair, but the USPTO has taken the firm position that if, at any time, an application includes claims that gain priority support only in a filing that occurred on or after March 16,2013, the entire application will be considered under the AIA. Further, the USPTO has also said that, once such claims have been presented in an application, there is no cure; the canceling of the AIA claims will not work to return the application to one deserving of pre-AIA treatment. Thus, (D) is incorrect.

(A) is incorrect. As initially filed, the Foreman application was not truly a transition application, because it was confined to claiming only what was disclosed in the provisional patent application. The addition of claims to the latching device, however, means that the nonprovisional will be considered under the AIA.

(B) and (C) are incorrect. It is true that the nonprovisional is entitled to claim the benefit of the provisional filing date, but only with respect to what was present at the time of filing. The latching device was not present at the time of the provisional filing. It is the claiming of the latching device that is problematic.

70
Q

Hannah Douglas, an independent inventor who resides in Lexington, Kentucky, is the inventor of an internal combustion engine and a method of operating that incorporates the direct injection of gaseous fuel into a combustion chamber and an ignition-assist apparatus disposed within the combustion chamber. The ignition-assist apparatus employs a catalytic coating, which provides advantages that become more evident when used with fuels that are more difficult to ignite by reducing the requisite temperature for assisting ignition and extending the service life of an ignition-assist apparatus. Without the assistance of an attorney or agent, Douglas files a provisional patent application on the device and method at the United States Patent and Trademark Office on December 25, 2012, using the Electronic Filing System. Once again without the assistance of an attorney or agent, a subsequent nonprovisional patent application was filed on Thursday, December 27, 2013. This non provisional filing claimed priority to the earlier filed provisional patent application and was identical to the provisional filing.

Which of the following most accurately reflects the way the December 27, 2013, nonprovisional patent application will be treated by the USPTO?

A)
The nonprovisional patent application will be examined under the provisions of the AIA.

B)
The nonprovisional patent application will be examined under the pre-AIA provisions.

C)
Since Douglas represented herself pro se, the failure to properly preserve priority can be restored because the delay in filing the application was unavoidable. Thus, the application will be afforded pre-AIA treatment.

D)
The failure to preserve priority can be restored via a PLTIA filing because the delay in filing the nonprovisional patent application was unintentional; thus, the application will be afforded pre-AIA treatment.

E)
The nonprovisional patent application will be considered a substitute application.

A

D)
The failure to preserve priority can be restored via a PLTIA filing because the delay in filing the nonprovisional patent application was unintentional; thus, the application will be afforded pre-AIA treatment.

Reasoning: Thanks to the Patent Law Treaties Implementation Act (“PLTIA”), applicants who unintentionally fail to file an application within the 12-month priority period can have that period extended up to an additional 2 months. Thus, (C) is incorrect. Further, this would not be a scenario for unavoidability. Douglas incorrectly calculated the filing date of the provisional patent application. While it is true that the USPTO was closed on December 25, 2013, if a filing were made on a Saturday, Sunday or Federal Holiday, the USPTO will still award the date of receipt via EFS as the filing date. Based on the story contained within the question, it is obvious that Douglas did not intend to lose the ability to claim priority to the provisional filing; thus, the mistake was unintentional. As a result, her ability to claim priority to the provisional filing may be restored. Once restored, since the provisional application was filed prior to March 16, 2013, and further because the provisional and nonprovisional applications are identical, the application will be examined under the provisions of pre-AIA, not AIA. Thus, (A) is incorrect.

(B) is close, but not completely correct. Douglas will still need to seek restoration of the ability to claim priority. The PLTIA does not automatically make it possible to claim priority within 14 months.

(E) is incorrect because the nonprovisional application is not a substitute application. Upon seeking restoration and paying the appropriate fee, Douglas will be allowed to claim priority from the December 25, 2013, provisional patent application.

71
Q

Gabriel Kaplan, a citizen of Canada and resident of Toronto, is the inventor of a new blackboard with a mirror system that incorporates an integrated laser stunning device. The device can be used by teachers to keep students awake, or otherwise maintain a disciplined learning environment. When facing the board, the teacher may glance at one or more of the mirrors and notice a student dosing, or perhaps engaging in mischief. With a triple-tap to the board with a single finger, a laser will fire, stunning the student. Kaplan first files a Canadian patent application on January 2, 2013. Subsequently, Kaplan files a US patent application on January 1, 2014, claiming priority to his Canadian filing date. At the time he files the US patent application, Kaplan also files papers to opt out of publication in the United States. The Canadian application publishes on June 2, 2014, and the Canadian patent issues on September 22, 2014. On March 14, 2015, the US patent examiner rejects all claims in the Kaplan application. Having determined the US is not a market worth exploiting, Kaplan does not file a response and the US application goes abandoned on September 15, 2015.

Which of the following would be the earliest date the Kaplan disclosure could be used as prior art against others?

A) January 2, 2013.
B) January 1, 2014.
C) June 2, 2014.
D) September 22, 2014.
E) September 15, 2015.

A

A) January 2, 2013.

Reasoning: I originally chose C because foreign patents are prior art as of their publication date. An applicant may opt out of publication, but then they waive their right to file a foreign patent. Since he already filed, he can’t opt out of publication. Thus, the US patent will publish and the prior art date will be the date of the foreign filing.

72
Q

Monte Lee is the inventor of a pulmonary mattress and associated method of use to treat severe upper respiratory infections. The invention is used as a patient support device and includes a cover defining an interior region, a layer of three-dimensional material, and an air circulation device disposed adjacent the layer of three-dimensional material. An International Patent Application is filed in the United States Patent Office on March 15, 2013. Believing that methods of use are not patentable outside the US, Lee does not disclose the method of use. A nonprovisional patent application is filed March 3, 2014, which claims the benefit of the earlier-filed International Application. The nonprovisional patent application discloses the same invention as included in the International Application, but also includes two new embodiments. The first further comprises a vibration device located within the inner region. The second incorporates a coin-activated mechanism for the vibration device. The nonprovisional patent application contains 25 claims. Only claims 1 – 14 relate to the pulmonary mattress, Claims 15 – 18 incorporate the vibration device, Claims 19 – 22 incorporate the coin-activation mechanism to the vibration device and Claims 23 – 25 claim the method. Due to lack of funds, Lee does not pursue the International Application. In the First Office Action in the nonprovisional application, the patent examiner rejects claims 1 – 14 under 35 U.S.C. 103 based on a US patent to Taft issued in 1925, in view of a Belgian patent application filed by Claire on April 1, 2013, which published on October 1, 2014, and subsequently was used to support priority in a published PCT application that designated the US. Claims 15 – 18 are rejected under 35 U.S.C. 103 based on Taft, in view of Claire and further in view of McHenry. McHenry is an Irish patent issued on June 15, 2014, which was filed in Ireland on February 15, 2013, and which was subsequently used to support a claim of priority for an issued US patent. Claims 19 – 22 are rejected under 35 U.S.C. 103 based on Taft, in view of Claire and McHenry, and further in view of Cosmappo, which is an Italian patent issued on November 22, 2015, on an application filed in Italy on March 2, 2014, and which was subsequently used to support a claim of priority for an issued US patent. Claims 23 – 25 are rejected under 35 U.S.C. 103 based on Taft, in view of Hampton. Hampton is a UK published patent application, which published on June 15, 2014, which was filed on January 14, 2013, and which was subsequently used to support a claim of priority for an issued US patent.

Which arguments can Lee successfully make to overcome these obviousness rejections?

A)
Lee will not be able to remove any of these references. Therefore, he will need to either amend the claims or argue the obviousness rejections on their merits by pointing out that there is no teaching, suggestion or motivation to combine the references, and the claims define an invention that provides unexpected results.

B)
Lee will be able to overcome the rejection of claims 1 – 22 by pointing out that the Claire reference is not prior art.

C)
Lee will be able to overcome the rejection of claims 15 – 18 by pointing out that the McHenry reference is not prior art.

D)
Lee will be able to overcome the rejection of claims 19 – 22 by pointing out that the Cosmappo reference is not prior art.

E)
Lee will be able to overcome the rejection of claims 23 – 25 by pointing out that the Hampton reference is not prior art.

A

B)
Lee will be able to overcome the rejection of claims 1 – 22 by pointing out that the Claire reference is not prior art.

Reasoning: First notice that the Lee nonprovisional patent application will be treated under AIA. Although the filing date of the International Application is prior to March 16, 2013, the nonprovisional patent application contains claims not entitled to the priority date of the International Application. Thus, the entirety of the nonprovisional patent application will be examined under the AIA. Most importantly, that means that US patent applications and US patents will be prior art as of their foreign filing dates, provided that the foreign filing dates are used to support priority in those published US application, granted US patents or published PCT applications that designate the US. Therefore, the Claire reference will be prior art as of its filing date on April 1, 2013, which means that the International Application filing date of March 15, 2013, will remove the Claire reference from the obviousness rejections. We know this to be true because the Claire reference in combination with Taft (from 1925) was used to reject the pulmonary mattress claims that are supported by the March 15, 2013, International Application filing. Thus, the claims that include the pulmonary mattress plus vibration and pulmonary mattress plus coin-operated vibration, will likewise be patentable once Claire is removed; even those extended features do not obtain priority until the filing of the nonprovisional patent application on March 3, 2014.

(A) is incorrect because Claire will be able to be removed as a reference.

(C), (D) and (E) are all incorrect because the McHenry, Cosmappo and Hampton references each have a filing date prior to the nonprovisional filing date of March 3, 2014.

73
Q

Elias James is the inventor of a system that compares, ranks, sorts and grades animals according to previously determined predicted maximum values. In its most advantageous use, the system uses magnetic resonance imaging (MRI) to evaluate the animal and determine a number of days the animal must be fed to reach a maximum value. A provisional patent application was filed on behalf of James on April 15, 2012. A subsequent nonprovisional patent application was filed on April 14, 2013. The nonprovisional patent application was identical in terms of disclosure when compared to the provisional filing with the exception of a scale connected to the MRI computer system for measuring a weight of the animal, which was not disclosed in the provisional patent application. The nonprovisional patent application does not mention the scale in the specification, but does include a single dependent claim containing reference to the scale embodiment.

What is the most appropriate action for James to take in accordance with patent laws and USPTO rules of procedure?

A)
Argue that the nonprovisional patent application filed by James is not considered a transition application because, as of the filing date, the specification was identical in scope to the invention disclosed in the provisional patent application.

B)
Argue that the nonprovisional patent application is entitled to the priority of the provisional filing date, which entitles the applicant to consideration using pre-AIA law.

C)
Argue that the nonprovisional patent application is entitled to the priority of the provisional filing date, which entitles the application to consideration using pre-AIA law.

D)
Provide a statement to the Patent Office no later than August 15, 2013, which identifies the nonprovisional patent application as a transition application.

E)
Cancel all claims that in any way refer to the scale and argue that the claimed invention now only relates to the invention disclosed in the provisional filing.

A

D)
Provide a statement to the Patent Office no later than August 15, 2013, which identifies the nonprovisional patent application as a transition application.

Reasoning: Remember, once the application is deemed AIA, then it can’t be reversed! Thus, the other answers are incorrect.

74
Q

On November 1, 2012, Christine conceived of a new and improved triple-action widget. Christine filed a patent application in the USPTO on December 31, 2012. On June 4, 2012, inventor Richard, independently and without knowledge of Christine’s work, conceived of the same invention. Richard immediately began working on this invention in a diligent manner four days a week through the second week in August. As per his usual family tradition, Richard spent the final weeks of August 2012 vacationing with his family in Southern California. Upon his return from vacation, Richard was busy catching up at work, concentrating on his fantasy football team and helping his kids get back into the swing of the new school year. He once again returned to his invention on October 15, 2012, at which time he again began working on the invention religiously four days a week. Richard finally achieved a working prototype just before Thanksgiving in 2012, on November 21, 2012, specifically. After the first of the year, on January 3, 2013, Richard discussed his invention with his patent attorney and requested the attorney prepare a patent application. Because Richard was on an extended business trip and unavailable to give final authorization to file the application, Richard’s patent application was not filed at the USPTO until March 18, 2013. The claims filed in Richard’s patent application are identical in scope to the claims filed in Christine’s patent application. Given only these facts, which of the following statements, if any, most accurately describes who has the right to obtain a patent?

A) Christine will obtain the patent because she was the first to file.
B) Richard will obtain the patent because he was the first to conceive and had diligence beginning before Christine’s filing date.
C) Christine will obtain the patent because, by going on vacation, Richard was not diligent.
D) Richard will obtain the patent because Christine did not actually reduce the invention to practice.
E) Christine will obtain the patent because Richard’s patent attorney did not diligently prepare and file the patent application.

A

B) Richard will obtain the patent because he was the first to conceive and had diligence beginning before Christine’s filing date.

Reasoning: The key to getting this question correct is to accurately determine which law will apply. Christine filed her application on December 31, 2012, which means her application is entitled to pre-AIA treatment. Richard filed his application on March 18, 2013, which means his application will receive AIA treatment. However, in this case, the claims of the two applications are identical, which means there will be a contest at the Patent Office to determine who is entitled to receive the claims. In the situation where one application is entitled to pre-AIA treatment and the other application is entitled to AIA treatment, the contest will be an Interference Proceeding, not a Derivation Proceeding. Thus, Christine will not prevail simply because she was the first to file, and (A) is incorrect. In fact, on these facts, Christine will not prevail at all. Richard conceived first and filed second. As the junior party, for him to prevail, he would have had to be diligent from immediately before the conception of the senior party (i.e., Christine) straight through and uninterrupted until his reduction to practice. Richard did break diligence by going on vacation, but the facts state that he once again diligently worked on his invention prior to Christine’s conception. This restarted diligence is acceptable as long as it now goes continuously through the time of Richard’s reduction to practice, which can either be actual or constructive. This time, Richard remains diligent until he achieves a working prototype (i.e., actual reduction). Any break in diligence after that is irrelevant. Therefore, Richard will prevail. This leaves us with answers (B) or (D). (D) is incorrect because it is irrelevant whether the senior party has achieved an actual reduction to practice. Thus, (B) is the best answer.

75
Q

Russell Raven, a resident of Australia, invented a new and improved digital zoom lens while on location filing A Gorgeous Cranium in London, England, during September 2012. After returning to Australia, he decided to file an Australian patent application, which was accomplished on December 14, 2012. On December 17, 2012, while traveling in the United States, Raven, an enthusiastic amateur photographer, offered several prototype versions of his lens for sale to friends attending a party in Las Vegas, Nevada. He successfully obtained his first sale, selling a prototype version to a successful country singer and reality TV personality. Raven also took a few prototypes with him to the Academy Awards gala in Hollywood, California in March 2013, successfully offering one of his prototypes for sale on March 1, 2013. Finally ready to file a US patent application, on December 5, 2013, Raven retains a US patent attorney. He vaguely tells his new attorney of his earlier Australian filing and explains that he will send the information as soon as he gets to his hotel and opens his laptop. Raven does send the information, but apparently because he was using the Internet connection at his hotel, it was routed into the SPAM folder in his attorney’s e-mail program. Raven contacts his attorney on December 20, 2013, to inquire about the US filing and ask about the possibility of filing another patent application. It is during this call that a displeased Raven discovers that the US patent application has not yet been filed. A US patent application claiming priority to the previously filed Australian patent application is filed on December 23, 2013. The US patent application filed is identical in scope with the previously filed Australian application. Based on these facts, which of the following statements is correct?
I. The Raven US patent application will be examined under AIA 102.

II. The Raven US patent application will be examined under pre-AIA 102.

III. Raven will be able to obtain a US patent.

IV. Raven will not be able to obtain a US patent because of the sale in Las Vegas on
December 17, 2012.

V. Raven will not be able to obtain a US patent because of the offers for sale in Hollywood, California on March 1, 2013.

A) I and III
B) II and III
C) I and IV
D) II and IV
E) II and V

A

D) II and IV

Reasoning: Raven filed his application in Australia on December 14, 2012. As a general matter, he has up to 12 months to file an application in the US claiming the benefit of this foreign filing date. However, as the result of the Patent Law Treaties Implementation Act, in the situation where the failure to claim priority was unintentional, it can be restored. The PLTIA gives the applicant an additional two months to claim priority if, and only if, the failure was the result of unintentional delay. Here, the question clearly suggests that the intent was to file within the 12-month timeframe, but due to the fact that his e-mail was erroneously filed as SPAM, the application was not filed and priority was not claimed within the typical 12-month window. Thus, Raven will be able to have priority restored. As a result, the Raven US patent application will be treated under pre-AIA because the Australian filing date is prior to March 16, 2013, and because the US filing does not include any claims that enjoy priority only on or after March 16, 2013. Therefore, I is false and II is true. Unfortunately the restoration of priority does not save him. Under pre-AIA 102(b), a US patent application must be filed within 12 months of a sale or offer for sale in the US. Here, Raven offered a prototype of his invention for sale on December 17, 2012, in Las Vegas, Nevada. Therefore, he had only until December 17, 2013, within which to file a US patent application. Because the patent application was filed in the US on December 23, 2013, Raven has missed and will be barred under pre-AIA 102(b) from ever obtaining a patent on the prototype version of his zoom lens. Therefore, III is false and IV is true. The sale activity in Hollywood, California in March 2013 does not create a pre-AIA 102(b) impediment to patentability because it occurs within 12 months of his US filing; therefore, V is false.

76
Q

Avery was unhappy that her friend Joan had obtained a patent in 2012 on something that she had told Joan about. She decided she would go right to the PTO and use derivation to get the PTO to revoke the patent. She puts together an affidavit detailing the facts regarding her inventorship, a copy of the patent, and all of the other necessary requirements for the request, along with the fee. The papers are filed November 11, 2018. The request

A) Will be considered on the basis of the patents, but the affidavit will not be considered.
B) Will be considered on the basis of both the prior art patents and the inventorship issue.
C) Will be not be considered; but the inventorship issue will be referred to the PTO solicitor.
D) Will not be considered.
E) Will provide a period of 2 months for Avery to make a statement.

A

D) Will not be considered.

Reasoning: Derivation is only available for patents issuing on applications filed after March 16, 2013, AND have to be filed, no matter what, within one year of the patent issuing or the application publishing.

77
Q

Quality Chips was concerned about a patent obtained by their off-shore competitor Chips-R-Us. The patent has a filing date of October 1, 1999, and has been re-issued twice following litigation where terrific prior art showed up. Somehow, Chips-R-Us still has the patent and has indicated to customers that they were not “afraid” of Quality Chips. You’ve got some good 102(b) prior art to use against the patent in the form of a chip spec sheet available on the web at least as early as September 1, 1998. You decide to file a Reexam request and shove Chips-R-Us back to the USPTO again. You put together all the necessary elements for an ex parte reexam and detail the substantial new question of patentability. You prep and file the papers on August 1, 2012. The ex parte reexam will

A) Not be considered.
B) Be considered.
C) Be refused because of PTO “triple jeopardy” rules.
D) Be considered, but if it is pending beyond Sept. 16, 2011, the standard of review will be changed to reasonable likelihood that Quality Chips will prevail with respect to at least one claim.
E) Be considered, but if it is still pending after Sept. 16, 2012, it will automatically convert to an inter partes review.

A

B) Be considered.

Reasoning: A chip spec sheet is considered a publication under reexamination

78
Q

An application has been treated and acted upon in accordance with the Prioritized Examination procedure under 37 CFR 1.102. At this point, a Final Office action has been mailed. In reply, the applicant has filed an After Final amendment that creates a total claim count of 35. As a result:

A) The application becomes abandoned.
B) The amendment is refused entry owing to the claim count.
C) The amendment is considered in accord with ordinary After Final PTO procedure.
D) The Prioritized Examination is terminated.
E) The amendment is returned for rewriting to comply with the limit of 30 total claims. The applicant is given a non-extendable 1 month period within which to respond.

A

C) The amendment is considered in accord with ordinary After Final PTO procedure.

Reasoning: The prioritized examination is abandoned after a final office action. Therefore, the amendment is considered as normal. D was my original answer but that is a step behind.

79
Q

When filing a request for Prioritized Examination under 37 CFR 1.102, the applicant may defer:

A) The submission of an oath or declaration signed by the inventors, or in lieu thereof, a signed ADS identifying the inventors.
B) The filing fee.
C) The publication fee.
D) The Prioritized Examination fee.
E) None of the above.

A

E) None of the above.

Reasoning: The application must be complete at the time of filing.

80
Q

Roberto has just finished reading the Examiner’s answer mailed in response to the Appeal Brief he filed 6 months ago. Apparently, the Brief was very convincing since the Examiner has completely changed his interpretation of the cited art under 35 USC 102, but has again rejected all pending claims under 35 USC 103. Roberto is convinced this a new ground of rejection, but this has not been so designated in the Answer. Roberto would like to argue against the new rejection. His best course of action is to

A) Call the Examiner and advise him/her of the error.
B) Call the Board and advise the PTO that this issue should be addressed.
C) File a Reply Brief and argue against the new rejection.
D) File an affidavit completely addressing the new rejection.
E) File a Petition under 1.181 to have the new grounds of rejection so designated.

A

C) File a Reply Brief and argue against the new rejection.

Reasoning: 1207.03(b) “This procedure should be used if an appellant feels an answer includes a new ground of rejection that has not been designated as such and wishes to reopen prosecution so that new amendments or evidence may be submitted in response to the rejection. However, if appellant wishes to submit only arguments, the filing of a petition under 37 CFR
1.181 is not necessary because appellant may submit arguments in a reply brief if they are responsive to arguments the examiner raised for the first time in the examiner’s answer.”

81
Q

The Xyborg company, devoted to Artificial Intelligence, has created a new Examiner’s Answer writing software for the PTO. It scans the existing file for evidence to help the Examiner bolster their arguments and make them appeal-proof. In one such Answer, the Examiner has listed as Evidence an affidavit that, although filed, was not admitted into the record by the Examiner. Such Evidence:

A) Is a part of the file and may be relied on by either the Examiner or the Applicant.
B) Should be remanded for consideration by the Examiner.
C) Is helpful, but can only be considered by the Board, not the Examiner.
D) Cannot be relied on unless admitted via petition to the Director under Rule 1.181.
E) Is admitted to the record by virtue of the Examiner’s reliance thereon.

A

D) Cannot be relied on unless admitted via petition to the Director under Rule 1.181.

Reasoning: Only Evidence that has been admitted and considered may form the basis of the Appeal. Such non-admitted evidence cannot be relied on by either the Examiner or Applicant.

82
Q

An acceptable Appeal Brief in the USPTO includes citations of case law authority to

A) Either the United States Reports or the West Reporter system.
B) the Federal Reporter only.
C) The United States Patent Quarterly only.
D) Commissioner’s Decisions only.
E) Case copies provided with the Brief only.

A

A) Either the United States Reports or the West Reporter system.

Reasoning: The Board has a preference for United States Reports and West but, when not covered, any other reporter system is acceptable.

(B)-(E) are all acceptable, but are wrong because of the use of “only.” The Board rules no longer require citation to a particular reporter system and requires copies only when a case is not found in either US Reports or West.

83
Q

The following may NOT be omitted in the preparation of an Appeal Brief at the USPTO.

A) A statement of the real party in interest.
B) A statement of related Appeals and Interferences.
C) A statement of the Status of Claims.
D) A statement of the Status of Amendments.
E) A claims appendix.

A

E) A claims appendix.

Reasoning: E is required! Omitted means removed. “What may not be removed” is the question.

84
Q

In the Examiner’s Answer that Ben was reading, the Examiner had argued the original rejections contained in the file but had also, in addition, argued that the references had enough alternative structures to contemplate other variations on the existing rejection. The Examiner said the alternatives merely elaborated on what was already in the existing rejection. Ben should

A) File a Petition under 1.181 to have the alternatives designated as a new ground of rejection and reopen prosecution.
B) File a Reply Brief with arguments.
C) File an RCE and add subject matter to the application to get a newer filing date, and update the arguments to address the alternatives.
D) A and B.
E) Reopen prosecution by filing an amendment.

A

B) File a Reply Brief with arguments.

Reasoning: The examiner didn’t cite the other structures, just indicated them. So it will likely not constiitute a new rejection.

85
Q

If a substitute examiner’s answer is written in response to a remand by the Board and further includes a new ground of rejection, the appellant must, within two months of the date of the substitute examiner’s answer,

A) Reopen prosecution by filing an amendment under 37 CFR 1.111.
B) Maintain the Appeal, by filing a reply brief.
C) A or B.
D) A and B.
E) All of the above.

A

E) All of the above.

Reasoning: Kind of a trick question. It is meant to be A or B. But, if any form of amendment accompanies a Reply Brief, it will treated as a request to reopen prosecution. Page 43 of MPEP 1200

86
Q

Christina Shelton owns City Slicker Pet Resorts, Inc. She has been catering to urban professionals with too much money who want to give their pets a true rural spa day at a secluded ranch. She has been employing what many in the industry believe to be manipulative mind games on the pets in her care. She seems to whisper to them and they suddenly become calm, which makes uptight pet owners enormously happy and willing to spend exorbitant sums sending their unruly pets for “treatment.”

Shelton learns that Adam Green has filed a patent application on a method of controlling unruly pets. Green filed a non-provisional utility patent application on March 9, 2012, which claims the benefit of an earlier filed provisional patent application filed on March 11, 2011. The application was published in due course on September 13, 2012. Because this is an active area of innovation, the examiner has not yet picked up the case to begin working on the First Office Action on the Merits. Green’s method relies upon a relatively harmless, but highly effective, herbal additive to the animal’s food.
Shelton approaches you because she wants to do whatever she can to make sure that Green is not awarded a patent. She presents to you several articles written in Javanese, which is the language of the Javanese people who are from the central and eastern parts of the island of Java in Indonesia. Shelton claims that if the patent examiner were to know about these documents, the patent claims would all be rejected.
Based on the above facts, which of the following statements, if any, are true and accurate regarding the availability of a third-party submission to present the patent examiner with the Javanese articles?

A) Third-party submissions are available only to applications filed on or after September 16, 2012, and therefore cannot be used in this situation.
B) Third-party submissions are only available to be used in non-provisional patent applications, design patent applications, plant patent applications or continuations. Because Green’s application claimed the benefit of a provisional patent application filing date third-party submissions are not available for use.
C) Third-party submissions are limited to U.S. patents, U.S. published patent applications, foreign patents or foreign published patent applications. Non-patent literature cannot be submitted as part of a third-party submission.
D) Third-party submissions must include an English language translation of any non-English language item identified, so it will be necessary to translate the articles from Javanese into English.
E) Third-party submissions must be filed prior to the publication of an application, and therefore, cannot be used in this situation.

A

D) Third-party submissions must include an English language translation of any non-English language item identified, so it will be necessary to translate the articles from Javanese into English.

Reasoning: D was so apparently true, why wasn’t it chosen right away? The rule is 6 months after publication or the date of first rejection, the later of those two.

87
Q

Howard Wolowitz is the inventor of the next generation, in-flight, self-contained toilet system used for deep space travel. He filed a non-provisional utility patent application on January 17, 2018, which claims the benefit of an earlier filed provisional patent application filed on January 18, 2017. The application was published in due course on July 18, 2018. On October 25, 2018, the patent examiner issued a First Office Action on the Merits, which rejected all claims. Wolowitz, by and through his patent attorney, responded with an Amendment of the claims on January 24, 2019. By action mailed on May 16, 2019, the examiner again rejected all claims, but suggested that if claim 1 were amended to include the limitations presented in dependent claim 2, it would be allowable. Wolowitz responded with an Amendment as the examiner suggested, which placed the application in condition for allowance, on September 29, 2019, together with the appropriate request for automatic extension and the required fee.

On October 2, 2019, Dr. Rajesh Koothrappali comes to your office. He is a very angry man because he has just learned that his one-time friend, Howard Wolowitz, has applied for a patent on an invention that Koothrappali knows not to be patentable. Koothrappali presents his Ph.D. thesis bearing a date stamp of August 3, 2016, which is the day it was indexed in the University library. He says that his thesis almost completely describes Wolowitz’s alleged invention. You review the thesis and come to the conclusion that the Wolowitz claimed invention contains only minor, obvious differences when compared with the Koothrappali thesis.

Based on these facts, what is the best path to pursue as Koothrappali seeks to challenge the Wolowitz application?

A) Do nothing, because Wolowitz has placed his application in condition for allowance. Therefore, it is already too late to file a third-party submission or protest in the Wolowitz application.
B) Immediately prepare and file a preissuance submission because the patent examiner has not yet issued a Notice of Allowance. If a preissuance submission is filed prior to the examiner issuing a Notice of Allowance, it will be considered timely.
C) Do nothing; it is already too late to file a third-party submission, because a third-party submission would have to have been filed prior to January 18, 2019.
D) Immediately prepare and file a preissuance submission, because one may be timely filed at any point in time prior to the issuance of the patent.
E) File a protest, because it is already too late to file a third-party submission of prior art.

A

C) Do nothing; it is already too late to file a third-party submission, because a third-party submission would have to have been filed prior to January 18, 2019.

Reasoning: MPEP Chapter 1100. The preissuance submissions by third parties applies to any patent application (filed before, on or after September 16, 2012). § 1.290(b) sets forth the rules relative to the deadline for filing a preissuance third-party submission. A third-party submission must be filed prior to the earlier of: (1) The date a notice of allowance; or (2) The later of: (i) 6 months after the date on which the application is first published by the USPTO, or (ii) The date of the first rejection of any claim by the examiner.

The Wolowitz application published July 18, 2018. Six months after that publication is January 18, 2019. The date the first rejection was given or mailed was October 25, 2018. Therefore, the later of these two dates is January 18, 2019. Thus, a third-party submission would have to be filed no later than the earlier of the Notice of Allowance, which has not yet occurred based on these facts, or January 18, 2019. Thus, a third-party preissuance submission of prior art must have been filed prior to January 18, 2019.

88
Q

Questions 12 and 13 are based on the following fact pattern: Miguel Hagarthy is the inventor of a hydraulic utility system for supplying a pressurized fluid to a number of user devices. He filed a nonprovisional patent application on July 20, 2008, and was ultimately awarded a patent on June 3, 2011. Upon receiving a Notice of Allowance, he contacted Norman Heathrow, a well-known patent litigator who specializes in representing so-called non-practicing entities in patent infringement matters. Hagarthy has reason to know that Narcissist Technologies is selling a hydraulic system that infringes upon at least one of the claims in his patent.
At the suggestion of Heathrow, an action is filed in the United States Federal District Court for the Eastern District of Texas. A complaint is also filed with the International Trade Commission (ITC). The ITC institutes an investigation. In the ITC proceeding, Hagarthy submits a particular interpretation of the claims in question, along with various claim charts and supporting documentation. As the case proceeds in the Eastern District of Texas, Hagarthy again submits documents and argument relative to what he believes is the proper interpretation of the claims at issue. The documents and argumentation to the district court are different than those submitted to the ITC.
Paranodial Automotive, Inc. (PA) fears that they may be next on the list for Hagarthy and Heathrow. They approach you and inquire about the possibility of filing something with the Patent Office to put forth the inconsistent interpretations of the claim scope, as well as certain other prior art they would like to be made of record in the event that someone decides to file a request for reexamination.
Which of the following is in accordance with USPTO practice and procedure?

A) PA can file a written submission with the USPTO that includes prior art consisting of patents or printed publications, and they may also file documents showing the different patent owner interpretations of the claims in the district court and ITC.
B) PA can file a written submission with the USPTO that includes prior art consisting of patents or printed publications.
C) PA can file a written submission with the USPTO that includes documents showing the different patent owner interpretations of the claims in the district court and ITC.
D) Because PA is not yet an “interested party” as that term is defined in the America Invents Act, they do not have standing to file a written submission with the USPTO.
E) In order for PA to file a written submission with the USPTO, it must be made in conjunction with a simultaneously filed request for ex parte reexamination.

A

B) PA can file a written submission with the USPTO that includes prior art consisting of patents or printed publications.

Reasoning: 37 CFR 1.501: Information content of submission: At any time during
the period of enforceability of a patent, any person may file a
written submission with the Office under this section, which is
directed to the following information:
(1) Prior art consisting of patents or printed publications
which the person making the submission believes to have a
bearing on the patentability of any claim of the patent; or
(2) Statements of the patent owner filed by the patent
owner in a proceeding before a Federal court or the Office in
which the patent owner took a position on the scope of any claim
of the patent. Any statement submitted under this paragraph
must be accompanied by any other documents, pleadings, or
evidence from the proceeding in which the statement was filed
that address the written statement, and such statement and
accompanying information under this paragraph must be
submitted in redacted form to exclude information subject to an
applicable protective order.

The ITC is not a federal court

89
Q

Questions 12 and 13 are based on the following fact pattern: Miguel Hagarthy is the inventor of a hydraulic utility system for supplying a pressurized fluid to a number of user devices. He filed a nonprovisional patent application on July 20, 2008, and was ultimately awarded a patent on June 3, 2011. Upon receiving a Notice of Allowance, he contacted Norman Heathrow, a well-known patent litigator who specializes in representing so-called non-practicing entities in patent infringement matters. Hagarthy has reason to know that Narcissist Technologies is selling a hydraulic system that infringes upon at least one of the claims in his patent.
At the suggestion of Heathrow, an action is filed in the United States Federal District Court for the Eastern District of Texas. A complaint is also filed with the International Trade Commission (ITC). The ITC institutes an investigation. In the ITC proceeding, Hagarthy submits a particular interpretation of the claims in question, along with various claim charts and supporting documentation. As the case proceeds in the Eastern District of Texas, Hagarthy again submits documents and argument relative to what he believes is the proper interpretation of the claims at issue. The documents and argumentation to the district court are different than those submitted to the ITC. Paranodial Automotive, Inc. (PA) fears that they may be next on the list for Hagarthy and Heathrow. They approach you and inquire about the possibility of filing something with the Patent Office to put forth the inconsistent interpretations of the claim scope, as well as certain other prior art they would like to be made of record in the event that someone decides to file a request for reexamination.
Before PA is able to determine what is the best course of action to pursue, Narcissist Technologies files a written submission containing Hagarthy’s claim scope statements made in the Eastern District of Texas. The following day, a request for ex parte reexamination of the Hagarthy patent is filed. The request for reexamination submitted to the USPTO, among other things, relies heavily on the statements made by Hagarthy in filings in the Eastern District of Texas.
Which of the following statements, if any, are correct with respect to how the Patent Office may use the information relative to the patent owner’s filings in the Eastern District of Texas?

A) Statements made by the patent owner in a Federal court cannot be used during reexamination proceedings.
B) Statements made by the patent owner in a Federal court may be used to determine whether there is a substantial new question of patentabiity, but may not thereafter be used to determine validity of any claim.
C) Statements made by the patent owner in a Federal court may not be used to determine whether to order reexamination of the Hagarthy patent, but if reexamination is ordered, such statements may be considered if they have previously been made a part of the file.
D) Statements made by the patent owner in Federal court can only be considered by the Patent Office if there are contradictory interpretations of the same claim offered by the patent owner.
E) None of the above.

A

C) Statements made by the patent owner in a Federal court may not be used to determine whether to order reexamination of the Hagarthy patent, but if reexamination is ordered, such statements may be considered if they have previously been made a part of the file.

Reasoning: 37 CFR 1.515: … A statement and any accompanying information submitted pursuant to § 1.501(a)(2) will not be considered by the examiner when making a determination on the request …

90
Q

Humphrey Totter is an inventor businessman and sometimes philanthropist. His latest entrepreneurial endeavor relates to a motor-controlled macro rail for close-up focus-stacking photography. It has come to his attention that there is an issued U.S. Patent to Koda Poliride which could present problems for him if he were to move forward. In researching the Poliride patent, Totter uncovers various litigations and enforcement actions brought by Poliride. It seems he is quite litigious and vigorously enforces his rights whenever possible.
As Totter continues to read the various litigation filings, he notices that Poliride has made important although nuanced statements that seem to conflict. For example in the U.S. Federal District Court for the Eastern District of Texas, attorneys for Poliride have made arguments regarding the proper interpretation of the term “controller communicatively coupled” which seem to contradict the arguments made by Poliride in the U.S. Federal Court for the Northern District of California.
Totter comes to your office for assistance. He is uncertain whether he will pursue this business endeavor and he is not willing to pay your fee to challenge the patent but he does think that the USPTO should be made aware of Poliride’s contradictory interpretations of what the claims mean. He also indicates that if he does move forward he may well want to challenge the patent at some future time. In accordance with his wishes you file a written submission with the Patent Office.
Subsequently Totter learns that counsel in California and Texas have raised the question of seemingly contradictory statements made by Poliride. Both district courts rejected the assertion that the statements by Poliride were contradictory. No appeal was taken in either case on this particular issue.
What if anything must be done to notify the USPTO?

A) As a registered patent practitioner you have a duty to supplement the submission made and notify the Patent Office that the district court judges have disagreed with your written assertions.
B) As the real party in interest Totter has an obligation to supplement the submission made to the USPTO to reflect later-learned information of a probative nature.
C) Both Totter and you have a duty to supplement the submission made to the Patent Office.
D) There is a duty to continue to supplement the submission but only if the party who filed the written submission subsequently files a request for ex parte reexamination.
E) There is no continuing duty to supplement the submission.

A

E) There is no continuing duty to supplement the submission.

Reasoning: During the comment phase after proposing rules some questioned the USPTO with respect to whether there was a continuing duty to supplement any written submission. The USPTO responded: “The statute does not impose a continuing duty to supplement any submissions made pursuant to 35 U.S.C. 301(a)(2). Should a party determine that a subsequent submission is needed one can be filed in accordance with § 1.501.”

91
Q

Sally Ramulant was the inventor of the world’s first cloaking device. She conceived of this innovation while employed by Neutral Zone, Inc. The invention was reduced to practice by Sally and others working for Neutral Zone. The surprising success of the invention has caused quite a stir in certain science fiction communities and on the convention circuit. Sally, a deeply private person who prefers to remain anonymous and simply observe the interaction of others, has been inundated with requests for interviews and has been accosted by various individuals in a variety of costumes. She took out a restraining order, to no avail, and has mysteriously vanished. It is unclear at this time whether she has been abducted by aliens, is in hiding or whether she has successfully employed an improved version of the cloaking device that makes her completely invisible and anonymous. What is certain, however, is that she has not reported to work (at least visibly) for weeks and no longer answers her phone or responds to e-mails.
It is now necessary to file a patent application at the United States Patent and Trademark Office claiming the cloaking device invented by Sally. Which of the following would best be in accordance with USPTO practice and procedures for an application filed on or after September 16, 2012.

A) Neutral Zone must demonstrate that Sally was under both an obligation to assign the invention and that she cannot be found after reasonable efforts to locate her, in order to be able to file a patent application without the inventor executing an oath or declaration.
B) An inventor’s oath or declaration must always be filed when a patent application is filed in the United States Patent and Trademark Office.
C) Each individual who is an inventor or joint inventor of a claimed invention in an application for patent shall execute an oath or declaration in connection with the application.
D) Each individual who is an inventor or joint inventor of a claimed invention in a non-provisional patent application must execute an oath or declaration in connection with the application.
E) If Neutral Zone can demonstrate that Sally was either under an obligation to assign the invention or that she cannot be found after reasonable efforts to locate her, a patent application may be filed without executing an oath or declaration.

A

A) Neutral Zone must demonstrate that Sally was under both an obligation to assign the invention and that she cannot be found after reasonable efforts to locate her, in order to be able to file a patent application without the inventor executing an oath or declaration.

Reasoning:
35 U.S.C. 115(a) says that when one files a patent application under either 35 U.S.C. 111(a) (i.e., a nonprovisional patent application) or enters the national stage under 35 U.S.C. 371, each inventor must be named and each inventor or joint inventor must execute an oath or declaration unless “otherwise provided.” It is 35 U.S.C. 115(b) that relates to the exception for each inventor to submit an oath or declaration. In lieu of executing an oath or declaration under subsection 115(a), the applicant for patent may provide a “substitute statement.” The substitute statement must identify the individual for whom the statement pertains. Furthermore, the substitute statement must either state that the inventor identified is under an obligation to assign and refused to execute the oath or declaration (see 115(d)(2)(B)) or that the inventor is deceased, legally incapacitated or cannot be found after reasonable effort (see 115(d)(2)(A)).

(E) is not correct because both elements need to be in the substitute statement. It is an assignment plus “either/or” situation.

92
Q

Which of the following applicants may file and prosecute the applicant’s own case without retaining the services of a patent practitioner?

A) An inventor
B) A corporation
C) An organizational assignee
D) A single member limited liability company established by an inventor to own the patent rights
E) All of the above

A

A) An inventor

Reasoning:
MPEP 401, 37 CFR 1.31 An applicant for patent may file and prosecute the applicant’s own case, or the applicant may give power of attorney so as to be represented by one or more patent practitioners or joint inventors, except that a juristic entity (e.g., organizational assignee) must be represented by a patent practitioner even if the juristic entity is the applicant. The Office cannot aid in the selection of a patent practitioner.

93
Q

Joseph Anthony is the inventor of a revolutionary process for selecting the ultimate fantasy football roster. Unfortunately, Anthony does not possess the financial resources to prepare and file a patent application protecting his method, nor does he have the financial resources to hire a programmer to code up the system for use. He pitches the idea to Richard Aloysius, who likes it and offers to partner with Anthony. Anthony and Aloysius form a limited liability company called JARA LLC. Anthony contributes the intellectual property rights, assigning the invention to JARA. Aloysius contributes the funds necessary for the patent process and development of the system. You are retained and asked to prepare and ultimately file a patent application on behalf of JARA. Aloysius is your main point of contact because he is in charge of paying your bill and manages the day-to-day business operations of JARA. Aloysius approaches you to tell you that while he trusts Anthony, he understands that recent changes to U.S. patent law mean that the inventor does not need to be listed as the “applicant” on the patent application. He would greatly prefer if the patent were issued to JARA and not Anthony directly.

If you file a nonprovisional application in the name of JARA LLC, which of the following is true?

A) You will receive a Notice of Missing Parts for failure to file the application in the name of the inventor. You will be given a shortened statutory period of 2 months to respond, which can be fully extended.
B) You will receive a Notice of Omitted Items for failure to file the application in the name of the inventor. You will be given two months to respond, which cannot be extended.
C) This is acceptable, but the application must contain an application data sheet specifying in the applicant information section that JARA LLC is the assignee.
D) This is acceptable, but Anthony and Aloysius must both execute either an oath or declaration and file it in the application before the patent examiner can issue a Notice of Allowability.
E) This is acceptable, but Anthony and Aloysius must both execute either an oath or a declaration and file it in the application no later than the expiration of the time period set in the Notice of Allowability.

A

C) This is acceptable, but the application must contain an application data sheet specifying in the applicant information section that JARA LLC is the assignee.

Reasoning: Pursuant to 37 CFR 1.46(b), if an application is made by a person other than the inventor the application must contain an application data sheet specifying the assignee, the person to whom the inventor is under an obligation to assign the invention, or a person who otherwise shows sufficient proprietary interest in the matter.

A and B are incorrect because the American Invents Act and associated USPTO rules now allow for one other than the inventor to file a patent application. Specifically, an assignee is considered an “applicant” and may file.

With respect to D and E. Section 1.495(c)(3)(ii) provides that the applicant must file an oath or declaration in compliance with § 1.63, or substitute statement in compliance with § 1.64, executed by or with respect to each actual inventor no later than the expiration of the time period set in the “Notice of Allowability” to avoid abandonment. However, both D and E are incorrect because Aloysius is not an inventor and would not be required to file an inventor’s oath or declaration. Furthermore, even if Aloysius were an inventor, D would be incorrect because the patent examiner can and will issue a Notice of Allowability prior to the filing of an oath or declaration. The Notice of Allowability will set the due date by which the oath or declaration, or substitute statement, must be filed.

94
Q

Keith Kensyr is the inventor of a corner-mounted light fixture, which was awarded U.S. Patent No. 7,866,999 on July 12, 2011. Kensyr has been attempting to sell his patent rights to Under the Cabinet Lighting, Inc. After getting close to a deal, talks ultimately broke off without either an assignment or license. Kensyr has now learned that UCL has made and is distributing a corner-mounted light fixture that looks eerily similar to the device claimed in the ‘999 patent. Kensyr comes to you to discuss bringing a patent infringement action against UCL.

Because Kensyr is seeking a contingency fee representation agreement, you do a quick patent search to see whether it looks like the prior art considered by the patent examiner seems to be the best available prior art. During your search, you identify U.S. Patent No. 7,226,888, to Feeley, which relates to a device for mounting an object at the corner of the room. The ‘888 patent was not disclosed by the patent practitioner representing Kensyr, and it was not located by the patent examiner in his search. You raise this with Kensyr and he says: “Oh, you found that one. Yeah, my patent attorney missed that one in the search, which made me happy because I thought for sure if the patent examiner knew about that, I wouldn’t have a prayer in the world of getting a patent.”

It is your belief that the ‘888 patent is not as bad as Kensyr thinks, but that it would have been prudent to have disclosed it to the examiner. The claims are still patentable, but this issue will certainly distract from any patent litigation and may wind up with the patent being declared unenforceable if the wrong district court judge is assigned the case.
What should you advise Kensyr?

A) No harm, no foul. It is best to ignore this matter and simply address it in litigation if and when UCL ever discovers the issue.
B) Filing a patent infringement lawsuit is fine because patents are presumed valid. If and when UCL ever discovers this issue, we can at that time file a request for supplemental examination.
C) Because there was willful and material fraud perpetrated on the Patent Office by the inventor, there is no way to correct this matter now.
D) It would be wise to file a request for supplemental examination before filing a patent infringement action.
E) If you file a request for supplemental examination, it is likely that the USPTO will refer the matter to the United States Attorney General because a material fraud has been perpetrated on the Office.

A

D) It would be wise to file a request for supplemental examination before filing a patent infringement action.

Reasoning: Supplemental examination is a desirable procedural path to follow because it can inoculate a patent owner against potential charges of inequitable conduct.
A and B are incorrect because inequitable conduct is always raised during patent litigation, and 35 U.S.C. 257(c) only absolves inequitable conduct if the supplemental examination and any reexamination ordered pursuant thereto are concluded before the date on which a patent infringement action is brought.
C and E are incorrect because this is not a situation where there is “material fraud.” The USPTO says that they will interpret “material fraud” to be more stringent than inequitable conduct. Inequitable conduct requires both materiality and intent. The question says that the claims are still patentable and the ‘888 patent is not as bad as Kensyr thinks. So while he may have intended to deceive the USPTO, there is no materiality. If there is no inequitable conduct, there could not be any “material fraud.”

95
Q

Which of the following challenges may be the subject of a petition for review of a covered business method patent in accordance with USPTO practices and procedures?
I. 35 U.S.C. 101
II. 35 U.S.C. 102
III. 35 U.S.C. 103
IV. 35 U.S.C. 112, enablement
V. 35 U.S.C. 112, written description
VI. 35 U.S.C. 112, best mode

A) II and III
B) I, II and III
C) I and IV, V and VI
D) II, III, IV, V
E) I, II, III, IV and V

A

E) I, II, III, IV and V

Reasoning: A covered business method patent review is conducted as a post-grant review.

96
Q

Under which of the following circumstances would it be inappropriate for a petitioner to seek to institute a covered business method patent review?

A) The petitioner has been sued for infringement of the patent.
B) The petitioner has been charged with infringement of a patent.
C) The petitioner has been offered a license to the patent.
D) A privy of the petitioner has been charged with infringement of a patent.
E) A privy of the petitioner has previously challenged different claims of the patent.

A

C) The petitioner has been offered a license to the patent.

Reasoning: Merely being offered a license to a patent would not support the filing of a petition for a covered business method patent review.
37 CFR 42.302(a), which is consistent with Section 18 of the AIA, explains that a petitioner may not institute a covered business method patent review of the patent unless the petitioner, the petitioner’s real party-in-interest, or a privy of the petitioner has been sued for infringement of the patent or has been charged with infringement under that patent. Charged with infringement is defined to mean a real and substantial controversy regarding infringement of a covered business method patent exists such that the petitioner would have standing to bring a declaratory judgment action in Federal court. Therefore, a challenge may be brought in A, B and D.
Furthermore, 37 CFR 42.302(b) explains that a petitioner may not file a petition to institute a covered business method patent review of the patent where the petitioner, the petitioner’s real party-in-interest, or a privy of the petitioner is estopped from challenging the claims on the grounds identified in the petition. Therefore, a challenge may be brought in E.

97
Q

Under which circumstances would it be inappropriate for a substitute statement to be filed in lieu of an oath or declaration?

A) The inventor is in a coma.
B) The inventor has suffered a head trauma and now exhibits signs of dementia.
C) The inventor has assigned the rights to the invention.
D) The person filing the substitute statement has reviewed the contents of the application.
E) The person filing the substitute statement is the assignee and the inventor is unavailable.

A

C) The inventor has assigned the rights to the invention.

Reasoning: This isn’t enough reasoning to sign a substitute statement. A and B is legal incapacity. D is required to sign a substitute statement. E is required to sign a substitute statement.

98
Q

Which of the following may not file a patent application in the United States?

A) An administrator of the estate of a deceased inventor
B) A joint inventor
C) A non-exclusive licensee
D) The inventor’s employer.
E) A person who shows sufficient proprietary interest in the matter.

A

C) A non-exclusive licensee

Reasoning: A non-exclusive licensee of the innovation would not have sufficient proprietary interest to file a patent application.

99
Q

Which of the following is not incorrect?

A) An oath made in the United States must be made before either a judicial official or a registered patent practitioner.
B) The oath or declaration does not always need to identify each inventor by his or her legal name; nor does the oath or declaration always need to identify a mailing address where the inventor customarily receives mail, and residence.
C) An oath or affirmation made in a foreign country must be administered by diplomatic or consular officer of the United States authorized to administer oaths.
D) Deficiencies in an oath or declaration cannot be fixed without filing either an RCE under Rule 1.114 or a Continuation under 37 CFR 1.53.
E) None of the above.

A

B) The oath or declaration does not always need to identify each inventor by his or her legal name; nor does the oath or declaration always need to identify a mailing address where the inventor customarily receives mail, and residence.

Reasoning: All the things in B can be supplemented with an ADS, making B true. C is too restrictive.

100
Q

In which of the following circumstances would a patent application be eligible for entry into the Patent Prosecution Highway (PPH) under the rules and procedures in effect at the United States Patent and Trademark Office (USPTO)?

I. You must have received a patent from a partnering PPH office.
II. The application with allowable subject matter and the application you file at the USPTO share a common earliest priority date.
III. A Notice of Missing Parts has been received, but not yet responded to, in the application filed at the USPTO prior to receiving an indication of allowable subject matter from a partnering PPH office.

A) I
B) II
C) III
D) I and III
E) II and III

A

E) II and III

Reasoning:

In order to be eligible for entry into the Patent Prosecution Highway (PPH), the applicant must have already received an indication of allowable subject matter in a related patent application from a partnering PPH office. The indication of allowable subject matter can come before the patent actually issues in the Office of Earlier Examination (OEE). Therefore, (I) is incorrect.

In order to partake in the PPH, it is a requirement that the application with allowable subject matter from a partnering PPH office must share the common earliest date of priority. Therefore, (II) is correct.

The rule for entry into the PPH says that examination cannot have started in the U.S. application prior to entry into the PPH. This requirement however relates to substantive examination. The USPTO has specifically explained: “if you have only received a notice to file missing parts, this would not be considered as substantive examination.” Thus, in this question, you have received but have not yet responded to the Notice of Missing Parts, which means that no substantive examination could possibly have started. Therefore, you could still enter the PPH.

101
Q

Which of the following statements is not true?

A) A proper showing of secondary considerations rebuts a prima facie obviousness rejection.
B) An inventor may be excluded from participating in the prosecution of his or her own application if the assignee so wishes.
C) Unless instructed otherwise, the Patent Office will send all further correspondence to the appointed attorney or agent of record.
D) An Information Disclosure Statement (IDS) should be filed within two months from the filing of a provisional application.
E) A reexamination can be filed up to six years after the patent expires.

A

D) An Information Disclosure Statement (IDS) should be filed within two months from the filing of a provisional application.

Reasoning:
(D) is correct because it is the only choice that is false. You do not file an Information Disclosure Statement (IDS) in a provisional patent application. There is no prosecution of a provisional patent application and, therefore, no duty to disclose prior art.

(A) is incorrect because it properly states that secondary considerations must be considered, if present, when making an obviousness rejection.

(B) is incorrect because it is a true statement: One or more assignees may conduct prosecution of a patent application as the applicant, or conduct prosecution of a supplemental examination or reexamination proceeding, to the exclusion of the inventor or previous applicant or patent owner.

(C) is incorrect because it is a true statement: In the absence of a correspondence address being provided that is different from that of the attorney/agent of record, the USPTO will communicate with the attorney/agent of record.

(E) is incorrect because the enforceability period of a patent lasts until 6 years after the patent has expired because the Statute of Limitations to bring a patent infringement action is 6 years. Thus, a reexamination can be filed at any time during the enforceability period of the patent.

102
Q

You prepare and file a patent application on January 24, 2017, one day before the first anniversary of a publication fully describing the invention. Unfortunately in the rush to file the application page 24 was omitted from the application. That page describes certain details which are set forth in dependent claims 4 and 5, but are not otherwise necessary to satisfy the requirements of 35 USC 112. A Notice of Omitted Items was mailed from the Patent Office on April 14, 2017. You advise the client of your error and, after expressing his view of your competence, the client instructs you to take the immediate action, which will best preserve her rights at the least expense. That action is:

A) Immediately file a preliminary amendment with the missing page and ask that the application be given a filing date as of the date the preliminary amendment is filed.
B) Telephone the examiner to explain your situation and ask him to insert page 24 into the Patent Office copy of the application.
C) Do nothing, but tell the Patent Office that will be your course of action.
D) Abandon the application and file a substitute.
E) File a complete application with all pages today.

A

C) Do nothing, but tell the Patent Office that will be your course of action.

Reasoning: Preliminary amendment can’t add new matter.

102
Q

Your new client Mickey Smith discloses to you a new invention which he has made relating to an automotive battery. In 2017 he had the idea that a better battery could be made using acid which was purified to an extraordinary degree, more than 99.99% pure. He built such a battery and it worked as he expected. At the time his battery was the subject of a newspaper article which did not describe what made it work so well. He was able to interest General Motors in buying the rights to his invention, but after much negotiation, no deal was struck. He continued to work over the years at improving his battery and this year made a remarkable improvement relating to an additive to the acid. He wants to obtain the best protection available under these facts. You should advise him that:

A) He should have filed a patent application many years ago and it is too late now.
B) He should file an application directed to both the original invention and the improvement.
C) A patent cannot be obtained on the original invention because it was made more than one year ago, but an application can be filed covering the improvement.
D) A patent cannot be obtained on the original invention because it was offered for sale more than a year ago. Patent protection may be available on the improvement if it is unobvious over the original invention.
E) He should file an application directed to the original invention. If it is found patentable, the application can then be amended to cover the improvement.

A

B) He should file an application directed to both the original invention and the improvement.

Reasoning: On these facts, there is nothing to suggest that he is unable to file and obtain a patent.

102
Q

You filed an application in 2017 for your client Wilbur Jones is an independent inventor who has never paid your bill despite numerous threats and demands. A first action was sent to you on February 12, 2018, which you promptly mailed to Mr. Jones together with a letter stating that you would take no further action without payment for your past bill. The action set a three-month shortened period for response. Today, Friday, August 9, 2018, Mr. Jones appears at your office at 4:30pm without an appointment. After providing full instructions to you, tells you that he sadly will not pay anything more than the necessary Patent Office extension fees at this time because he has his heart set on buying a new Ferrari. You are, at this time, driving a Dodge Dart with 175,000 miles on it, no radio, and no back seat. You are planning to leave that night to attend the wedding of your only son, and are not scheduled to return to the office until August 13, 2018. Your spouse has made clear the consequences should you not attend the wedding and all of the festivities. The proper course of action would be:

A) Allow the application to become abandoned because Jones knew for many months your intention, can find other lawyers, and because no one is required to work for free in the United States.
B) Tell Jones that you will use the money that he gave you for the extension to file a continuation because your family is more important to you than anything, and there will be no loss to him.
C)
Prepare and file a full and complete response on or before August 12, 2018, together with the required extension of time.

D) File a petition asking the Commissioner to extend the deadline for response for two days in light of your situation and the pro-family attitude of the present administration.
E) Advise Mr. Jones that he is no longer your client and should act to avoid loss of his rights.

A

C)
Prepare and file a full and complete response on or before August 12, 2018, together with the required extension of time.

Reasoning: The question here is really whether you must respond or whether you can get away with not responding. Unfortunately, it is too late to withdraw from this case. The old rule relating to withdrawal was that you could not get out of a case if there was less than 30 days remaining on the statutory period. That rule has changed to a much more squishy rule that provides no bright line. Still, the rule says that you cannot get out of a case if there is not sufficient time for the client to find alternative representation. In this case, where you are dealing with an independent inventor as a client (i.e., one who is not sophisticated vis-à-vis the patent laws), the Patent Office will not let you dump the client at this late date because you have not left him with sufficient time to find alternative representation. Thus, you must file a complete response before the statutory period expires and pay the required extension fee.

103
Q

A design patent must be filed____________

A) With no more than one independent claim and one dependent claim directed to each embodiment.
B) Only in English.
C) Without a drawing if a drawing is unnecessary to understanding the invention.
D) Within eight months of the first filing in a foreign country if the benefit of priority is to be preserved.
E) By transmitting a copy to the International Association of Design Architects and Engineers (IADAE) at 603 Water Street, S.W. Washington D.C. 20002, which will then send a copy to the Patent Office.

A

D) Within eight months of the first filing in a foreign country if the benefit of priority is to be preserved.

Reasoning: The general rule with respect to preserving priority in a design patent case is that the design patent application must be filed within six months of the filing of the previously filed priority application. However, the Patent Law Treaties Implementation Act provides an additional two months within which to claim priority. In order to avail yourself of this additional two months to claim priority, the right to claim priority must be restored. The right to claim priority can be restored only if the delay in filing the subsequent application was unintentional. Thus, in the situation where the subsequent application was unintentionally filed outside of the six-month priority window, priority can still be established provided it is filed within the additional two months provided, but if and only if the delay was unintentional. Thus, it can be said that, to preserve priority in the case of a design application, the subsequent application must be filed within eight months of the first filing in a foreign country.

103
Q

The examiner issued a Final Rejection, which was mailed to you on June 23, 2018 and received two days later. In the action the examiner rejected claims 10 - 15 based upon two newly cited prior art references. These claims were added to the application in an amendment made responsive to the first official action. Claims 1 - 9 continue to be rejected in the Final Rejection for the same reasons set forth in the first action. Your client believes that the new claims 10 - 15 are patentable over the art, but require some amendments to add additional limitations which are disclosed in the application, but which have not yet been claimed. Claims 1 - 9 the client now agrees are not patentable. The most appropriate course of action would be:

A) File a petition asking that the finality of the action be withdrawn and file an amendment canceling claims 1 - 9, and amending claims 10 - 15.
B) File a continuation under Rule 1.53 accompanied by a preliminary amendment canceling claims 1-9, and adding new claims 10 - 15, in a non-marked-up from, with the proposed amendments included.
C) File an amendment canceling claims 1 - 9 and amending claims 10 - 15.
D) File a Notice of Appeal.
E) Abandon the application.

A

B) File a continuation under Rule 1.53 accompanied by a preliminary amendment canceling claims 1-9, and adding new claims 10 - 15, in a non-marked-up from, with the proposed amendments included.

Reasoning:

For the first time, you realize that claims 1-9 are not patentable, which means you must cease the pursuit of those claims. Otherwise, you are engaging in an ethical violation. You also realize that claims 10-15 can be made allowable but require amendment to add limitations not previously considered by the examiner, but which are present in the application (thus not new matter). The trouble is you are after final rejection procedurally. You could cancel claims after final rejection, but you cannot substantively amend claims the way you want to here after final amendment. Thus, you would need to file either an RCE (not an answer choice) or a continuation under Rule 53 in order to amend claims 10-15.

103
Q

On June 24, 2010, a former client by the name of Edgar Bilton assigned all the rights in his then pending patent to his son-in-law John in return for a promise that John would take care of Edgar’s aged poodle while Edgar was in jail. John did so and the poodle died of old age in the spring of 2011, just before Edgar died in prison. The agreement was in writing and at your urging John recorded it in the Patent Office on July 14, 2011. Today, August 26, 2013, John gets a letter from Edgar’s old cell mate Henry indicating that Edgar assigned all rights to Henry in writing on July 4, 2010, in return for several cartons of cigarettes. Henry says that he knew nothing about any previous assignment when he made the deal and learned about John’s alleged rights only last week upon his release from jail and after he attempted to assign the patent. Your advice to John would be:

A) If the facts are as Henry alleges, Henry has better title to the patent than does John.
B) Even if the facts are as Henry alleges, Henry does not have good title to the patent.
C) An inmate in a prison cannot make a valid contract, and therefore Henry does not have title.
D) Because John recorded his assignment before Henry found out about the previous transfer, John has good title.
E) Disclaim the patent now before Henry can take title.

A

A) If the facts are as Henry alleges, Henry has better title to the patent than does John.

Reasoning: In order to perfect ownership of a patent, it is necessary to record within three months. 35 U.S.C. 261 explains: “An assignment, grant, or conveyance shall be void as against any subsequent purchaser or mortgagee for valuable consideration, without notice, unless it is recorded in the Patent and Trademark Office within three months from its date or prior to the date of such subsequent purchase or mortgage.” The term “valuable consideration” means that the person obtaining the assignment cannot get it via gift, but rather must exchange something of value. The law does not require much value, so several cartons of cigarettes, which would clearly have some monetary value, will suffice. The rest of the question makes clear that Henry did not know that anyone else obtained an assignment before him. Thus, John’s rights are inferior to Henry’s rights. Of course, notice that the question does not say that Henry recorded within three months. If a subsequent bona fide purchaser acquired for value after Henry, then that subsequent purchaser would have rights superior to Henry. Thus, we cannot say that Henry owns the patent. We can only say that Henry has better title to the patent than does John. (B) is not the best choice. We don’t know whether Henry has good title (i.e., meaning he is the true owner), although he well may have good title. We do know for certain, however, that if Henry’s story checks out, he has superior rights to John; thus, (A) is a superior answer.

104
Q

On June 3, 2024, your client Wilbur, a Royal Canadian Mounted Policeman, conceived of a method making a cold weather elixir, which he reduced to practice in the Yukon within a week thereafter. He filed an application in Finland, believing that cold and snowy place a good market for his invention, but he abandoned the application a year later because it was too expensive. He secretly sold the elixir recipe to a friend in Canada during July 2024, and on August 3, 2024, he granted an exclusive license to a company in Wisconsin to manufacture the elixir. The product was fully described in its Winter catalog, which first reached customers on January 4, 2025. A U.S. application was filed on December 3, 2025.

Which of the following sections of 102 either bar patentability or provide a prior art exception to Wilbur’s application to the method?

A) 102(a)(1)
B) 102(a)(2)
C) 102(b)(2)(a)
D) 102(b)(2)(b)
E) None of the above

A

E) None of the above

Reasoning: The secret sale in Canada is of no concern because, for it to create a “sale,” the method must be performed. The sale of the recipe alone does not matter as to a later claim to the “method.” The offer of an exclusive license, like the offer of an assignment, is not a prior art trigger event. The first potential issue is the disclosure in the Winter catalog, but that occurred less than 12 months before Wilbur filed the application in the U.S., so there is no concern here either because of an exception under 102(b)(1)(a).

105
Q

McCoy conceives of and reduces to practice an invention relating to an improved masonry tool in Canada on September 25, 2008. He files a patent application in China on September 26, 2008, which is granted on September 3, 2010, but the patent file was not available to the public and published until December 23, 2010. During 2009, McCoy sold a few of the tools to various of his friends in Asia and on October 12, 2009 sold all rights in the invention to Federation Planet Enterprises (FPE), a leading manufacturer of masonry tools. Today is August 28, 2013, and FPE wants to file a U.S. patent application directed to the McCoy invention.

Which of the following sections of 35 USC 102 bar patentability of the McCoy invention?

A) 102(a)(1)
B) 102(b).
C) 102(d).
D) 102(b) and 102(d).
E) None of the above.

A

A) 102(a)(1)

Reasoning: Don’t overthink it. It’s barred from being able to be patented because it already is patented. You thought about foreign filing licenses….. don’t think about that. It asks “What sections of 35 USC 102 bar patentability”. 102(a)(1) do because it is already patented

106
Q

Regarding obviousness, which of the following are not false?

A) “Teaching away” is a valid rebuttal to an obviousness rejection only when a person of ordinary skill would be explicitly discouraged.
B) Rejections under 35 U.S.C. 103 must explain the reasoning associated with a rejection of a particular claim as being obvious.
C) A reasonable expectation of success is not a mandatory requirement for a proper obviousness rejection.
D) Cursory obviousness rejections can be sustained when it is readily apparent that the combination of references would be within the common-sense understanding of one of skill in the art.
E) None of the above.

A

B) Rejections under 35 U.S.C. 103 must explain the reasoning associated with a rejection of a particular claim as being obvious.

Reasoning: The question is looking for the answer that is “not false,” which means we are looking for the true statement. Only (B) is true. An examiner must explain the reason why a claim is being rejected as obvious. Unsubstantiated conclusions that a claim is obvious are not acceptable and do not satisfy the examiner’s burden to establish a prima facie rejection. NOTE: The examiner’s initial burden is sometimes discussed as the need for an examiner to make a prima facie case. Prima facie is a Latin phrase that literally translates to “at first face.” When spoken in the legal world, it means “on its face” or “at first appearance.” In other words, the examiner must come up with a rejection that on its face is plausible based on the examiner’s first impression.

(A) is incorrect because there is no requirement that a reference explicitly teach away for “teaching away” to be a valid rebuttal. In fact, a prior art reference must be considered in its entirety, i.e., as a whole, including portions that would lead away from the claimed invention. It is improper to combine references where the references teach away from their combination. A prior art reference that “teaches away” from the claimed invention is a significant factor to be considered in determining obviousness.

(C) is incorrect because, if there is not a reasonable expectation of success, then by definition the combination realized by the inventor would be non-obvious.

(D) is incorrect because the examiner must explain the reason for reaching the obviousness rejection.

(E) is incorrect because (B) is correct.

107
Q

Which of the following is correct with respect to a derivation proceeding initiated at the USPTO?

A) In a derivation proceeding, the applicant must detail how the invention was arrived at (or derived) so that the examiner can determine sufficiency of disclosure for purposes of 35 USC 112.
B) No corroborating evidence is required where the affidavit is submitted by the inventor because the inventor is considered to be in the best place to offer accurate, reliable, first-hand information about what actually happened.
C) Parties to a derivation proceeding may resort to binding arbitration. However, the Board will not consider the arbitration award unless it is filed within 20 days of the date of the award.
D) A petition for derivation and supporting evidence must be served by a process server or other official designated by state law to serve court filings on the respondent at the correspondence address of record for the earlier application or subject patent.
E) The petition in a derivation proceeding must explain why the claimed invention is the same or substantially the same as the invention disclosed to the respondent, but need not offer a claim construction.

A

C) Parties to a derivation proceeding may resort to binding arbitration. However, the Board will not consider the arbitration award unless it is filed within 20 days of the date of the award.

Reasoning: The text is very limited in what you can read about derivations. The only thing close to a right answer is C, which is about derivations.

Parties may resort to binding arbitration. Notwithstanding, the Board is not bound by, and may independently determine, any question of patentability. Furthermore, the Board will not consider the arbitration award unless it: (1) is binding on the parties; (2) is in writing; (3) states in a clear and definite manner each issue arbitrated and the disposition of each issue; and (4) is filed within 20 days of the date of the award.

108
Q

Smith works for General Massive Corporation (GMC) where he works with biodegradable nanoparticles. Wollensky works for Central Biotech Services (CBS), where he also researches on various biodegradable nanoparticles. On March 13, 2017, a U.S. patent application was filed by GMC on behalf of Smith, claiming a unique biodegradable composition. On May 12, 2018, a U.S. patent was granted to GMC on the Smith composition. On March 1, 2018, Wollensky independently developed the same composition as did Smith, but also developed an obvious modification of Smith’s composition. This obvious variation was not disclosed or claimed in the Smith patent application. On April 1, 2018, after a long, drawn-out legal battle with both shareholders and the Federal Trade Commission, GMC acquired CBS together with all of it assets and liabilities. On May 1, 2018, GMC filed an application on behalf of Wollensky in the United States Patent Office, disclosing and claiming both the same identical composition as disclosed in the Smith patent application, as well as the obvious variant developed only by Wollensky. On March 31, 2019, the examiner rejected all of the claims in the application over the Smith patent. The first set of claims was rejected under 35 U.S.C. 102, and a second set of claims were rejected on the ground of obviousness-type double patenting. With respect to the claims rejected under 35 U.S.C. 102, which of the following statements is correct?

A) The rejection is appropriate under AIA 35 U.S.C. 102(a)(1) because the claimed invention was described in the Smith patent, which was filed prior to the effective filing date of the Wollensky application.
B) The rejection is appropriate under AIA 35 U.S.C. 102(a)(2) because the claimed invention was described in the Smith patent, which was filed prior to the effective filing date of the Wollensky application.
C) The rejection is appropriate because GMC did not commonly own the subject matter of Smith and Wollensky at the time the inventions were made.
D) The rejection is inappropriate because the subject matter disclosed and claimed in the Wollensky application and the Smith patent were commonly owned prior to the effective filing date of the Wollensky application.
E) The rejection is inappropriate because, since the Smith application did not publish prior to the date the patent was granted, the earliest it would be considered prior art is May 12, 2018, which Wollensky can pre-date by showing an earlier date of conception.

A

D) The rejection is inappropriate because the subject matter disclosed and claimed in the Wollensky application and the Smith patent were commonly owned prior to the effective filing date of the Wollensky application.

Reasoning: With any questions related to 102, the first question you must ask is what law applies. The question deals with a rejection in the Wollensky application, which was first filed on May 1, 2018; therefore, AIA 102 will apply. Under AIA 102, a disclosure that is prior art under AIA 102(a)(2) is not prior art if the subject matter disclosed and the claimed invention, not later than the effective filing date of the claimed invention, were owned by the same entity or subject to an obligation of assignment to the same entity. So the Smith patent would be prior art under AIA 102(a)(2). It would not be prior art under AIA 102(a)(1) for Wollensky because the Smith application did not publish and the patent was not granted until after the Wollensky application was filed. Therefore, as long as Smith and Wollenksy were commonly owned by the effective filing date of the Wollenksy application, the Smith patent cannot be used against Wollensky under AIA 102(a)(2). Therefore, the rejection is inappropriate and (D) is the correct answer. (A), (B) and (C) are all incorrect because the rejection is inappropriate.

(A) is further incorrect because the Smith patent would not be prior art under 102(a)(1), and still further because 102(a)(1) prevents a patent in the situation where the claimed invention was patented, described in a printed publication, in public use, on sale, or otherwise publicly available before the effective filing date of the claimed invention. (C) is further incorrect because, since the passage of the AIA, common ownership no longer needs to be as of the date of invention, as was the case for double-patenting under pre-AIA.

(E) is incorrect because, under the AIA, the date of conception is no longer relevant.

109
Q

An invention relates to a bicycle having a “shaped handlebar” that provides improved aerodynamic properties for bicycles. The invention is described in a pending U.S. patent application. The “shaped handlebar” is disclosed as being only “Y” shaped. The application as filed, however, contained only a single claim (i.e., claim 1) to the bicycle having a “shaped handlebar.” Claim 1 was properly rejected under 35 U.S.C. 102 as anticipated by a U.S. patent to Jones, which discloses a “V” shaped handlebar on a bicycle. Claim 1 was then amended to add a bicycle wheel structure not disclosed or suggested by the Jones patent, but which is supported in the original disclosure of the application, although only in the drawings. Dependent claims 2 and 3 are added and provide further limitations to the invention. Claim 2 is dependent from claim 1 and further defines the handlebar as being “Y” shaped. Claim 3 is also dependent from claim 1 and further defines the handlebar as being “U” shaped. Which of the following statements is true?

A) Claim 3 is unpatentable under 35 U.S.C. 112 as being indefinite.
B) Claim 2 is unpatentable under 35 U.S.C. 112(d) because it does not further limit the subject matter of independent claim 1.
C) Claim 3 is unpatentable under 35 U.S.C. 112(a) since the description requirement is not satisfied.
D) Claim 2 is unpatentable under 35 U.S.C. 132 as being drawn to new matter.
E) Claims 2 and 3 are unpatentable under 35 U.S.C. 102 as being anticipated by the Jones patent.

A

C) Claim 3 is unpatentable under 35 U.S.C. 112(a) since the description requirement is not satisfied.

Reasoning: The addition of a wheel structure not disclosed in the Jones patent will successfully cause claim 1 to overcome the 102 rejection based on Jones. It does not matter that the wheel structure was only supported in the drawings. Since the wheel structure was present at the time of the original filing, it will not be considered to be new matter. Now, more directly, the call of the question: Are claims 2 and 3 patentable? While claim 2 may be unpatentable because it depends from claim 1, it will not be unpatentable for the reasons set forth in (B), (D) and (E). Therefore, (B), (D) and (E) are incorrect.

More specifically, the limitation added in claim 2 is a limitation that requires the handlebar to be “Y” shaped. This is supported in the original disclosure, so it is not new matter. Furthermore, it narrows claim 1 because in claim 1 the handlebar need only to be a “shaped handlebar.” Therefore, the introduction of a specific shape narrows from any shape to the “Y” shape. Claim 3 will not be patentable. There is nothing on these facts to tell us one way or another whether the claim will be indefinite, thus (A) is incorrect. What we know about claim 3 is that it is dependent from claim 1, but the shape limitation inserted is a “U” shape. While this does narrow the claim, there is nothing in the question to suggest that the “U” shape embodiment has been disclosed in the original filing. In fact, the question specifically says that the only specifics given in the disclosure state that the handlebar is “Y” shaped. Thus, the written description does not support the claim, which means that the claim is unpatentable under 112(a).

110
Q

William and Fran are employees of ACME. They work together on a project and jointly invented an improved electrode for a battery. William and Fran turn their work over to their supervisor, Alex. Alex discusses the invention with William and Fran and then turns the project over to Mary, who is patent counsel for ACME. Without reviewing the project details, Mary turns the file over to an associate, Albert, who is a newly minted patent practitioner. Albert prepares a draft application, which Mary reviews and files on December 5, 2018. On March 18, 2019, Alex stumbles across an article that relates to William and Fran’s invention. Alex notifies Albert. Alex and Albert believe that at least some of the claims filed are unpatentable as a result of this article. Albert notifies Mary, who reaches the same conclusion. Given these facts, who has a duty to bring the article to the attention of the Office?

A) William and Fran
B) William, Fran, Alex, Mary and Albert
C) Mary and Albert
D) Alex and Albert
E) Mary, Albert and Alex

A

C) Mary and Albert

Reasoning: At a basic level, this question is about the duty of disclosure under Rule 56, but notice that the question does not ask who has a duty to disclose under Rule 56. This question specifically asks who has a duty to bring this particular reference to the attention of the patent examiner.

The rule is that everyone substantively involved in the prosecution of the patent application owes the Patent Office a duty. Thus, William, Fran, Alex, Mary and Albert each owe a duty to disclose under Rule 56. In this situation, however, William and Fran did not know about the article. Thus, William and Fran are not obligated with respect to this article because there is nothing in these facts to suggest that they knew of the article. Therefore (A) and (B) are incorrect.

The parties who knew of this were Alex (who found the article), Albert who was alerted to the article by Alex, and Mary who was alerted to the article by Albert. This may tempt you to select answer (E) as the correct choice, but that would be a mistake. Rule 56(d) allows those who are not the attorney, agent or inventor to satisfy the duty to disclose by notifying the agent, attorney or inventor. Since Alex notified Albert, he successfully discharged his duty to disclose. Therefore, (E) and (D) are incorrect.

The parties that have a duty to notify the USPTO of this article are Mary and Albert. Thus, (C) is correct.

111
Q

On September 15, 2016, you file an application on behalf of your client. Because you are a new patent practitioner without many clients, you do not have a deposit account and instead, as a matter of routine practice, you pay as you make filings. Ultimately, the examiner mails a final rejection in the application, dated October 1, 2018. The action sets a three-month shortened statutory period for a response. An amendment was filed on February 18, 2019, in response to the final rejection. The amendment included a petition and fee for a two-month extension of time. On March 10, 2019, an Advisory Action was mailed indicating that the amendment had been considered, but was not deemed to have placed the application in condition for allowance. On March 11, 2019, a Notice of Appeal was filed. On April 21, 2019, it was noted that a petition and fee for a third month extension of time had not been filed with the Notice of Appeal. On April 22, 2019, a petition and fee for an extension of time for the third month was filed. On which of the following dates, if any, would the filing of an Appeal Brief be considered timely?
A) May 10, 2019
B) May 11, 2019
C) June 20, 2019
D) June 21, 2019
E) None of the above

A

E) None of the above

Reasoning: The first thing to notice here is that the question explains that you do not have a deposit account. This means that there can be no general authorization in the application on file because the USPTO does not accept a general authorization to charge fees to a credit card. Therefore, if you needed an extension at any time, you needed to file a request for the extension and pay for the necessary extension (i.e., with no general authorization to charge a deposit account, there is no implicit request for extensions of time).

When you filed the Notice of Appeal on March 11, 2019, you needed to also file a request for a three-month extension together with the appropriate fee, which did not happen. The six-month statutory period to take action ran through April 1, 2019. The application went abandoned on April 2, 2019. Therefore, your attempt on April 22, 2019, to pay the fee and request a three-month extension was too late; the application was already abandoned. Clearly, you did not intend the application to go abandoned; thus you would be able to revive this application as having been unintentionally abandoned. But that is not what the questions asks.

112
Q

Raymond Stantz and Egon Spengler are co-inventors of a patented paranormal-activity detection device, which operates by identifying electrostatic changes to the environment not explainable by ordinary earthly phenomena. Together with long-time friend and local celebrity Peter Venkman, the three found a company – Ghost Exterminators ‘R Us – to exploit this new and radical innovation. While some call the detection of paranormal activity a hoax, the company, called Ghost Ex for short, can’t handle all the business.

Ghost Ex is in the process of expanding, training new teams of ghost exterminators, and making plans to launch do-it-yourself home detection kits, which will include disposable paranormal detection devices. Of course, as with all businesses, expansion funds are an issue. Venkman finds an eccentric billionaire, Tonald Drump, who expresses interest in investing in this exciting growth company. During the due diligence phase, it is discovered that a reference (a patent application filed by inventor Ghoul) appearing on the patent search report prepared for Stantz and Spengler by their patent attorney was not disclosed to the Patent Office. This is concerning because the reference was referred to in the opinion letter as one of the four most closely related references located during the search. The patent attorney is consulted and he acknowledges that he probably should have included that reference in an Information Disclosure Statement since it was material and non-cummulative, but he thinks the claims are likely patentably unique over that reference anyway. Drump remains interested in investing, but without a solid patent valuation, will be dramatically impacted. Venkman comes to you for assistance. What, if any, of the following do you advise Ghost Ex to do?

A) File a request for ex parte reexamination since there is some question about whether the claims are patentable in light of the Ghoul application.
B) File a request for a supplemental examination since there is some question about whether the Ghoul application was appropriately withheld.
C) File a reissue application since there is an error in the patent that needs to be corrected.
D) File a petition for post grant review so that the patent examiner can consider the Ghoul application in light of the claims granted in the Ghost Ex patent.
E) None of the above.

A

B) File a request for a supplemental examination since there is some question about whether the Ghoul application was appropriately withheld.

Reasoning: The AIA amended the patent laws to provide that a patent owner may request supplemental examination of a patent to consider, reconsider, or correct information believed to be relevant to the patent. The supplemental examination will determine whether the information presented in the request raises a substantial new question of patentability. If the information presented in the request raises a substantial new question of patentability, the Office will order ex parte reexamination of the patent. This will likely cause some to question why (A) is not correct. While filing a reexamination could address the patentability issue, it would not address any potential inequitable conduct problem. The question explains that the Ghoul reference is material to patentability and non-cumulative, which means it should have been disclosed. Even if the claims are patentable over the Ghoul reference, the claims remain in jeopardy. If inequitable conduct is found, all of the claims to the patent are lost because they will be deemed unenforceable. To fix this problem, you need to pursue a supplemental examination. The entire point of supplemental examination is to submit information to the Office so they can consider whether it matters, while at the same time inoculating a patent from a charge of inequitable conduct, or in the terms of the statute, from being “held unenforceable.” Therefore, (B) is the far superior answer to (A). (C) is incorrect because this type of error cannot be fixed through reissue. (D) is incorrect because a post grant review may not be filed by the patent owner.

113
Q

On March 21, 2018, you file a patent application on behalf of your client, a small company. The application contains 2 claims. Claim 1 is to a cleaning solution wherein the primary ingredient is uric acid. Claim 2 is to a cleaning solution wherein uric acid and lemons are the primary ingredients. You received an Office action dated August 19, 2019, wherein all the claims were rejected. You provide your client with a copy of the Office action. Today, August 27, 2019, the president of the client company calls you and informs you, for the first time, that what makes the cleaning solution so effective and popular with consumers is that they use lemons from citrus trees located only in Seminole County. He further explains the company wants to keep the fact that the lemons come from Seminole County a trade secret. He further informs you that they have tested lemons from all over and for reasons he can’t understand (he is not himself a chemist), there is something, in his words, “peculiarly unique about the cleansing and freshening power of those Seminole lemons.” Other lemons in combination with uric acid work, but not nearly as well. He finally tells you that the company has known of the peculiar and advantageous properties of the Seminole lemons since at least January 2018.

You do believe that the claims as filed can be amended to overcome the examiner’s rejections by adding a limitation that relates to the presence of sulfuric acid in trace quantities, which is disclosed in the originally filed specification. What advice should you give?

A) Advise that the application should be abandoned because the best mode has not been disclosed, which hopelessly compromises the application.
B) Advise that there is no longer a need to disclose the best mode, which means that the amendments you envision can be submitted and a patent obtained.
C) Failure to disclose the best mode is no longer grounds to invalidate claims already obtained, but best mode is still required to be disclosed to satisfy 35 U.S.C. 112(a). Therefore, it would violate your ethical obligations to proceed with this application.
D) Cancel claim 2 and make the amendments to Claim 1 you believe will overcome the examiner’s rejections and ask for reconsideration.
E) Amend claims 1 and 2 to add sulfuric acid in trace quantities to overcome the examiner’s rejections and ask for reconsideration.

A

D) Cancel claim 2 and make the amendments to Claim 1 you believe will overcome the examiner’s rejections and ask for reconsideration.

Reasoning: It is true that, since the passage of the AIA, the lack of best mode is no longer an acceptable reason to find a patent claim to be invalid. It is also true that 35 U.S.C. 112(a) still requires best mode to be disclosed. But best mode is not a concept that exists in a vacuum. 35 U.S.C. 112(a) requires the disclosure of the best mode of the claimed invention. In Claim 1, there is no mention of lemons whatsoever. As such, the fact that, in one embodiment of the invention, the cleaning solution uses lemons, and specifically Seminole County, Florida, lemons, is irrelevant. With respect to Claim 2, however, you have included lemons and you have not disclosed that the preferred lemons come from Seminole County, Florida. We know you have not disclosed this fact in the specification because the question tells us that on the August 27 call your client tells you this for the first time. This also means that you have not engaged in any kind of ethical violation by seeking claims you know you are not entitled to receive, but it also means that you cannot ethically proceed to attempt to obtain Claim 2.

(A) is incorrect because there is no need to abandon the entire application. Claim 1 is still viable.

(B) is incorrect because the best mode still must be disclosed.

(C) is incorrect, although it is close. The first two statements are correct, but it would not be unethical to proceed with Claim 1.

(E) is incorrect because you cannot proceed with Claim 2 now that you know that the Claim should be rejected under 35 U.S.C. 112(a) for failure to disclose the best mode. Although not implicated in the question, notice that, when you do amend, you always must ask for reconsideration.

114
Q

In which of the following situations would it be appropriate for the United States Patent and Trademark Office to accept an international patent application filed pursuant to the Patent Cooperation Treaty as either an International Receiving Office or directly on behalf of the International Bureau in Geneva?
A) All applicants are identified as residents and nationals of Germany and the international application is filed in English.
B) At least one applicant is a resident or national of the United States and the international application is filed in English.
C) All applicants are identified as residents and nationals of the United Kingdom and the international application is filed in English
D) The nationality and residence of at least one applicant is not indicated and the international application is filed in English.
E) All of the above.

A

A) All applicants are identified as residents and nationals of Germany and the international application is filed in English.

Reasoning: This question is tricky. The key word in the question is “accept.” Many of you may pick (B) because 37 CFR 1.412(a) says: “(a) The United States Patent and Trademark Office is a Receiving Office only for applicants who are residents or nationals of the United States of America.” However, 37 CFR 1.421(a) says that, if there is no identification of at least one U.S. applicant, and there is at least one applicant that is either a resident or national in a PCT Contracting State or there is no residence or nationality indicated, the U.S. will forward the application to the International Bureau for processing.

The United States will accept an international application for processing in the U.S. where at least one applicant is a resident or national of the U.S. The U.S. will also accept and then forward the application to the International Bureau for processing in (A), (C) and (D), which makes (E) the correct answer.

115
Q

Rajesh Ramayan Koothrappali works in the Physics Department at Caltech, where he specializes in particle astrophysics. One day, while attempting to use a device that emits electromagnetic radiation through a process of optical amplification based on the stimulated emission of photons, Koothrappali identified a temporal anomaly within the polarized wave at a particular frequency. He conceives of a method that reduces distortion induced by the introduction of “watermarks” into signals by exploiting this anomaly. In the context of signal processing, watermarking is a technique by which an original is manipulated so as to embed within it additional data. By employing the Koothrappali method, the additional data would be imperceptible to anyone who intercepts the signal, making it indistinguishable between the watermarked and unwatermarked versions of the signal without a properly calibrated decoder ring.

Koothrappali prepares and files a patent application fully describing his invention. He includes claims to the process he invented, a device that performs that process, a storage medium holding the resulting signals and the altered signals themselves. With respect to claims directed to the signals, which of the following is the most likely outcome?

A) The patent examiner will reject the claims under 35 U.S.C. 101 because what is described is not patent-eligible subject matter.
B) The patent examiner will not reject the claimed invention under 35 U.S.C. 101 because the Koothrappali signals are patent eligible since they have been transformed from their natural state by the addition of imperceptible embedded data.
C) Assuming the claimed invention is new and non-obvious, the patent examiner will issue claims narrowly tailored to the Koothrappali signals.
D) The patent examiner will issue claims, provided that the invention is deemed useful, new, non-obvious, and the disclosure enables the invention and otherwise satisfies the written description requirement.
E) The patent examiner will issue claims, provided that Koothrappali provides a working prototype that can be tested and verified to work by the National Institute of Standards and Technology (NIST).

A

A) The patent examiner will reject the claims under 35 U.S.C. 101 because what is described is not patent-eligible subject matter.

Reasoning: This question is based on the fact pattern of In re Nuijten. In that case, the United States Court of Appeals for the Federal Circuit determined that transitory signals are not patent-eligible subject matter. The Federal Circuit concluded: “A transitory, propagating signal like Nuijten’s is not a ‘process, machine, manufacture, or composition of matter.’ Those four categories define the explicit scope and reach of subject matter patentable under 35 U.S.C. § 101; thus, such a signal cannot be patentable subject matter.” Nuijten was, however, granted patent claims covering the process, the device that performed the process, and a storage medium holding resulting signals.

(B)is incorrect because, despite any particular transformation that may be present, the Federal Circuit has announced a bright line rule associated with transitory signals because they are not one of the four categories of statutory subject matter envisioned within 35 U.S.C. 101.

116
Q

Applicant receives an international search report transmitted on August 7, 2018, relative to an international application filed on April 7, 2017, with the U.S. Receiving Office, and which did not claim priority from any previously filed patent application. The applicants plan to pursue PCT Chap 2, International Preliminary Examination. In this Chapter 2 scenario, what is the maximum amount of time the applicant would have within which to amend the application and what may be amended?

A) August 7, 2018, and only the claims may be amended pursuant to Article 19
B) October 7, 2018, and only the claims may be amended pursuant to Article 19
C) November 7, 2018, and the specification, claims and drawings may be amended pursuant to Article 34
D) February 7, 2019, and the specification, claims and drawings may be amended pursuant to Article 34
E) February 7, 2019, and only the claims may be amended pursuant to Article 19

A

D) February 7, 2019, and the specification, claims and drawings may be amended pursuant to Article 34

Reasoning: Article 34 of the PCT relates to procedures before the International Preliminary Examining Authority, and grants the applicant the right to amend the claims, the description, and the drawings before the international preliminary examination report is established. The amendment made pursuant to Article 34 must not go beyond the disclosure in the international application as filed.

Examination by the International Preliminary Examining Authority will start either by the expiration of the 22nd month from the priority date, or three months from the date of transmittal of the international search report, whichever is later. In this case, three months from the transmittal of the international search report is November 7, 2018, and 22 months from the priority date is February 7, 2019. Therefore, the later is February 7, 2019. Thus, amendments made pursuant to Article 34 of the PCT must be made by February 7, 2019.

Amendments may, however, also be made under Article 19 of the PCT. Article 19 deals with amending the claims before the International Bureau. The opportunity to make amendments under Article 19 is available after the applicant has received the international search report and the written opinion of the International Search Authority, and remains available until the end of 16 months from the priority date or two months after the date the report and opinion are mailed, whichever expires later. In this question, the search report was transmitted on August 7, 2018, so two months later would be October 7, 2018. 16 months from the priority date would be August 7, 2018. Thus, October 7, 2018, is later.

Thus, it would be possible to amend under Article 19 only until October 7, 2018, but possible to amend under Article 34 until February 7, 2019. Thus, (A), (B), (C) and (E) are incorrect.

117
Q

You filed a patent application at the USPTO on behalf of your client on an invention that is not subject to a secrecy order. At the time the application is filed, a non-publication request is also filed. Eight months after the application is filed, your client informs you that she wants to seek foreign patent protection for her invention. Your client is a citizen of the United States and resides in Wyoming. Which of the following, if any, is most appropriate for you to do?

A) Prepare a PCT patent application claiming priority to the earlier filed U.S. patent application and file it with the USTPO.
B) File a petition with the Commissioner for Patents seeking a license to file a foreign patent application.
C) File a petition for a retroactive foreign filing license.
D) Prepare a PCT patent application claiming priority to the earlier filed U.S. patent application, file it with the USPTO, and simultaneously file an appropriate rescission of the previous nonpublication request.
E) A nonpublication request cannot be rescinded and, therefore, foreign patent rights have been waived.

A

D) Prepare a PCT patent application claiming priority to the earlier filed U.S. patent application, file it with the USPTO, and simultaneously file an appropriate rescission of the previous nonpublication request.

Reasoning: A nonpublication request must be made, if at all, at the time of filing an application. By filing a nonpublication request, the applicant is waiving the right to seek foreign patent protection in another country or under a multilateral international agreement that requires publication of applications at 18 months. The application may, however, rescind the previously filed nonpublication request at any time. In order to rescind the nonpublication request, the applicant must both rescind the nonpublication request and provide notice of foreign filing. Specifically, the applicant must, not later than 45 days after the date of filing of a foreign application or international application, notify the USPTO of such filing. Failure to provide the required notice will result in the U.S. application being regarded as abandoned, unless it is deemed that the delay in submitting the notice was unintentional. Therefore (D) is correct and is more complete than (A).

(B) and (C) are incorrect because a license to file is not implicated in the question. The invention is not subject to a secrecy order, and therefore can be filed overseas.

(E) is incorrect because an applicant can rescind a nonpublication request at any time.

118
Q

On January 4, 2013, the applicant files a patent application at the USPTO. The filing contains the appropriate filing fee. Informal copies of the drawings were submitted with the application papers. However, due to an oversight, Figures 10, 11 and 12 were missing. The executed oath was also not included with the application when it was filed. On March 1, 2013, the USPTO notified the applicant that Figures 1 through 9 were deemed unacceptable and replacement drawings were required. The Office further provided a Notice of Missing Parts, relating to the failure to provide an executed oath, and a Notice of Omitted Items, which relates to the three missing Figures. On April 2, 2013, the applicant filed formal replacement drawings for Figures 1 through 9. The executed oath was filed with the Office on April 14, 2013. The applicant filed a preliminary amendment on May 3, 2013, which provided formal drawings for Figures 10, 11 and 12. Given only these facts, what filing date will be accorded to the patent application?

A) January 4, 2013
B) March 1, 2013
C) April 2, 2013
D) April 14, 2013
E) May 3, 2013

A

E) May 3, 2013

Reasoning: The USPTO no longer makes a distinction between formal drawings and informal drawings. The only distinction made is between drawings that are acceptable or drawings that are unacceptable. If the original drawings are not acceptable, the applicant will be notified and informed of the objections and that new corrected drawings are required. Notwithstanding, even drawings deemed unacceptable will be accepted for the purpose of satisfying the requirements for obtaining a filing date. Therefore, the filing date will be the date that at least a specification, at least one claim and one drawing (if necessary) arrived at the Patent Office. That date is January 4, 2013, which makes (A) the correct answer. NOTICE: A specification, at least one claim and at least one drawing (if necessary) are required based on this filing date. Under the Patent Law Treaties Implementation Act (PLTIA), on or after December 18, 2013, only a specification needs to be present in order to obtain a filing date.

119
Q

Which of the following would favor patent eligibility for a claimed computer-implemented method or process?
A) The involvement of a machine with the claimed steps is tangentially related to the performance of the steps.
B) The machine is merely an object on which the method operates.
C) The claim describes a particular solution to a specific problem to be solved.
D) Both known and unknown uses can be performed without any apparatus.
E) A machine is generically recited in a claim such that it would cover any machine capable of performing the claim steps.

A

C) The claim describes a particular solution to a specific problem to be solved.

120
Q

Which of the following examples of claims would not be considered to be directed to statutorily eligible categories of invention?

I. Transitory forms of signal transmission

II. A computer program per se

III. A human per se

IV. A mere arrangement of printed matter

A)
I.

B)
I., III.

C)
I., III., and IV.

D)
I., II., III., and IV.

E)
III. and IV.

A

D)
I., II., III., and IV.

Reasoning: This will present as a tricky question for some, which is why it is here. Some no doubt ruled out the correct answer because at least some software is patent eligible with software patents being issued by the USPTO every week. Thus, how could a computer program be considered to be directed to a statutorily ineligible category of invention? That is a very good question for which there probably is not going to be a satisfactory answer other than to say this is what the MPEP specifically says. The MPEP, which cites Gottschalk v. Benson for the proposition that a computer program per se is not patent eligible, thus (II) is not statutorily eligible. If you read Gottschalk the decision is quite different and it is probably a stretch to cite Gottschalk for this proposition. Such a proposition is even more difficult to reconcile with Diamond v. Diehr and the fact that even in Alice the Supreme Court refused to rule software patent ineligible. Do not get caught up in this! Just accept that the MPEP must be splitting some hair that is not particularly well explained. Your mission is to pass the exam! The MPEP clearly and unambiguously says that computer programs per se do not fall within the statutorily eligible categories of invention. That means that claims directed to computer programs per se (whatever that means) would be patent ineligible under 35 U.S.C. 101. Given the newness and uncertainty, as well as the fact that the Office will not test any of the difficult Federal Circuit decisions of the last several years, you can anticipate if you get a question like this it will very closely track the MPEP language. Thus, know that computer programs per se (whatever that means) are not a statutorily eligible category of invention.

(I) is not statutorily eligible as the result of In re Nuijten. (III) is not statutorily eligible as the result of the America Invents Act (AIA), which specifically made human organisms patent ineligible. There is no definition of the term human organisms in the AIA, so a question will either be related to humans per se, which the MPEP says is not patent eligible, or specifically to human organisms. (IV) is not statutorily eligible as the result of In re Miller, an old CCPA decision.

121
Q

Which of the following statements is not true with respect to determining patent eligibility of a claim?

A)
The subject matter courts have found to be outside of the statutory categories of invention are limited to abstract ideas, laws of nature and natural phenomena.

B)
In order to transform a patent-ineligible law of nature into a patent-eligible invention, the claim must incorporate applications of those laws.

C)
The Supreme Court’s concern that drives its jurisprudence relating to judicial exceptions to patent eligibility is pre-emption.

D)
Abstract ideas, laws of nature, and natural phenomenon are the basic tools of scientific and technological work. Thus, the courts have expressed concern that monopolizing these tools by granting patent rights may impede innovation rather than promote it.

E)
The courts have held that an application of an abstract idea, law of nature or natural phenomenon to a new and useful end is eligible for patent protection.

A

B)
In order to transform a patent-ineligible law of nature into a patent-eligible invention, the claim must incorporate applications of those laws.

Reasoning: In Mayo v. Prometheus, the Supreme Court explained that one must do more than simply state the law of nature while adding the words “apply it” (citing Gottschalk v. Benson). (A) is not the correct choice because it is a true statement. At least for the time being, abstract ideas, laws of nature and natural phenomena are the only judicially recognized exceptions to patent eligibility (See MPEP 2103). Courts will from time to time use additional characterizations (i.e., mental processes, disembodied concepts, disembodied mathematical algorithms) but they will related to one of the three judicially recognized exceptions.

(C) is not the correct choice because it is a true statement. The Supreme Court continues to explain in case after case that it is primarily concerned with pre-emption.

(D) is not the correct choice because it is a true statement. The Supreme Court views abstract ideas, laws of nature and natural phenomena as basic scientific building blocks and fears that granting exclusive rights (or, as they say, “monopoly rights”) will inhibit rather than promote innovation.

(E) is not the correct choice because it is a true statement. The fact that an invention relies upon, leverages or incorporates an abstract idea, law of nature or natural phenomena does not as a matter of fact (or law) mean that the claimed invention is patent-ineligible. This makes sense; otherwise, nothing would be patent-eligible, given that every invention necessarily starts with an idea.

122
Q

Application A was filed after November 29, 2016. Reference X and application A were commonly owned at the time the invention of application A was made, and later filed. In accordance with the patent laws, rules and procedures as related in the MPEP the prior art exclusion of 35 USC 102(c) can be properly invoked to obviate which of the following rejections?

A) A rejection under 35 USC 102 based on reference X, if reference X is prior art under 35 USC 102(a)(1).
B) A double patenting rejection based on reference X, if reference X is available as prior art under 35 USC 102.
C) A rejection under 35 USC 103 based on reference X, if reference X is available as prior art only under 35 USC 102(a)(2).
D) (B) and (C).
E) All of the above.

A

C) A rejection under 35 USC 103 based on reference X, if reference X is available as prior art only under 35 USC 102(a)(2).

Reasoning: (A) is wrong because the 102(c) exception does not apply to 102(a)(1) prior art.

(B) is wrong because the 102(c) exception does not apply to either obviousness-type or same-invention double patenting.

(D) and (E) are wrong because of the above.

123
Q

A utility application filed in May 2016 has been prosecuted through a second action final rejection. In the final rejection some claims were allowed and other claims were finally rejected. Which of the following accords with the patent laws, rules and the procedures as related in the MPEP for a proper reply to a second action final rejection in the utility application?

A) An amendment canceling all rejected claims and complying with 37 CFR 1.116.
B) Only a Notice of Appeal.
C) The appropriate fee for a request for continued examination (RCE).
D) A continued prosecution application (CPA) under 37 CFR 1.53(d).
E) All of the above.

A

A) An amendment canceling all rejected claims and complying with 37 CFR 1.116.

Reasoning: I know you probably thought all were correct because you read A and B and thought they were both correct, but B is incorrect because an appeal fee is due as well.

The filing of an amendment complying with 37 CFR § 1.116 is a proper reply under 37 CFR § 1.113 to a final rejection. See MPEP § 714.13, under the heading “Entry Not A Matter of Right,” which states, in pertinent part , “A reply under 37 CFR 1.113 is limited to: (A) an amendment complying with 37 CFR 1.116.” (B) is not the most correct answer because the Notice of Appeal must be accompanied by the appeal fee required by 37 CFR § 1.17(b). (C) is not the most correct answer because the RCE must be accompanied by a submission (i.e., an amendment that meets the reply requirement of 37 CFR § 1.111). (D) is not the correct answer because CPA practice does not apply to utility or plant applications if the prior application has a filing date on or after May 29, 2000. See MPEP § 706.07(h), paragraphs I and IV. (E) is not the correct answer since (A) is a proper reply.

124
Q

Inventor files an application containing the following original Claim 1:

  1. A widget comprising element A, and element B.

In a first Office action on the merits, a primary examiner rejects claim 1 under 35 USC 103 as being obvious over reference X. Reference X explicitly discloses a widget having element A, but it does not disclose element B. The examiner, however, takes official notice of the fact that element B is commonly associated with element A in the art and on that basis concludes that it would have been obvious to provide element B in the reference X widget. In reply to the Office action, the registered practitioner representing the applicant makes no amendments, but instead requests reconsideration of the rejection by demanding that examiner show proof that element B is commonly associated with element A in the art. Which of the following actions, if taken by the examiner in the next Office action would be in accord with the patent laws, rules and procedures as related in the MPEP?

I. Vacate the rejection and allow the claim.

II. Cite a reference that teaches element B is commonly associated with element A in the art and make the rejection final.

III. Deny entry of applicant’s request for reconsideration on the ground that it is not responsive to the rejection and allow applicant time to submit a responsive amendment.
A) I and II only.
B) II only.
C) II and III only.
D) I, II, and III.
E) I and III only.

A

A) I and II only.

Reasoning: MPEP § 2144.03 provides that when an applicant seasonably traverses an officially noticed fact, the examiner may cite a reference teaching the noticed fact and make the next action final. Here, applicant did seasonably traverse the noticed fact by demanding proof in response to the rejection. II is therefore an appropriate action by the examiner. I is also an appropriate action because the examiner should vacate a rejection based on official notice if no support for the noticed fact can be found in response to a challenge by the applicant. See In re Ahlert, 424 F.2d 1088, 1091 (C.C.P.A. 1970) (“[a]ssertions of technical facts in areas of esoteric technology must always be supported by citation to some reference work” and “[a]llegations concerning specific “knowledge” of the prior art, which might be peculiar to a particular art should also be supported”). (B) is incorrect because (A) is correct. (C), (D), and (E) are incorrect because action III is improper. An applicant is entitled to respond to a rejection by requesting reconsideration, with or without amending the application. 37 CFR § 1.111(a)(1). Applicant is also required to timely challenge a noticed fact in order to preserve the issue for appeal. MPEP § 2144.03.

125
Q

A registered practitioner filed an application for an applicant claiming a “a means for pulling the door open.” The specification describes a handle and a knob as being used together as a corresponding structure for pulling the door open. A prior art patent discloses a door opened by pulling on an attached bar. The primary examiner issued an Office action rejecting the claim under 35 USC 102 as being anticipated. In the action, the examiner properly identified the corresponding structure described in applicant’s specification as the means for pulling the door open, and properly explained why the prior art attached bar is the equivalent of the structure described in applicant’s specification. In accordance with the patent laws, rules and procedures as related in the MPEP, which of the following is the most correct reply to overcome the rejection under these circumstances?

A) An amendment to the claim changing the pulling means to expressly include an attached bar.
B) Only argue that the claimed pulling means is not found in the prior art relied-upon reference and therefore the claim is patentable.
C) An amendment to the specification that adds an attached bar to correspond to the prior art.
D) An amendment to the claim substituting for the term “means for pulling the door open” the structure of a handle and a knob.
E) An amendment to the specification that excludes an attached bar as a pulling means.

A

D) An amendment to the claim substituting for the term “means for pulling the door open” the structure of a handle and a knob.

Reasoning: ANSWER: (D) is the most correct answer. MPEP § 2181 under the heading “Procedures For Determining Whether The Written Description Adequately Describes The Corresponding Structure, Material, Or Acts Necessary To Support A Claim Limitation Which Invokes 35 U.S.C. 112(f).” 35 U.S.C. 112(f) states that a claim limitation expressed in means plus function language “shall be construed to cover the corresponding structure, materials, or acts described in the specification and ‘equivalents thereof.’” See also B. Braun Medical, Inc. v. Abbott Lab., 124 F.3d 1419, 1424, 43 USPQ2d 1896, 1899 (Fed. Cir. 1997).” The examiner has made a prima facie case of equivalent in the Office action to support the rejection based on 35 U.S.C. § 102. By amending the claim to no longer include the means limitation in question, the claim becomes narrower inasmuch as it no longer includes equivalents under 35 U.S.C. § 112(f) for examination purposes. Thus, (D) overcomes the lack of novelty rejection under these circumstances. (A) is not the most correct answer because such an amended claim would continue to lack novelty, since both it and the prior art would have the attached bar expressly. Furthermore, such an amendment would introduce new matter lacking support in the application as originally filed. 35 U.S.C. § 112(a). (B) is not the most correct answer because the “not found in the prior art” argument does not rebut the prima facie case of equivalents raised by the examiner. (C) is not the most correct answer because it does not address the rejection. (E) is not the most correct answer because the amendment would raise a new matter issue.

126
Q

When, in accordance with the patent laws, rules and procedures as related in the MPEP, is a supplemental oath or declaration treated as an amendment under 37 CFR 1.312?

A) When filed in a nonprovisional application after the Notice of Allowance has been mailed.
B) When filed in a reissue application at any point during the prosecution.
C) When filed in a nonprovisional application after the payment of the Issue Fee.
D) When filed in a reissue application after the Notice of Allowance has been mailed.
E) (A) and (D).

A

D) When filed in a reissue application after the Notice of Allowance has been mailed.

Reasoning: You missed this because you found it in the MPEP but didn’t look at the header. Know the context of the MPEP part you are reading! This was in 714.16, which is titled “Amendment After Notice of Allowance”. It says “With the exception of a supplemental oath or
declaration submitted in a reissue, a supplemental oath or declaration is not treated as an amendment under 37 CFR 1.312”. You read this and didn’t read the heading.

127
Q

A registered practitioner files a patent application with the following claim: 1. A plastic insert for the bottom of a shopping cart comprising circular receptacles to receive wine bottles and to maintain them in an upright and stable position even while the shopping cart is moved about a store so that they do not fall and break. Patent A discloses a plastic insert for the bottom of a shopping cart comprising rectangular receptacles to receive cereal boxes and to maintain them in an upright and stable position even while the shopping cart is moved about a store in order to keep them organized in the cart. Patent A also discloses that the receptacles could be any circular diameter to receive complementary shaped bottles or jars such as to securely hold 2-liter soft drink bottles or mayonnaise jars. A primary examiner rejected the claim as being obvious under 35 USC 103 over Patent A reasoning that Patent A suggests to one of ordinary skill in the art an insert for a shopping cart with circular receptacles for the purpose of stably maintaining any bottle, including wine bottles, while pushing the cart about a store so that the cart remains organized. Assume the examiner has made a sufficient prima facie case of obviousness. Following receipt of the rejection, the practitioner filed a timely reply. The practitioner argued that Patent A does not render obvious the claimed subject matter because there is no suggestion of a plastic insert to keep a wine bottle from falling and breaking in a shopping cart. Which of the following best explains why, in accordance with the patent laws, rules and the procedures as related in the MPEP, the examiner should or should not be persuaded by the practitioner’s argument?

A) No, because Patent A suggests circular receptacles for any complementary bottle, albeit for a different purpose.
B) Yes, because there is no suggestion in Patent A that the plastic insert can hold a wine bottle.
C) Yes, because the claim uses the insert to keep the bottles from falling and breaking while Patent A uses the insert to keep the cart organized.
D) Yes, because Patent A is more interested in organizing boxes than holding bottles.
E) Yes, because the prevention from breakage is an unexpected property of the plastic insert.

A

A) No, because Patent A suggests circular receptacles for any complementary bottle, albeit for a different purpose.

Reasoning: MPEP § 2144, under the heading “Rationale Different From Applicant’s Is Permissible.” Patent A suggests an insert with receptacles that are circular and which can be shaped to complement the shape of the object to be received. The purpose for this in Patent A is to keep the cart organized, not as in the claim to prevent the object from falling and breaking. The difference in objectives does not defeat the case for obviousness because, as MPEP § 2144 states, the “reason or motivation to modify the reference may often suggest what the inventor has done, but for a different purpose or to solve a different problem. It is not necessary that the prior art suggest the combination to achieve the same advantage or result discovered by applicant. In re Linter, 458 F.2d 1013, 173 USPQ 560 (CCPA 1972) …; In re Dillon, 919 F.2d 688, 16 USPQ2d 1897 (Fed. Cir. 1990), cert. denied, 500 U.S. 904 (1991) … .” In other words, it does not matter that Patent A does not appreciate the claimed purpose of preventing breakage. It suggests an insert with receptacles to hold bottles. That is enough to render the claimed subject matter prima facie obvious. The prima facie case is not rebutted by arguing that the purpose for the claimed insert is different form that specified for the insert described in Patent A. That is why answer (C) is wrong. To rebut the prima facie case, the practitioner must show a difference in structure instead. Answer (B) is wrong because the prima facie case is not rebutted by showing that Patent A does not teach wine bottles. This is not an anticipation rejection where identity of subject matter might be an issue. This is a question of obviousness. Therefore, it is sufficient to point out that Patent A is a generic teaching of shopping cart inserts that hold objects of any size and shape. (D) is not the most correct answer because what Patent A is interested in doing is irrelevant to the question of obviousness. (E) is not the most correct answer inasmuch as it was not the practitioner’s argument. However, the question inquires about the merits of the argument that the practitioner made as set forth in the penultimate sentence of the question, not the merits of some hypothetical reply the examiner may communicate.

128
Q

A registered practitioner filed a utility application on February 11, 2016. On April 4, 2016, the practitioner filed an information disclosure statement (IDS) in the application. The practitioner received a notice of allowance dated January 3, 2017 soon after it was mailed. When discussing the application with the practitioner on January 21, 2017, and before paying the issue fee, the client notices for the first time that a reference, which is one of many patents obtained by the client’s competitor, was inadvertently omitted from the IDS. The client has been aware of this reference since before the application was filed. The client is anxious to have this reference appear on the face of the patent as having been considered by the USPTO. Which of the following actions, if taken by the practitioner, would not be in accord with the patent law, rules and procedures as related by the MPEP?
A) Before paying the issue fee, timely file an IDS citing the reference, along with the certification specified in 37 CFR 1.97(e), and any necessary fees.
B) Within three months of the mail date of the notice of allowance, without paying the issue fee, timely file a Request for Continued Examination (RCE) under 37 CFR 1.114, accompanied by the fee for filing an RCE, and an IDS citing the reference.
C) Within three months of the mail date of the notice of allowance, without paying the issue fee, timely file a continuing application under 37 CFR 1.53(b), an IDS citing the reference, and any necessary fees.
D) After paying the issue fee, timely file a petition to withdraw the application from issue to permit the express abandonment of the application in favor of a continuing application, a continuation application under 37 CFR 1.53(b), an IDS citing the reference, and any necessary fees.
E) After paying the issue fee, timely file a petition to withdraw the application from issue to permit consideration of a Request for Continued Examination (RCE) under 37 CFR 1.114, the fee for filing an RCE, and an IDS citing the reference.

A

A) Before paying the issue fee, timely file an IDS citing the reference, along with the certification specified in 37 CFR 1.97(e), and any necessary fees.

Reasoning: (A), describing a procedure that is not in accordance with the USPTO rules and the procedures set forth in the MPEP, is the most correct answer. MPEP § 609, under the heading “Minimum Requirements for an Information Disclosure Statement,” under the subheading “B(3). Information Disclosure Statement Filed After B(2), but Prior to Payment of Issue Fee 37 CFR 1.97 (d)”, and subheading “B(5) Statement Under 37 CFR 1.97(e).” (A) The statement specified in 37 CFR § 1.97(e) requires that the practitioner certify, after reasonable inquiry, that no item of information contained in the IDS was known to any individual designated in 37 CFR § 1.56(c) more than three months prior to the filing of the information disclosure statement. The practitioner cannot certify this because the reference was known to the client before February 11, 2016, the time of filing of the utility application, which was more than three months prior to the filing of the information disclosure statement.

(B), stating a procedure that conforms with the USPTO rules and the procedures set forth in the MPEP, is an incorrect answer. Under 37 CFR § 1.313(a), a petition to withdraw the application from issue is not required if a proper RCE is filed before payment of the issue fee.

(C), stating a procedure that conforms with the USPTO rules and the procedures set forth in the MPEP, is an incorrect answer. A practitioner can file a continuing application on or before the date that the issue fee is due and permit the parent application to become abandoned for failure to pay the issue fee.

(D), stating a procedure that conforms with the USPTO rules and the procedures set forth in the MPEP, is an incorrect answer. Under 37 CFR § 1.313(c)(3), a petition to withdraw the application from issue can be filed after payment of the issue fee to permit the express abandonment of the application in favor of a continuing application.

(E), stating a procedure that conforms with the USPTO rules and the procedures set forth in the MPEP, is an incorrect answer. Under 37 CFR § 1.313(c)(2), a petition to withdraw the application from issue can be filed after payment of the issue fee to permit consideration of a Request for Continued Examination (RCE) under 37 CFR § 1.114. See also MPEP § 1308.

129
Q

A registered practitioner files an international application submission that includes a description, claims and drawings in the United States Receiving Office (RO/US) on Wednesday, January 8, 2003. The submission did not include the required request, international and search fees, or the designation of a PCT contracting State. The RO/US mails an “Invitation to Correct the Purported International Application,” dated January 10, 2003, to the practitioner indicating that the designation of at least one Contracting State, as required by PCT Article 11(1)(iii)(b), was not included. A one-month period for response is set in the Invitation. On Monday, February 10, 2003, the practitioner submits by facsimile a designation sheet of the Request Form designating every available Contracting State, and authorization to charge all required fees. In accordance with the patent laws, rules and procedures as related in the MPEP, will the application be accorded an international filing date?

A) Yes. The application will be accorded a filing date of January 8, 2003.
B) Yes. The application will be accorded an international filing date of February 10, 2003.
C) No. The application will not be accorded an international filing date because the failure to designate at least one contracting State cannot be cured by a facsimile transmission.
D) No. The application was given a one-month period for response. The practitioner would have had to have filed the response on Friday, February 7, 2003 in order to have been timely.
E) None of the above.

A

C) No. The application will not be accorded an international filing date because the failure to designate at least one contracting State cannot be cured by a facsimile transmission.

Reasoning: 37 CFR §§ 1.6(d)(3) and 1.8(a)(2)(i)(d); MPEP § 502 (reproducing Rule 1.6(d)(3)); MPEP § 512 (reproducing Rule 1.8(a)(2)(i)(d)); and MPEP §1817.01. As stated in MPEP § 1817.01, “[a]ll designations must be made in the international application on filing; none may be added later.” The application will not be accorded an international filing date since the practitioner has tried to cure the failure to designate at least one contracting State by filing a paper using facsimile which is not permitted according to 37 CFR §§ 1.6(d)(3) and 1.8(a)(2)(i)(d).

Tip: Read the remaining answers. I didn’t know about facsimile, and if I read the answer C, I might have checked

130
Q

On January 2, 2016, a registered practitioner filed a patent application with the USPTO for inventor Beck. The application includes a specification and a single claim to the invention which reads as follows:

  1. Mixture Y made by the process Q1.

In the specification, Mr. Beck discloses that mixture Y has a melting point of 150 degrees F. On June 2, 2016, the practitioner received an Office action from the primary examiner rejecting the claim. The claim is rejected under 35 USC 102/103 as being clearly anticipated by or obvious over Patent A. The examiner states “Patent A teaches mixture Y but made by a different process Q2.” Beck believes he is entitled to a patent to mixture Y. In accordance with the patent laws, rules and procedures as related in the MPEP, which of the following would be the best reply to the rejection of his claim?
A) An argument that the claimed product has an unexpectedly low melting point of 150 degrees F, supported by an affidavit showing that the mixture Y made by process Q2 exhibits a melting point of 300 degrees F.
B) An argument that the processes used by applicant and patent A are different, supported by a third-party declaration stating only that the processes are different.
C) An argument that the claimed product has an unexpectedly low melting point of 150 degrees F, supported by a third-party declaration stating only that the products are different.
D) An argument that the processes used by applicant and patent A are different, supported by an affidavit showing that the mixture Y made by process Q2 exhibits a melting point of 300 degrees F.
E) An argument that the claimed product has an unexpectedly low melting point of 150 degrees F because the claimed mixture Y has a melting point of 150 degrees F and the mixture Y of patent A has a melting point of 300 degrees F.

A

A) An argument that the claimed product has an unexpectedly low melting point of 150 degrees F, supported by an affidavit showing that the mixture Y made by process Q2 exhibits a melting point of 300 degrees F.

Reasoning: You have to show a difference in products, not processes. You need to back this up with evidence. You originally selected A, but changed the answer because You thought 300 came from nowhere. That is okay.

131
Q

Which of the following statements is or are in accord with the patent laws, rules and procedures as related in the MPEP?

(1) In a 35 USC 103 obviousness analysis, the proper question is whether the differences between the prior art and the claims would have been obvious to one of ordinary skill in the art.

(2) In a 35 USC 103 obviousness analysis, an inventor’s assertion the he has discovered the source or cause of an identified problem should never be considered.

(3) A 35 USC 103 obviousness analysis requires consideration not just of what is literally recited in the claims, but also of any properties inherent in the claimed subject matter that are disclosed in the specification.
A) Statement 1
B) Statement 2
C) Statement 3
D) Statements 1 and 2
E) Statements 1 and 3

A

C) Statement 3

Reasoning: The principle in Statement 3, that consideration of inherent properties is part of proper consideration of the invention as a whole, is recited in MPEP § 2141.02, under the heading “Disclosed Inherent Properties Are Part Of ‘As A Whole’ Inquiry,” and in In re Antonie, 559 F.2d 618, 620, 195 USPQ 6, 8 (CCPA 1977). (A) is incorrect, because the proper question is whether the invention as a whole, not just the differences, would have been obvious. See MPEP § 2141.02, under the heading “The Claimed Invention As A Whole Must Be Considered,” (citing Stratoflex, Inc. v. Aeroquip Corp., 713 F.2d 1530, 218 USPQ 871 (Fed. Cir. 1983). (B) is incorrect because an examiner should consider such assertions by an inventor as part of the “subject matter as a whole.” See MPEP § 2141.02 (citing In re Sponnoble, 405 F.2d 578, 585, 160 USPQ 237, 243 (CCPA 1969)). (D) and (E) are incorrect because they include incorrect Statements 1 and/or 2.

132
Q

Which of the following would comply with the patent laws, rules and procedures as related in the MPEP and would be a fully responsive reply to a non-final Office action on the merits rejecting all the claims in the application as being unpatentable under 35 USC 102 and/or 103 over prior art references?
A) A timely filed and properly signed written reply which does not include an amendment to the claims, but includes a request for the examiner’s rejections to be reconsidered supported by arguments replying to every ground of rejection and distinctly and specifically points out the supposed errors in every rejection. and pointing out the specific distinctions believed to render the claims patentable over any applied references.
B) A timely filed and properly signed written reply which includes an amendment canceling all the claims in the application and adding new claims, and a request for the examiner’s rejections to be reconsidered in view of the newly presented claims.
C) A timely filed and properly signed written reply which does not include an amendment to the claims, but does generally alleges that the claims define a patentable invention.
D) A timely filed and properly signed written request for continued examination (RCE).
E) All of the above.

A

A) A timely filed and properly signed written reply which does not include an amendment to the claims, but includes a request for the examiner’s rejections to be reconsidered supported by arguments replying to every ground of rejection and distinctly and specifically points out the supposed errors in every rejection. and pointing out the specific distinctions believed to render the claims patentable over any applied references.

Reasoning: A is the only one fully complete. B and C don’t address all the rejections and an RCE can only be filed after prosecution is closed.

133
Q

Applicant filed a provisional patent application in the USPTO under 35 USC 111(b) on Tuesday, November 30, 1999. On Tuesday, November 28, 2000, applicant filed a nonprovisional application in the USPTO under 35 USC 111(a) that properly claimed priority under 35 USC 119(e) to the filing date of the provisional application. On Wednesday, November 29, 2000, applicant filed an international application for patent in the USPTO under the Patent Cooperation Treaty that designated the United States and properly claimed priority to both the provisional and the nonprovisional applications. On Friday, July 28, 2001, applicant filed a national stage application in the USPTO under 35 USC 371, providing all of the requirements under 35 USC 371 and properly claiming benefit to the filing date of the provisional application under 35 USC 119(e) and the nonprovisional application under 35 USC 120. The national stage application was published on Tuesday, January 30, 2002 and issued as a patent on Tuesday, February 4, 2003. Assuming no patent term extension or adjustment, the patent term ends on the date that is 20 years from which of the following dates in accordance with the patent laws, rules and procedures as related in the MPEP?

A) Tuesday, November 30, 1999
B) Tuesday, November 28, 2000
C) Wednesday, November 29, 2000
D) Friday, July 28, 2001
E) Tuesday, February 4, 2003

A

B) Tuesday, November 28, 2000

Reasoning: The filing date of the nonprovisional application, (B), is the correct answer. See MPEP § 201.04(b), which states “[t]he [Uruguay Agreement Round Act] provides a mechanism to enable domestic applicants to quickly and inexpensively file provisional applications. Under the provisions of 35 U.S.C. § 119(e) applicants are entitled to claim the benefit of priority in a given application in the United States. The domestic priority period will not count in the measurement of the 20-year patent term. See 35 U.S.C. 154(a)(3). Thus, domestic applicants are placed on the same footing with foreign applicants with respect to the patent term.” A provisional application is filed under 35 U.S.C. § 119(e) and according to 35 U.S.C. § 154(a)(3), such a filing date is not taken into account in determining patent term. Therefore, (A) is incorrect. The fact pattern states that benefit was properly claimed in the international application to both the provisional application and the national application and that the national stage application filed under 35 U.S.C. § 371 claimed benefit to the filing date of the nonprovisional application under 35 U.S.C. § 120. According to 35 U.S.C. § 154(a)(2), where an application contains a reference to an earlier filed application or applications under 35 U.S.C. § 120, 121, or 365(c), the patent term ends 20 years from the date on which the earliest such application was filed; in this fact pattern that date would be (B), the filing date of the nonprovisional application. The filing date of the international application, (C), is not correct in view of 35 U.S.C. § 154(a)(2) since the international application claimed the benefit under 35 U.S.C. § 120 to the filing date of the nonprovisional application.

You found the correct answer in MPEP 2701, but YOU DIDNT READ ALL THE TEXT.

134
Q

Co-inventors Smith and Jones filed an application for a patent on a cell phone, on May 15, 2016. They received a first Office action from a primary examiner rejecting the claims under 35 USC 102(a)(1) over a publication by Bell and Watson, published on April 5, 2016, describing a cell phone having all the same features as is claimed in the patent application. In reply, the co-inventors each submitted a declaration under 37 CFR 1.131 stating that they had actually reduced the invention to practice no later than March 13, 2016. However, the declarations failed to include two claimed features. Neither the particular antenna needed to enable the cell phone could receive transmissions from the local cellular transmitting tower, nor a detachable carrying strap was included in the declarations. As evidence of their prior reduction to practice, Smith and Jones submitted their co-authored journal article. The journal article contained a figure of the cell phone as described in the declarations. That is, the cell phone shown in the figure of the article lacked an antenna and a detachable strap. The article was received by the journal on March 13, 2016, and was published on April 30, 2016. The cell phones shown in the figure in the Bell and Watson publication, and in the Smith and Jones patent application have the particular antenna and a detachable strap. Which of the following actions, if taken by the examiner, would be the most proper in accordance with the patent laws, rules and the procedures as related in the MPEP?
A) The examiner should maintain the rejection of the claims under 35 USC 102(a) and make the rejection final inasmuch as a 1.131 affidavit has no application in an AIA case.
B) The examiner should withdraw the rejection and look for references which have a publication date prior to May 15, 2015.
C) The examiner should withdraw the rejection and notify Smith and Jones that their application is in condition for allowance.
D) The examiner should maintain the rejection, but indicate that the claims would be allowable if Smith and Jones provided an original copy of the figure published in their journal article as factual support for their declarations.
E) The examiner should maintain the rejection and inform Smith and Jones that the declarations are insufficient because they cannot “swear behind” a reference which is a statutory bar.

A

A) The examiner should maintain the rejection of the claims under 35 USC 102(a) and make the rejection final inasmuch as a 1.131 affidavit has no application in an AIA case.

Reasoning: (A) is the correct answer. A 1.131 affidavit, proving a date of invention, has no application vis-a-vis an application being examined under AIA “first to file” law. Accordingly, the examiner should maintain the rejection and make it final.

(B) and (C) are incorrect choices since the evidence of record shows that Smith and Jones are simply unable to overcome the prior art.

(D) is wrong because an original copy of the published figure which shows that Smith and Jones were not in possession of the claimed invention prior to Bell and Watson publication cannot help their case.

(E) is incorrect because prior art under 102(a) is not necessarily a statutory bar if inventors can “pre-date” with a disclosure of their own using a 1.130 affidavit.

135
Q

In accordance with the patent laws, rules and procedures as related in the MPEP , which of the following paper is precluded from receiving the benefit of a certificate of mailing or transmission under 37 CFR 1.8?
A) An amendment, replying to an Office action setting a period for reply, transmitted by mail with a certificate of mailing to the USPTO from a foreign country.
B) An amendment, replying to an Office action setting a period for reply, transmitted by facsimile with a certificate of transmission to the USPTO from a foreign country.
C) An information disclosure statement (IDS) under 37 CFR 1.97 and 1.98 transmitted after the first Office action.
D) A request for continued examination (RCE) under 37 CFR 1.114.
E) An appeal brief.

A

A) An amendment, replying to an Office action setting a period for reply, transmitted by mail with a certificate of mailing to the USPTO from a foreign country.

Reasoning: See MPEP § 512, which states “The Certificate of Mailing procedure does not apply to papers mailed in a foreign country.”

136
Q

The sole claim in an application filed by A and having an effective filing date of June 5, 2016, recites an electrical signal amplifier comprising a plurality of germanium transistors connected together in a particular configuration. The claim is rejected under 35 USC 103 as being obviousness over a primary nonpatent reference publication (Reference P) in view of a secondary nonpatent reference publication (Reference S). Reference P has an effective date of April 3, 2016, and names A and B as the authors. Reference S has an effective date of December 10, 2015, and names C as the sole author. Reference P discloses an electrical signal amplifier including a plurality of silicon transistors connected together in the same configuration as that set forth in the claim. Reference S discloses a signal amplifier employing germanium transistors connected in a configuration different from the claimed configuration. The applicant does not deny that the references render the claimed subject matter prima facie obvious. Which, if any, of the declarations under 37 CFR 1.130 set forth below should be sufficient under the patent laws, rules and procedures as related in the MPEP to overcome the rejection?
A) An uncontradicted declaration by A asserting that the subject matter relied on by the examiner in reference P constitutes A’s sole invention, with the result that Reference P is not available as prior art against the claim.
B) A declaration by A asserting that “the claimed amplifier has satisfied a long-felt need in the art.”
C) A declaration by A and accompanying copies of competitors’ advertisements which conclusively show that those competitors have exactly copied appellant’s commercial embodiment of the claimed amplifier.
D) A declaration by A and supporting documentation establishing that ever since the filing date of A’s application, sales of the commercial embodiment of A’s claimed amplifier have consistently constituted ninety percent or more of the relevant signal amplifier market in the United States.
E) None of the above.

A

A) An uncontradicted declaration by A asserting that the subject matter relied on by the examiner in reference P constitutes A’s sole invention, with the result that Reference P is not available as prior art against the claim.

Reasoning: See MPEP § 717.01 (III), which states “Under certain circumstances an affidavit or declaration may be submitted which attempts to attribute an activity, a reference or part of a reference to the applicant. If successful, the activity or the reference is no longer applicable. . . . An uncontradicted ‘unequivocal statement’ from the applicant regarding the subject matter disclosed in an article, patent, or published application will be accepted as establishing inventorship.

137
Q

Following a restriction requirement and election, a registered practitioner received a first Office action dated Friday, December 1, 2016. The primary examiner indicated that claims 1 to 10 were rejected and claims 11 to 20 were withdrawn from consideration. The first Office action set a 3 month shortened statutory period for reply. On February 28, 2017, the practitioner properly filed an express abandonment in the application and at the same time filed a request for continuing application. In a non-final Office action dated May 1, 2017 in the continuing application, the examiner indicated in that claims 1 to 20, all of the pending claims, are rejected. The practitioner filed a notice of appeal on Monday, July 2, 2017. In accordance with USPTO rules and procedures set forth in the MPEP, which of the following most accurately describes the propriety of the practitioner’s reply to the May 1st Office action?
A) The notice of appeal is not a proper response because the claims of the continuing application have not been finally rejected.
B) The notice of appeal is not a proper reply because all of the claims in the continuing application have not been twice rejected.
C) The filing of a notice of appeal is not a proper reply because not all the claims in the continuing application have been twice rejected.
D) A notice of appeal is never a proper response to a non-final rejection.
E) The reply is proper.

A

E) The reply is proper.

Reasoning: MPEP § 1204, under the heading “Appeal By Patent Applicant,” states that “[a] notice of appeal may be filed after any of the claims has been twice rejected, regardless of whether the claim(s) has/have been finally rejected. The limitation of ‘twice or finally…rejected’ does not have to be related to a particular application. For example, if any claim was rejected in a parent application, and the claim is again rejected in a continuing application, then applicant will be entitled to file an appeal in the continuing application, even if the claim was rejected only once in the continuing application.”

138
Q
A
139
Q

A registered practitioner filed a patent application naming Sam as the sole inventor without an executed declaration under 37 CFR 1.63. The USPTO mailed a Notice to File Missing Parts dated January 3, 2016. The Notice to File Missing Parts set a two-month period for reply. Which of the following statements is in accordance with proper USPTO rules and the procedure set forth in the MPEP?

I. Submit an appropriate reply to the Notice to File Missing Parts by filing, on August 3, 2016, a declaration under 37 CFR 1.63 executed by Sam, accompanied by a petition under 37 CFR 1.136(a) for an extension of five months, and the fee set forth in 37 CFR 1.17(a).

II. In no situation can any extension requested by the practitioner carry the date on which a reply is due to the Notice to File Missing Parts beyond Monday, July 3, 2016.

III. An appropriate reply by the practitioner to the Notice to File Missing Parts is to file, on August 3, 2016 a declaration under 37 CFR 1.63 executed by Sam, accompanied by a petition under 37 CFR 1.136(b).
A) I
B) II
C) III
D) I and III
E) None of the above.

A
140
Q

Which of the following requests by the registered practitioner of record for an interview with an examiner concerning an application will be granted in accordance with proper USPTO rules and procedure?
A) A request for an interview in a substitute application prior to the first Office action, for the examiner and attorney of record to meet in the practitioner’s office without the authority of the Commissioner.
B) A request for an interview in a continued prosecution application prior to the first Office action, to be held in the examiner’s office.
C) A request for an interview in a non-continuing and non-substitute application, prior to the first Office action to be held in the examiner’s office.
D) All of the above.
Cross Out Answer
E) None of the above.

A

B) A request for an interview in a continued prosecution application prior to the first Office action, to be held in the examiner’s office.

Reasoning: 37 CFR § 1.133; MPEP § 713.02. As stated in MPEP § 713.02, “[a] request for an interview prior to the first Office action is ordinarily granted in continuing or substitute applications. A request for an interview in all other applications before the first action is untimely and will not be acknowledged if written, or granted if oral. 37 CFR 1.133(a).”

A continued prosecution application counts as a continued application. Duh.

141
Q

In accordance with the USPTO rules and the procedures set forth in the MPEP, which of the following documents, if any, must also contain a separate verification statement?
A) Small entity statements.
B) A petition to make an application special.
C) A claim for foreign priority.
D) An English translation of a non-English language document.
E) None of the above.

A

E) None of the above.

Reasoning: MPEP § 410 makes clear that the certification requirement set forth in 37 CFR § 10.18(b) “has permitted the PTO to eliminate the separate verification requirement previously contained in 37 CFR …1.27 [small entity statements], …1.52 [English translations of non-English documents], …1.55 [claim for foreign priority], [and] …1.102 [petition to make an application special].”

142
Q

Inventor files an application containing the following original Claim 1:

  1. A widget comprising element A, and element B.

In a first Office action on the merits, a primary examiner rejects claim 1 under 35 USC 103 as being obvious over reference X. Reference X explicitly discloses a widget having element A, but it does not disclose element B. The examiner, however, takes official notice of the fact that element B is commonly associated with element A in the art and on that basis concludes that it would have been obvious to provide element B in the reference X widget. In reply to the Office action, the registered practitioner representing the applicant makes no amendments, but instead requests reconsideration of the rejection by demanding that examiner show proof that element B is commonly associated with element A in the art. Which of the following actions, if taken by the examiner in the next Office action would be in accord with the USPTO rules and the procedures set forth in the MPEP?

I. Vacate the rejection and allow the claim.

II. Cite a reference that teaches element B commonly associated with element A in the art and make the rejection final.

III. Deny entry of applicant’s request for reconsideration on the ground that it is not responsive to the rejection and allow applicant time to submit a responsive amendment.
A) I and II only.
B) II only.
C) II and III only.
D) I, II, and III.
E) I and III only.

A

A) I and II only.

Reasoning: MPEP § 2144.03 provides that when an applicant seasonably traverses an officially noticed fact, the examiner may cite a reference teaching the noticed fact and make the next action final. Here, applicant did seasonably traverse the noticed fact by demanding proof in response to the rejection. II is therefore an appropriate action by the examiner. I is also an appropriate action because the examiner should vacate a rejection based on official notice if no support for the noticed fact can be found in response to a challenge by the applicant. See In re Ahlert, 424 F.2d 1088, 1091 (C.C.P.A. 1970) (“[a]ssertions of technical facts in areas of esoteric technology must always be supported by citation to some reference work” and “[a]llegations concerning specific “knowledge” of the prior art, which might be peculiar to a particular art should also be supported”). (B) is incorrect because (A) is correct.

143
Q

Applicant properly appealed the primary examiner’s final rejection of the claims to the Board of Patent Appeals and Interferences (Board). Claims 1 to 10 were pending in the application. The examiner did not reject the subject matter of claims 7 to 10, but objected to these claims as being dependent on a rejected base claim. Claim 1 was the sole independent claim and the remaining claims, 2 through 10, were either directly or indirectly dependent thereon. After a thorough review of Appellant’s brief and the examiner’s answer, the Board affirmed the rejection of claims 1 to 6. In accordance with the USPTO rules and the procedures set forth in the MPEP, which of the following is the appropriate action for the examiner to take upon return of the application to his jurisdiction when the time for appellant to take further action has expired?
A) Abandon the application since the Board affirmed the rejection of independent claim 1.
B) Convert the dependent claims 7 to 10 into independent form by examiner’s amendment, cancel claims 1 to 6, and allow the application.
C) Mail an Office action to applicant setting a 1-month time limit in which the applicant may rewrite dependent claims 7 to 10 in independent form. If no timely reply is received, the examiner should amend the objected to claims, 7 to 10, and allow the allow the application.
D) Mail an Office action to applicant with a new rejection of claims 7 to 10 based on the Board’s decision.
E) No action should be taken by the examiner since the Board affirmed the rejection of independent claim 1, the application was abandoned on the date the Board decision was mailed.

A

A) Abandon the application since the Board affirmed the rejection of independent claim 1.

Reasoning: MPEP § 1214.06, under the heading “Examiner Sustained in Whole or in Part.” Under the heading “No Claims Stand Allowed” it states “Claims indicated as allowable prior to appeal except for their dependency from rejected claims will be treated as if they were rejected.” (B) and (C) are not the most correct answers. These options would apply to applications where the Board reversed the rejection of the dependent claims and affirmed the rejection of the independent claim. (D) is not correct. The Board does not render a decision on objected to claims. See 37 CFR § Part 41. (E) is not correct because the mailing of a Board decision does not abandoned an application. See 37 CFR § Part 41

144
Q

In accordance with the USPTO rules and the procedures set forth in the MPEP, which of the following is true?
A) Once the issue fee has become due, provided an original application has not been pending more than three years, the applicant may request and the Office may grant a request for deferral of payment of the issue fee.
B) The time period set for the payment of the issue fee is statutory and cannot be extended. However, if payment is not timely made and the delay in making the payment is unintentional, upon payment of a fee for delayed payment, it may be accepted as though no abandonment had occurred, but there will be a reduction on the patent term adjustment for the period of abandonment.
C) Upon written request, a person citing patents and printed publications to the Office that the person believes has a bearing on the patentability of a particular patent, may request that his or her name remain confidential.
D) To obtain benefit of priority based on an earlier filed U.S. patent application, an applicant in a later filed continuation application is not required to meet the conditions and requirements of 35 USC 120.
E) Each of statements (B) and (C) is true.

A

E) Each of statements (B) and (C) is true.

Reasoning: As to (B), see 35 U.S.C. §§ 151; 154(b)(2)(ii) and (iii); 37 CFR § 1.704(c)(3); MPEP § 1306. As to (C) see MPEP §§ 2203 and 2212. As to (D), the claim for priority is not required, as a person may not wish to do so in order to increase the term of his or her patent. As to (A) deferral under 37 CFR § 1.103 is not available following the notice of allowance. Since (B) and (C) are correct, (E) is the best answer.

145
Q

Regarding a power of attorney or authorization of agent in a patent application, which of the following is in accordance with the USPTO rules and the procedure set forth in the MPEP?
A) All notices and official letters for the patent owner or owners in a reexamination proceeding will be directed to the attorney or agent of record in the patent file at the address listed on the register of patent attorneys and agents.
B) Powers of attorney to firms submitted in applications filed in the year 2016 are recognized by the USPTO.
C) The associate attorney may appoint another attorney.
D) The filing and recording of an assignment will operate as a revocation of a power or authorization previously given.
E) Revocation of the power of the principal attorney or agent does not revoke powers granted by him or her to other attorneys or agents.

A

A) All notices and official letters for the patent owner or owners in a reexamination proceeding will be directed to the attorney or agent of record in the patent file at the address listed on the register of patent attorneys and agents.

Reasoning: See 37 CFR § 1.33(c)

146
Q

A claim in an application recites “[a] composition containing: (a) 35-55% polypropylene; and (b) 45-65% polyethylene.” The sole prior art reference describes, as the only relevant disclosure, a composition containing 34.9% polypropylene and 65.1% polyethylene. In accordance with USPTO rules and procedures set forth in the MPEP, the primary examiner should properly:
A) Indicate the claim allowable over the prior art because there is no teaching, motivation or suggestion to increase the amount of polypropylene from 34.9% to 35% and decrease the amount of polyethylene from 65.1% to 65%.
B) Reject the claim under 35 USC 102 as anticipated by the prior art reference.
C) Reject the claim under 35 USC 103 as obvious over the prior art reference.
D) Reject the claim alternatively under 35 USC 102 as anticipated by or under 35 USC 103 as obvious over the prior art reference.
E) None of the above.

A

C) Reject the claim under 35 USC 103 as obvious over the prior art reference.

Reasoning: A prima facie case of obviousness exists where the claimed ranges and the prior art are close enough that one of ordinary skill in the art would have expected them to have the same properties. See MPEP § 2144.05. In Titanium Metals Corp. v. Banner, 778 F.2d 775, 783, 227 USPQ 773, 779 (Fed. Cir. 1985), a claim recited a titanium base alloy consisting essentially of 0.8% nickel, 0.3% molybdenum, up to 0.1% maximum iron, and the balance titanium. A prior art reference described two similar alloys: (i) one with 0.25% molybdenum, 0.75% nickel, and balance titanium; and (ii) another with 0.31% molybdenum, 0.94% nickel, and balance titanium. The court held: As admitted by appellee’s affidavit evidence from James A. Hall, the Russian article discloses two alloys having compositions very close to that of claim 3, which is 0.3% Mo and 0.8% Ni, balance titanium. The two alloys in the prior art have 0.25% Mo-0.75% Ni and 0.31% Mo-0.94% Ni, respectively. The proportions are so close that prima facie one skilled in the art would have expected them to have the same properties. Appellee produced no evidence to rebut that prima facie case. The specific alloy of claim 3 must therefore be considered to have been obvious from known alloys. Id. Thus, (A) is incorrect. (B) and (D) are incorrect because a claim is anticipated by a prior art reference only when the prior art discloses, either expressly or inherently, every limitation of the claimed invention. (E) is incorrect because (C) is correct.

147
Q

In accordance with the USPTO rules and the procedures set forth in the MPEP, which of the following does not constitute probative evidence of commercial success to support a contention of non-obviousness?
A) In a utility case, gross sales figures accompanied by evidence as to market share.
B) In a utility case, gross sales figures accompanied by evidence as to the time period during which the product was sold.
C) In a utility case, gross sales figures accompanied by evidence as to what sales would normally be expected in the market.
D) In a utility case, gross sales figures accompanied by evidence of brand name recognition.
E) In a design case, evidence of commercial success clearly attributable to the design, and not to improved performance of the device.

A

D) In a utility case, gross sales figures accompanied by evidence of brand name recognition.

Reasoning: Gross sales figures must be measured against a logical standard in order to determine whether or not there is commercial success. The recitations of accompanying evidence in (A), (B), and (C) are logical in that they provide a comparative basis for determining commercial success. (D), on the other hand, recites accompanying evidence which is illogical in that it does not provide a comparative basis for determining commercial success. (E) is wrong because it provides a logical basis for attributing commercial success to the design of the device, rather than the utilitarian function of the device. MPEP § 716.03(b).

148
Q

In accordance with the USPTO rules and the procedures in the MPEP, in which of the following instances is the reference properly available as prior art under 35 USC 102?
A) A U.S. patent application is filed within the one year anniversary date of the filing date of the foreign application. The reference is the foreign application.
B) The applicant files a foreign application, later timely files a U.S. application claiming priority based on the foreign application, and then files a continuation-in-part (CIP) application, and the claims in the CIP are not entitled to the filing date of the U.S. parent application. The foreign application issues as a patent before the filing date of the CIP application and is used to reject the claims directed to the added subject matter under 35 USC 102/103. The reference is the foreign application.
C) The applicant files a foreign application, and later timely files a U.S. application claiming priority based on the foreign application. The examined foreign application has been allowed by the examiner and has not been published before the U.S. application was filed.
D) The cited date is the filing date and the reference is a defensive publication.
E) All of the above.

A

B) The applicant files a foreign application, later timely files a U.S. application claiming priority based on the foreign application, and then files a continuation-in-part (CIP) application, and the claims in the CIP are not entitled to the filing date of the U.S. parent application. The foreign application issues as a patent before the filing date of the CIP application and is used to reject the claims directed to the added subject matter under 35 USC 102/103. The reference is the foreign application.

Reasoning: See 35 U.S.C. § 102; MPEP § 2135.01, under the heading “A Continuation - In - Part Breaks The Chain Of Priority As To Foreign As Well As U.S. Parents.” If an applicant files a foreign application, later files a U.S. application claiming priority based on the foreign application, and then files a continuation - in - part (CIP) application whose claims are not entitled to the filing date of the U.S. parent, the effective filing date of the CIP application is the filing date of the CIP. The applicant cannot obtain the benefit of either the U.S. parent or foreign application filing dates. In re van Langenhoven, 173 USPQ 426, 429 (CCPA 1972); Ex parte Appeal No. 242 - 47, 196 USPQ 828 (Bd. App. 1976).

(A) is incorrect. 35 U.S.C. § 102.

(C) is not correct. 35 U.S.C. § 102.

(D) is not correct. 35 U.S.C. § 102(d); MPEP § 2136, under the heading “Defensive Publications Are Not Prior Art As Of Their Filing Date,” citing Ex parte Osmond, 191 USPQ 334 (Bd. App. 1973).

(E) is not correct inasmuch as (A), (C) and (D) are not correct.

149
Q

The Office mailed an Office action containing a proper final rejection dated July 8, 2016. The Office action did not set a period for reply. On January 7, 2017, in reply to the final rejection, a registered practitioner filed a request for continued examination under 37 CFR 1.114, a request for a suspension of action under 37 CFR 1.103(c) to suspend action for three months, and proper payment all required fees. No submission in reply to the outstanding Office action accompanied the request for continued examination. No other paper was submitted and no communication with the Office was held until after Midnight, January 8, 2017. Which of the following statements accords with the USPTO rules and the procedures set forth in the MPEP?
A) If an appropriate reply is submitted within the three month period of suspension permitted under 37 CFR 1.103(c), the application will not be held abandoned.
B) The application will not be held abandoned if an appropriate reply is submitted within the three month period of suspension and it is accompanied by a showing that the reply could not have been submitted within the period set in the final rejection. For example, the reply includes a showing based on an experiment that required 8 months to conduct.
C) No reply will prevent the application from being held abandoned.
D) If, on January 10, 2017, the primary examiner and applicant agree to an examiner’s amendment that places the application in condition for allowance and a notice of allowance is mailed within the three month period of suspension, application X will not be held abandoned.
E) No other submission by applicant is necessary because application X is still pending. The examiner is required to act on the request for continued examination after expiration of the three month period of suspension.

A

C) No reply will prevent the application from being held abandoned.

Reasoning: As stated in MPEP § 709, under the heading “Request By The Applicant,” subheading “Request for Suspension Under 37 CFR 1.103(b) or (c),” “The Office will not grant the requested suspension of action unless the following requirements are met: (A) the request must be filed with the filing of a CPA or an RCE…(1) if the request is filed with an RCE, the RCE must be in compliance with 37 CFR 1.114, i.e., the RCE must be accompanied by a submission and the fee set forth in 37 CFR 1.17(e). Note that the payment of the RCE filing fee may not be deferred and the request for suspension cannot substitute for the submission.” The RCE was improper because no submission in reply to the outstanding Office action accompanied the RCE. Since the RCE was improper, the Office will not recognize the request for suspension. The time period set in the final rejection continues to run from the mail date of the Office action. Since the Office action did not set a period for reply, applicant has a maximum period of six months for reply. A reply was due on January 8, 2017. Since the RCE was improper and the Office did not recognize the request for suspension, the application became abandoned at Midnight of January 8, 2017.

(A), (B) and (E) are not correct. As stated in MPEP § 706.07(h), under the heading “Submission Requirement,” “If a reply to an Office action under 35 U.S.C. 132 is outstanding, the submission must meet the reply requirements of 37 CFR 1.111. See 37 CFR 1.114(c).” An RCE that is not accompanied by a submission is an improper RCE. As stated in MPEP § 706.07(h), under the heading “Initial Processing,” subheading “Treatment of Improper RCE,” “An improper RCE will not operate to toll the running of any time period set in the previous Office action for reply to avoid abandonment of the application.” The period for filing a proper reply was six months inasmuch as no shortened statutory period for reply was set. A proper reply to the final rejection was not filed. Therefore, the application went abandoned for failure to file a proper reply to the final rejection.

(D) is not correct. As set forth in MPEP § 706.07(f) under the heading “Examiner’s Amendments,” paragraph (H), “[a]n examiner’s amendment may not be made more than 6 months from the date of the final Office action, as the application would be abandoned at that point by operation of law.”

150
Q

In accordance with the USPTO rules and the procedures set forth in the MPEP, which of the following is true?
A) Interferences will generally be declared even when the applications involved are owned by the same assignee since only one patent may issue for any given invention.
B) A senior party in an interference is necessarily the party who obtains the earliest actual filing date in the USPTO.
C) Reexamination proceedings may not be merged with reissue applications since third parties are not permitted in reissue applications.
D) After a reexamination proceeding is terminated and the certificate has issued, any member of the public may obtain a copy of the certificate by ordering a copy of the patent.
E) None of the above.

A

D) After a reexamination proceeding is terminated and the certificate has issued, any member of the public may obtain a copy of the certificate by ordering a copy of the patent.

Reasoning: MPEP 2292: Distribution of Certificate

151
Q

The claims in a patent application having been twice or finally rejected, the applicant files a timely Notice of Appeal on January 2, 2016. In accordance with USPTO rules and procedures set forth in the MPEP, which of the following situations should the USPTO not notify the applicant that the Appeal Brief is defective and allow him an opportunity to correct the deficiency?
A) The Appeal Brief is filed on July 10, 2016, without a request for extension of time under 37 CFR 1.136.
B) The Appeal Brief is submitted unsigned.
C) The Appeal Brief states that the claims do not stand or fall together, and presents argument as to why the claims are separately patentable, but the primary examiner does not agree with the applicant’s argument.
D) The Appeal Brief does not state whether the claims stand or fall together, but presents arguments why the claims subject to the same rejection are separately patentable.
E) The Appeal Brief does not address any of the grounds of rejection stated by the primary examiner, has no argument section, and is non-compliant as to 37 CFR 41.37.

A

C) The Appeal Brief states that the claims do not stand or fall together, and presents argument as to why the claims are separately patentable, but the primary examiner does not agree with the applicant’s argument.

Reasoning: See MPEP § 1207. If the examiner disagrees with the reasons given, the reason for disagreement should be addressed in the Examiner’s Answer. If the brief includes a statement that a grouping of claims does not stand or fall together but does not provide reasons, [the examiner is to] notify appellant of the non-compliance. If the examiner disagrees with appellant’s statement in the brief that certain claims do not stand or fall together, the examiner explains in the examiner’s answer why the claim grouping listed in the brief is not agreed with and why, if appropriate, e.g., the claims as listed by the appellant are not separately patentable.

Answer (A) is incorrect. The Appeal Brief was filed less than seven months after the Notice of Appeal was filed. The applicant should be notified of the deficiency and provided an opportunity to request a five-month extension of time.

152
Q

The specification in a patent application has been objected to for lack of enablement. To overcome this objection in accordance with the USPTO rules and the procedures set forth in the MPEP, a registered practitioner may do any of the following except:
A) traverse the objection and specifically argue how the specification is enabling.
B) traverse the objection and submit an additional drawing to make the specification enabling.
C) file a continuation-in-part application that has an enabling specification.
D) traverse the objection and file an amendment without adding new matter in an attempt to show enablement.
E) traverse the objection and refer to prior art cited in the specification that would demonstrate that the specification is enabling to one of ordinary skill.

A

B) traverse the objection and submit an additional drawing to make the specification enabling.

Reasoning: 35 U.S.C. § 113 reads “Drawings submitted after the filing date of the application may not be used (i) to overcome any insufficiency of the specification due to lack of an enabling disclosure.” Since choice (A) may be done, 37 CFR § 1.111, it is an incorrect answer to the above question. Since choice (C) may be done, 35 U.S.C. § 120, it is an incorrect answer to the above question. Since choice (D) may be done, 37 CFR § 1.121, it is an incorrect answer to the above question. Since choice (E) may be done, 37 CFR § 1.111, it also is an incorrect answer to the above question.

153
Q

In accordance with the USPTO rules and the procedures set forth in the MPEP, which of the following is true?
A) When the subject matter of an appeal is particularly difficult to understand, a patentability report is prepared by an examiner in order to present the technical background of the case to the Board of Patent Appeals and Interferences.
B) In those appeals in which an oral hearing has been confirmed and either the Board of Patent Appeals and Interferences or the primary examiner has indicated a desire for the examiner to participate in the oral argument, oral argument may be presented by the examiner whether or not the appellant appears.
C) If a patent applicant files a notice of appeal which is unsigned, it will be returned for signature, but the applicant will still receive the filing date of the unsigned notice of appeal.
D) Statements made in information disclosure statements are not binding on an applicant once the patent has issued since the sole purpose of the statement is to satisfy the duty of disclosure before the Office.
E) None of the above.

A

B) In those appeals in which an oral hearing has been confirmed and either the Board of Patent Appeals and Interferences or the primary examiner has indicated a desire for the examiner to participate in the oral argument, oral argument may be presented by the examiner whether or not the appellant appears.

Reasoning: See MPEP § 1209, under the heading “Participation by Examiner.” As to (A), see MPEP § 705. As to (C) signature requirement does not apply. 37 C.F.R. § 1.196(b); MPEP § 1205. The notice will not be returned. As to (D), see Gentry Gallery v. Berkline Corp., 134 F.3d 1473, 45 U.S.P.Q.2d 1498 (Fed. Cir. 1998).

154
Q

On January 2, 2016, a registered practitioner filed a patent application with the USPTO for inventor Bloc. The application includes a specification and a single claim to the invention which reads as follows: 1. Compound Y. In the specification, Bloc explains that compound Y is an intermediate in the chemical manufacture of synthetic Z. With respect to synthetic Z, the specification discloses its structural formula and further states that synthetic Z is modeled on the natural form of Z to give it the same therapeutic ability to alleviate pain. The specification goes on to state that synthetic Z is also a cure for cancer. On June 2, 2016, the practitioner received an Office action from the primary examiner rejecting the claim. The claim is rejected under 35 USC 101 as being inoperative; that is, the synthetic Z does not operate to produce a cure for cancer (i.e., incredible utility). Bloc believes he is entitled to a patent to his compound Y. In accordance with USPTO rules and procedures set forth in the MPEP, how best should the practitioner reply to the rejection of the claim?
A) Advise Bloc that he should give up because a cure for cancer is indeed incredible and is unproven.
B) File a reply arguing that a cure for cancer is not incredible and he can prove it if given the chance.
C) File a reply arguing that whether or not a cure for cancer is incredible is superfluous since Bloc has disclosed another utility - alleviating pain, which is not incredible.
D) File a reply arguing that the claim is directed to compound Y, not synthetic Z.
E) File a reply arguing that synthetic Z is modeled on the natural form of Z.

A

C) File a reply arguing that whether or not a cure for cancer is incredible is superfluous since Bloc has disclosed another utility - alleviating pain, which is not incredible.

Reasoning:

155
Q

Which of the following is not a policy underlying the public use bar of 35 USC 102?
A) Discouraging the removal, from the public domain, of inventions that the public reasonably has come to believe are freely available.
B) Favoring the prompt and widespread disclosure of inventions.
C) Allowing the inventor(s) a reasonable amount of time following sales activity to determine the potential economic value of a patent.
D) Increasing the economic value of a patent by extending the effective term of the patent up to one year.
E) Prohibiting the inventor(s) from commercially exploiting the invention for a period greater than the statutorily prescribed time.

A

D) Increasing the economic value of a patent by extending the effective term of the patent up to one year.

Extending patent term is not a policy underlying any section of 35 U.S.C. § 102. Answers (A), (B), (C) and (E) do state policies underlying the public use bar. Lough v. Brunswick Corp., 86 F.3d 1113, 39 USPQ2d 1100 (Fed. Cir. 1996).

155
Q

Al files an application for a patent. After the Notice of Allowance is mailed and the issue fee has been paid Al discovers a prior art reference which is material to patentability. What should Al do in accordance with the USPTO rules and the procedures set forth in the MPEP?
A) Al should file a prior art statement under 37 CFR 1.501 that will be placed in the patent file upon issuance of the application as a patent.
B) Since the issue fee has been paid, Al no longer has a duty to disclose to the Office material prior art. He is under no obligation to submit the prior art reference to the Office.
C) Since the issue fee has been paid, it is too late to have the examiner consider the reference in this application. Al should file a continuation application to have the reference considered and allow the original patent application to issue as a patent.
D) Al should file a petition to have the application withdrawn from issuance, citing the finding of additional material prior art as the reason for withdrawal. A continuation application should also be filed with an information disclosure statement containing the reference in order to have the reference considered.
E) Al should file an amendment under 37 CFR. 1.312 deleting all of the claims which are unpatentable over the reference since an amendment deleting claims is entitled to entry as a matter of right.

A

D) Al should file a petition to have the application withdrawn from issuance, citing the finding of additional material prior art as the reason for withdrawal. A continuation application should also be filed with an information disclosure statement containing the reference in order to have the reference considered.

Reasoning: You didn’t trust your gut

155
Q

To satisfy the written description requirement of the first paragraph of 35 USC 112, an applicant must show possession of the invention. An applicant’s lack of possession of the invention may be evidenced by:

A) Describing an actual reduction to practice of the claimed invention.

B) Describing the claimed invention with all of its limitations using such descriptive means as words, structures, figures, diagrams, and formulas that fully set forth the claimed invention.

C) Requiring an essential feature in the original claims, where the feature is not described in the specification or the claims, and is not conventional in the art or known to one of ordinary skill in the art.

D) Amending a claim to add a limitation that is supported in the specification through implicit or inherent disclosure.

E) Amending a claim to correct an obvious error by the appropriate correction.

A

C) Requiring an essential feature in the original claims, where the feature is not described in the specification or the claims, and is not conventional in the art or known to one of ordinary skill in the art.

Reasoning: The question is asking “THE LACK OF POSSESSION OF THE INVENTION MAY BE EVIDENCED BY:”. You read it as if it is asking how to satisfy the 35 USC 112 first paragraph requirement.

See, “Guidelines for Examination of Patent Applications under 35 U.S.C. § 112, ¶ 1, ‘Written Description’ Requirement,” MPEP 2163. “The claimed invention as a whole may not be adequately described if the claims require an essential or critical feature that is not described in the specification and is not conventional in the art or known to one of ordinary skill in the art.”

155
Q

An application as originally filed contains the following Claim 1:

Claim 1. A doughnut making machine comprising:

(i) an input conveyor that receives dough to be used in making said doughnuts;

(ii) means for portioning dough from said input conveyor into a plurality of dough balls, each of said plurality of balls containing dough sufficient to create a single doughnut;

(iii) means for forming each of said dough balls into a ring of dough;

(iv) a deep fat fryer which receives rings of dough from said forming means and cooks said rings of dough;

(v) means for selectively applying a flavored coating on cooked rings of dough to produce doughnuts; and

(vi) means for placing a plurality of said doughnuts on a flat sheet.

The specification adequately describes the claimed subject matter. Two different “means for selectively applying” are described in the specification: a sprayer and a brush. Which of the following original claims is an improper dependent claim?

A) Claim 2. The doughnut making machine of Claim 1, wherein said placing means is a conveyor that extends from said applying means to said flat sheet.

B) Claim 3. The doughnut making machine of Claim 1, wherein said forming means includes a cutter that removes a center portion of each of said dough balls to form a ring of dough.

C) Claim 4. The doughnut making machine of Claim 1, wherein said applying means includes a sprayer which receives a sugar based flavored coating, wherein said sugar based flavored coating is sprayed on said cooked rings of dough.

D) Claim 5. The doughnut making machine of Claim 1, wherein said applying means is a sprayer.

E) Claim 6. The doughnut making machine of Claim 1, wherein said applying means is omitted for making plain doughnuts.

A

E) Claim 6. The doughnut making machine of Claim 1, wherein said applying means is omitted for making plain doughnuts.

Reasoning: A dependent claim must further limit the claim from which it depends. 35 U.S.C. § 112, ¶4; 37 C.F.R. § 1.75(c). Dependent claim 6 (Answer E) improperly seeks to broaden Claim 1 by omitting an element set forth in the parent claim.

156
Q

John filed a nonprovisional patent application in the USPTO claiming two distinct inventions, a combination and a subcombination. At the time of filing the nonprovisional application, he recorded an assignment of all right, title, and interest in the inventions claimed in the application to ABC Corporation. In the first Office action, the examiner required restriction, and John elected the combination. A year later, during the pendency of the nonprovisional application, John filed a divisional patent application claiming the subcombination. At the time of filing the divisional application, John assigned all right, title, and interest in the inventions claimed in the divisional application to XYZ Corporation, and the latter party recorded the assignment within three months of the assignment. Following recordation of the assignment to XYZ Corporation, which of the following statements is false?

A) The Office should treat John as having no ownership rights in the combination.

B) The Office should treat John as having no ownership rights in the subcombination.

C) ABC Corporation has no ownership rights in the subcombination.

D) XYZ Corporation has no ownership rights in the combination.

E) XYZ Corporation has no ownership rights in the subcombination.

A

C) ABC Corporation has no ownership rights in the subcombination.

Reasoning: John signed over the rights of the combination and subcombination to the ABC corporation. Under 35 U.S.C. § 261, “An assignment, grant, or conveyance shall be void as against any subsequent purchaser or mortgagee for a valuable consideration, without notice, unless it is recorded in the Patent and Trademark Office within three months from its date or prior to the date of such subsequent purchase or mortgage.” John signed over the subcombination to the XYZ Corporation 1 year later! Which means the XYZ assignment is null. Thus, ABC owns the combination and the subcombination. That makes C) the false statement.

157
Q

The USPTO notifies John, a registered patent agent who is representing applicant A, that after a reasonable search, the USPTO has been unable to locate applicant A’s patent application. By which of the following procedures may John avoid abandonment of applicant A’s application within the time period set by the USPTO?

A) Provide the USPTO with a copy of his record of all the correspondence between his office and the USPTO, assuming the existence of such record.

B) Provide the USPTO with a list of all the correspondence between his office and the USPTO, assuming the existence of such list, and a statement that the list is complete and accurate.

C) Provide the USPTO with a statement that he does not possess any record of the correspondence between his office and the USPTO because his files were destroyed.

D) Provide the USPTO with a record of all the correspondence between his office and the USPTO, and a statement that the papers produced are his complete record of all the correspondence between his office and the USPTO, assuming the existence of such record.

E) Provide the USPTO with a copy of his record of all the correspondence between his office and the USPTO, assuming the existence of such record, a list of all such correspondence, and a statement that he is not aware of any correspondence between his office and the USPTO that is not among his records.

A

C) Provide the USPTO with a statement that he does not possess any record of the correspondence between his office and the USPTO because his files were destroyed.

Reasoning: (C) is correct because there is compliance with 37 C.F.R. § 1.251(a)(3). (A) is wrong because along with a copy of the record, he is required to provide a list of all correspondence, and a statement that the copy is complete and accurate and that he is not aware of any correspondence between his office and the USPTO that is not among his records. 37 C.F.R. § 1.251(a)(1)(ii). (B) is wrong because along with a list of all correspondence and a statement that the copy of his record of all the correspondence is complete and accurate, he is required to provide a copy of his record of all the correspondence, and the statement must recite that he is not aware of any correspondence between his office and the USPTO that is not among his records. 37 C.F.R. § 1.251(a)(1)(i). (D) is wrong because the statement omits the recitation that he is not aware of any correspondence between his office and the USPTO that is not among his records. 37 C.F.R. § 1.251(a)(2)(ii). (E) is wrong because the statement omits the recitation that the copy of his record of all the correspondence is complete and accurate. 37 C.F.R. § 1.251(a)(1)(iii).

158
Q

On Monday, May 13, 2022, John’s secretary deposited in a “Priority Express Mail” drop box prior to the last scheduled pick-up for that day, an envelope properly addressed to the USPTO for delivery to the USPTO by the “Express Mail Post Office to Addressee” service. The envelope was received by the USPTO on Wednesday, May 15, 2022, containing a reply to an Office action which set a shortened statutory period (“SSP”) for reply ending on Tuesday, May 14, 2022. The reply was marked by the Office as being received on May 15, 2022. The number of the “Express Mail” mailing label had not been placed on the response papers, and upon receipt of the “Express Mail” mailing label John learned that the “date in” was not clearly marked. John promptly filed a petition requesting the filing date to be the date of deposit. The petition included a showing that the date of deposit accompanied by evidence of USPS corroboration of the deposit. Accordingly,

A) The reply will be regarded as timely filed in the USPTO on May 15, 2022.

B) The reply will be regarded as timely filed in the USPTO on May 14, 2022.

C) The reply will be regarded as timely filed in the USPTO on May 13, 2022.

D) The reply will be regarded as timely filed in the USPTO if a petition with proper fee for a one month extension of time is filed in the USPTO on or before June 14, 2022.

E) The reply will be regarded as timely filed in the USPTO if the number of the “Priority Express Mail” mailing label is placed on each page of a copy of the original response and hand carried to the USPTO on May 15, 2022, rather than being sent by “Priority Express Mail.”

A

D) The reply will be regarded as timely filed in the USPTO if a petition with proper fee for a one month extension of time is filed in the USPTO on or before June 14, 2022.

Reasoning:37 C.F.R. § 1.136(a). 37 C.F.R. § 1.136(a) states, “[A]pplicant may extend the time period for reply up to the earlier of the expiration of any maximum period set by statute or five months after the time period set for reply, if a petition for an extension of time and the fee set in § 1.17(a) are filed…”

(A) is wrong because the response was not timely filed since it was received by the USPTO after the SSP expired.

(B) and (C) are wrong. The reply was not filed on May 14, 2022, because the conditions of 37 C.F.R. § 1.10(b) were not satisfied. For example, the number of the “Priority Express Mail” mailing label must have been placed on each page of the response prior to the original mailing by “Priority Express Mail.” The petition should not be expected to be granted inasmuch as the papers did not include the number of the “Priority Express Mail” mailing label on them. See § 1.10(c)(2), (d)(2), and (e)(2).

(E) is wrong because 37 C.F.R. § 1.10(b) requires that “the number of the ‘Priority Express Mail’ mailing label must have been placed on each page of the response prior to the original mailing by ‘Priority Express Mail.’” [emphasis added]

159
Q

The specification in your client’s patent application has been objected to for lack of enablement. To overcome this objection, your client may do any of the following except:
A) traverse the objection and specifically argue how the specification is enabling.
B) traverse the objection and submit an additional drawing to make the specification enabling.
C) file a continuation-in-part application that has an enabling specification.
D) traverse the objection and file an amendment without adding new matter in an attempt to show enablement.
E) traverse the objection and refer to prior art cited in the specification that would demonstrate that the specification is enabling to one of ordinary skill.

A

B) traverse the objection and submit an additional drawing to make the specification enabling.

Reasoning: 35 U.S.C. § 113 reads “Drawings submitted after the filing date of the application may not be used (i) to overcome any insufficiency of the specification due to lack of an enabling disclosure.” Since choice (A) may be done, 37 C.F.R. § 1.111, it is an incorrect answer to the above question. Since choice (C) may be done, 35 U.S.C. § 120, it is an incorrect answer to the above question. Since choice (D) may be done, 37 C.F.R. § 1.121, it is an incorrect answer to the above question. Since choice (E) may be done, 37 C.F.R. § 1.111, it also is an incorrect answer to the above question.

159
Q

Adams filed Application X on March 1, 2021. Beth filed application Y on May 1, 2021. Neither application has been published. Applications X and Y are copending and commonly assigned. Earlier filed application X claims the same invention as claimed in application Y using identical language. In accordance with the MPEP, which of the following actions should the examiner or assignee follow?
A) The claims to the same invention in application Y should be rejected under 35 USC 102(a)(1) as being anticipated by application X.
B) The claims to the same invention in application Y should be rejected under 35 USC 102(a)(2) as being anticipated by application X.
C) The claims to the same invention in application Y should be rejected under 35 USC 102(a)(2) as being provisionally anticipated by application X.
D) The common assignee should file a terminal disclaimer in application Y to avoid any question of double patenting.
E) The claims to the same invention in application Y should be rejected under 35 USC 101 as being anticipated by application X.

A

C) The claims to the same invention in application Y should be rejected under 35 USC 102(a)(2) as being provisionally anticipated by application X.

Reasoning: 35 U.S.C. § 102(a)(2); MPEP § 804. (C) is correct because section I. of MPEP § 706.02(f) states, “If (1)…the applications are commonly assigned and (2) the effective filing dates are different, then a provisional rejection of the later filed application should be made.”

(A) is wrong because the facts do not indicate that the application Y is patented or published.

(B) is wrong because the facts do not indicate that the application Y is published.

(D) is wrong. This is a statutory double patenting situation that cannot be avoided by filing a terminal disclaimer. See In re Bartfeld, 17 USPQ2d 1885 (Fed. Cir. 1991). MPEP § 706.02(f), section I; MPEP § 804.02, part I.

(E) is wrong. It is improper to make a nonprovisional rejection under § 102(e) in the circumstances described in the question.